You are on page 1of 215

JULITO SAGALES, Petitioner, - versus - RUSTAN’S COMMERCIAL CORPORATION, Respondent.

THIRD DIVISION , G.R. No. 166554, November 27, 2008


REYES, R.T., J.:

Facts:

Petitioner Julito Sagales was employed by respondent Rustan’s Commercial


Corporation from October 1970 until July 26, 2001, when he was terminated. At the time
of his dismissal, he was occupying the position of Chief Cook at the Yum Yum Tree
Coffee Shop. He was paid a basic monthly salary of P9,880.00. He was also receiving
service charge of not less than P3,000.00 a month and other benefits under the law and
the existing collective bargaining agreement between respondent and his labor union.

In the course of his employment, petitioner was a consistent recipient of numerous


citations for his performance. After receiving his latest award on March 27, 2001,
petitioner conveyed to respondent his intention of retiring on October 31, 2001, after
reaching thirty-one (31) years in service. Petitioner, however, was not allowed to retire
with his honor intacts.

On June 18, 2001, Security Guard Waldo Magtangob, upon instructions from Senior
Guard Bonifacio Aranas, apprehended petitioner in the act of taking out from Rustan’s
Supermarket a plastic bag. Upon examination, it was discovered that the plastic bag
contained 1.335 kilos of squid heads worth P50.00. Petitioner was not able to show any
receipt when confronted. Thus, he was brought to the Security Office of respondent
corporation for proper endorsement to the Makati Headquarters of the Philippine
National Police. Subsequently, petitioner was brought to the Makati Police Criminal
Investigation Division where he was detained. Petitioner was later ordered released
pending further investigation.

On June 19, 2001, petitioner underwent inquest proceedings for qualified theft before
Assistant Prosecutor Amado Y. Pineda. Although petitioner admitted that he was in
possession of the plastic bag containing the squid heads, he denied stealing them
because he actually paid for them. As proof, petitioner presented a receipt. The only
fault he committed was his failure to immediately show the purchase receipt when he
was accosted because he misplaced it when he changed his clothes. He also alleged that
the squid heads were already “scraps” as these were not intended for cooking. Neither
were the squid heads served to customers. He bought the squid heads so that they
could be eaten instead of being thrown away. If he intended to steal from respondent,
he could have stolen other valuable items instead of scrap.
Assistant Prosecutor Pineda believed the version of petitioner and recommended the
dismissal of the case for “lack of evidence.” The recommendation was approved upon
review by the City Prosecutor.

Notwithstanding the dismissal of the complaint, respondent, on June 25, 2001, required
petitioner to explain in writing within forty-eight (48) hours why he should not be
terminated in view of the June 18, 2001 incident. Respondent also placed petitioner
under preventive suspension.

Respondent did not find merit in the explanation of petitioner. Thus, petitioner was
dismissed from service on July 26, 2001. At that time, petitioner had been under
preventive suspension for one (1) month.

Aggrieved, petitioner filed a complaint for illegal dismissal against respondent. He also
prayed for unpaid salaries/wages, overtime pay, as well as moral and exemplary
damages, attorney’s fees, and service charges.

Issues:

(1) Is the evidence on record sufficient to conclude that petitioner committed the crime
charged? and

(2) Assuming that the answer is in the affirmative, is the penalty of dismissal proper?

Held:

1. YES. The evidence on record is sufficient to conclude that petitioner committed the
crime charged.

Security of tenure is a paramount right of every employee that is held sacred by the
Constitution. The reason for this is that labor is deemed to be “property” within the
meaning of constitutional guarantees. Indeed, as it is the policy of the State to guarantee
the right of every worker to security of tenure as an act of social justice, such right
should not be denied on mere speculation of any similar or unclear nebulous basis.
Indeed, the right of every employee to security of tenure is all the more secured by the
Labor Code by providing that “the employer shall not terminate the services of an
employee except for a just cause or when authorized” by law. Otherwise, an employee
who is illegally dismissed “shall be entitled to reinstatement without loss of seniority
rights and other privileges and to his full backwages, inclusive of allowances, and to his
other benefits or their monetary equivalent computed from the time his compensation
was withheld from him up to the time of his actual reinstatement.”
Necessarily then, the employer bears the burden of proof to show the basis of the
termination of the employee.

In the case at bar, respondent has discharged its onus of proving that petitioner
committed the crime charged.

We stress that the quantum of proof required for the application of the loss of trust and
confidence rule is not proof beyond reasonable doubt. It is sufficient that there must
only be some basis for the loss of trust and confidence or that there is reasonable
ground to believe, if not to entertain the moral conviction, that the employee concerned
is responsible for the misconduct and that his participation in the misconduct rendered
him absolutely unworthy of trust and confidence.

It is also of no moment that the criminal complaint for qualified theft against petitioner
was dismissed. It is well settled that the conviction of an employee in a criminal case is
not indispensable to the exercise of the employer’s disciplinary authority.

2. NO. The penalty of dismissal is too harsh under the circumstances. The free will of
management to conduct its own business affairs to achieve its purpose cannot be
denied. The only condition is that the exercise of management prerogatives should not
be done in bad faith or with abuse of discretion. Truly, while the employer has the
inherent right to discipline, including that of dismissing its employees, this prerogative
is subject to the regulation by the State in the exercise of its police power.

In this regard, it is a hornbook doctrine that infractions committed by an employee


should merit only the corresponding penalty demanded by the circumstance. The
penalty must be commensurate with the act, conduct or omission imputed to the
employee and must be imposed in connection with the disciplinary authority of the
employer.

We do not condone dishonesty. After all, honesty is the best policy. However,
punishment should be commensurate with the offense committed. The supreme penalty
of dismissal is the death penalty to the working man. Thus, care should be exercised by
employers in imposing dismissal to erring employees. The penalty of dismissal should
be availed of as a last resort.
SOLVIC INDUSTRIAL CORP. and ANTONIO C. TAM, petitioners, vs. NATIONAL
LABOR RELATIONS COMMISSION and DIOSDADO LAUZ, respondents.
[G.R. No. 125548. September 25, 1998] FIRST DIVISION
PANGANIBAN, J.:

Facts:

Complainant started his employment with respondent sometime in 1977. He occupied


the position as extruder operator. In the course of his employment, he performed his
utmost best, and in fact has never been suspended or reprimanded. On 17 January 1994,
sans cause or due process, he was arbitrarily terminated from service. Additionally,
complainant alleged that he was not paid his service leave pay.

Respondent on the other hand, averred that the complainant who was hired in 1977 was
actually terminated for cause. That the termination of complainant arose from the
incident that transpired on 17 January 1994 at about 7:00 p.m. On said occasion,
complainant upon seeing Foreman Carlos Aberin confronted him and thereafter struck
him in the shoulder beside the neck with a bladed weapon in the process, inflicting
bodily injury on him. That several days after said incident, complainant did not report
for work, hence, was issued a memorandum of preventive suspension.
Correspondingly, Mr. Aberin executed an affidavit and submitted a medical certificate.
Complainant on the other hand, submitted his letter of explanation denying complicity
in the acts imputed to him. Thereafter, a series of administrative investigation was
conducted where complainant refused to give any further statement or explanation.
Subsequently, he was served his letter of termination dated 21 February 1994, which
however, he refused to receive. Relatedly, in a meeting/conference held with the union
officers by Carlos Aberin and Diosdado Lauz, complainant admitted to attempting to
take the life of Mr. Aberin and apologized for the same.

In reply, complainant countered that he never struck Mr. Aberin with a bladed weapon,
and that the incident was not job related, hence cannot serve as basis for termination.
Respondents, on the other hand in reply, argued that complainant was given his day in
court as an investigation was conducted. Moreover, complainant in the course of his
meeting with Mr. Aberin and with the union officers, admitted that he assaulted the
latter and even apologized in exchange for the withdrawal of the criminal case filed
against him.

The NLRC found that the wrong imputed to the private respondent did not merit the
penalty of dismissal. Thus, ordering his reinstatement, but omitting the award of back
wages. It held that the imposition of the supreme penalty of dismissal is not
commensurate with the gravity of the offense he committed.

Issue:

Whether or not the reinstatement of private respondent is proper.

Held:

YES. Petitioner’s arguments are not persuasive. Fighting within work premises may be
deemed a valid ground for the dismissal of an employee. Such act adversely affects the
employer’s interests for it distracts employees, disrupts operations and creates a hostile
work atmosphere. The facts of this case, however, do not justify the dismissal of private
respondent. As found by Respondent NLRC, the infraction was committed outside the
work premises and did not lead to any disruption of work or any hostile environment
in the work premises.

We agree with the NLRC that the acts of private respondent are not so serious as to
warrant the extreme penalty of dismissal. Private respondent was accused of hitting the
victim once with the blunt side of a bolo. Private respondent could have attacked him
with the blade of the weapon, and he could have struck him several times. But he did
not, thus negating any intent on his part to inflict fatal injuries. In fact, the victim merely
sustained a minor abrasion and has since forgiven and reconciled with the private
respondent. If the party most aggrieved -- namely, the foreman -- has already forgiven
the private respondent, then petitioner cannot be more harsh and condemning than the
victim. Besides, no criminal or civil action has been instituted against private
respondent. Furthermore, in his twenty years of service in the company, he has not been
charged with any similar misconduct.

Verily, we do not condone the action of the private respondent. We believe, however,
that the NLRC did not commit grave abuse of discretion in ruling that the penalty of
dismissal was too harsh and not commensurate with the said offense. “Where a penalty
less punitive would suffice, whatever missteps may be committed by labor ought not to
be visited with a consequence so severe.”
In so ruling, we reiterate that an employer’s power to discipline its workers must be
exercised with caution, lest it erode the constitutional guarantee of security of tenure.
This is especially true when the penalty being imposed is dismissal, which leads to
severance of employment ties and the economic dislocation of the employee. Because of
the serious implications of this penalty, “our Labor Code decrees that an employee
cannot be dismissed, except for the most serious causes. The overly concern of our laws
for the welfare of employees is in accord with the social justice philosophy.

ELMER M. MENDOZA, petitioner, vs. RURAL BANK OF LUCBAN, respondent.


G.R. No. 155421 , July 7, 2004, FIRST DIVISION
PANGANIBAN, J.:

Facts:
On April 25, 1999, the Board of Directors of the Rural Bank of Lucban, Inc., issued
several Board Resolutions which states that all officers and employees are subject to
reshuffle of assignments. Moreover, this resolution does not preclude the transfer of
assignment of bank officers and employees from the branch office to the head office and
vice-versa.

Pursuant to such Resolution, petitioner was one of the employees who were reshuffled
to a new assignment without changes in their compensation and other benefits. He was
assigned as an appraiser and later assigned as a Clerk-Meralco Collection.
The new assignments were to be effective on May 1, 1999 without changes in salary,
allowances, and other benefits received by the aforementioned employees. However, in
a letter, petitioner wrote to the management about such assignment. In his letter, he
averred that his assignment is tantamount to a demotion without any legal basis.
In its reply, the respondent informed petitioner that it was never the intention of
management to downgrade his position in the bank considering that his due
compensation as Bank Appraiser is maintained and no future reduction was intended.
The respondent also said that the conduct of reshuffle is also a prerogative of bank
management.

On June 7, 1999, petitioner applied for a leave of absence from work for ten (10) days.
On June 21, 1999, petitioner again submitted a letter asking for another leave of absence
for twenty days effective on the same date.

On June 24, 1999, while on his second leave of absence, petitioner filed a Complaint
before the NLRC for illegal dismissal, underpayment, separation pay and damages
against the Rural Bank of Lucban and/or its president, Alejo B. Daya.

The labor arbiter held that respondent is guilty of illegal dismissal. It also ordered the
reinstatement of the complainant to his former position without loss of seniority rights
with full backwages.

On appeal, the NLRC reversed the labor arbiter and held that there was no bad faith or
malice to the respondent bank for its implementation of its Board Resolution directing
the reshuffle of employees at its Tayabas branch to positions other than those they were
occupying.

After the NLRC denied his Motion for Reconsideration, petitioner brought before the
CA. The CA held that there was no grave abuse of discretion committed by the NLRC
in issuing its decision. It ruled thus that when Mendoza was reshuffled to the position
of clerk at the bank, he was not demoted as there was no diminution of his salary
benefits and rank. The reshuffling of its employees was done in good faith and cannot
be made the basis of a finding of constructive dismissal. The fact that Mendoza was no
longer included in the bank's payroll for July 1 to 15, 1999 does not signify that the bank
has dismissed the former from its employ. Mendoza separated himself from the bank's
employ when, on June 24, 1999, while on leave, he filed the illegal dismissal case against
his employer for no apparent reason at all. Hence, this Petition.

Issue:

Whether or not the reshuffling of private respondent's employees was done in good
faith and cannot be made as the basis of a finding of constructive dismissal, even as the
petitioner's demotion in rank is admitted by both parties.

Held:

NO. Jurisprudence recognizes the exercise of management prerogatives. For this reason,
courts often decline to interfere in legitimate business decisions of employers. Indeed,
labor laws discourage interference in employers' judgments concerning the conduct of
their business. The law must protect not only the welfare of employees, but also the
right of employers.

In the pursuit of its legitimate business interest, management has the prerogative to
transfer or assign employees from one office or area of operation to another provided
there is no demotion in rank or diminution of salary, benefits, and other privileges; and
the action is not motivated by discrimination, made in bad faith, or effected as a form of
punishment or demotion without sufficient cause.

This privilege is inherent in the right of employers to control and manage their
enterprise effectively. The right of employees to security of tenure does not give them
vested rights to their positions to the extent of depriving management of its prerogative
to change their assignments or to transfer them.

Managerial prerogatives, however, are subject to limitations provided by law, collective


bargaining agreements, and general principles of fair play and justice. The test for
determining the validity of the transfer of employees was explained in Blue Dairy
Corporation v. NLRC as follows:
"Like other rights, there are limits thereto. The managerial prerogative to transfer personnel
must be exercised without grave abuse of discretion, bearing in mind the basic elements of justice
and fair play. Having the right should not be confused with the manner in which that right is
exercised. Thus, it cannot be used as a subterfuge by the employer to rid himself of an
undesirable worker. In particular, the employer must be able to show that the transfer is not
unreasonable, inconvenient or prejudicial to the employee; nor does it involve a demotion in rank
or a diminution of his salaries, privileges and other benefits. Should the employer fail to
overcome this burden of proof, the employee's transfer shall be tantamount to constructive
dismissal, which has been defined as a quitting because continued employment is rendered
impossible, unreasonable or unlikely; as an offer involving a demotion in rank and diminution in
pay. Likewise, constructive dismissal exists when an act of clear discrimination, insensibility or
disdain by an employer has become so unbearable to the employee leaving him with no option but
to forego with his continued employment."

PHILIPPINE TELEGRAPH AND TELEPHONE CORPORATION, petitioner, vs.


ALICIA LAPLANA, Hon. RICARDO ENCARNACION, and NATIONAL LABOR
RELATIONS COMMISSION, respondents.
G.R. No. 76645 , July 23, 1991, FIRST DIVISION
NARVASA, J.

Facts:

Alicia Laplana was the cashier of the Baguio City Branch Office of the Philippine
Telegraph and Telephone Corporation (hereafter, simply PT & T). Sometime in March
1984, PT & T's treasurer, Mrs. Alicia A. Arogo, directed Laplana to transfer to the
company's branch office at Laoag City. Laplana refused the reassignment and proposed
instead that qualified clerks in the Baguio Branch be trained for the purpose. She set out
her reasons therefor in her letter that she have already established Baguio City as her
permanent residence and that working in Laoag will involve additional expenses for
her part like her board and lodgingly, fare, and other miscellaneous expenses. As a
result, her salary alone will not be enough for her family.

On April 12, 1984, Mrs. Arogo reiterated her directive for Laplana's transfer to the
Laoag Branch, this time in the form of a written Memorandum, informing Laplana that
effective April 16, 1984, she will be reassigned to Laoag branch assuming the same
position of branch cashier and ordering her to turn over your accountabilities such as
PCF, undeposited collections, used and unused official receipts, other accountable
forms and files to Rose Caysido who will be in charge of cashiering in Baguio.

Apparently Laplana was not allowed to resume her work as Cashier of the Baguio
Branch when April 16, 1984 came. She thereupon wrote again to Mrs. Arogo advising
that the directed transfer was unacceptable, reiterating the reasons already given by her
in her first letter. Subsequently, Laplana received a telegram from Mrs. Arogo ordering
her to report to Manila for a ner job assignment but the same was refused by petitioner
and asked that she be retrenched instead.

Termination of Laplana's employment on account of retrenchment thereupon followed.


On July 4, 1984, Laplana signed the quitclaim and received the check representing her
13th month and separation pay.

On October 9, 1984, Laplana filed with the Labor Arbiters' Office at Baguio City, thru
the CLAO, a complaint against PT & T its Baguio Northwestern Luzon Branch, Baguio
City, and Paraluman Bautista, Area Manager. In her complaint she alleged, as right of
action, that when she insisted on her right of refusing to be transferred, the Defendants
made good its warning by terminating her services on May 16, 1984 on alleged ground
of "retrenchment," although the truth is, she was forced to be terminated and that there
was no ground at all for the retrenchment; that the company's act of transferring is not
only without any valid ground but also arbitrary and without any purpose but to harass
and force her to eventually resign.

In answer, the defendants alleged that Laplana was being transferred to Laoag City
because of increase in sales due to the additional installations of vodex line; they also
alleged that the company was exercising management prerogatives in transferring
complainant and there is no showing that this exercise was arbitrarily and whimsically
done; Lastly, Laplana's services were terminated on her explicit declaration that "she
was willing to be retrenched rather than be assigned to Laoag City or Manila;"

Issue:

Whether or not Laplana’s termination is legal.

Held:

YES. In Philippine Japan Active Carbon Corp. v. NLRC, promulgated on March 8, 1989
the Court held that it is the employer's prerogative, based on its assessment and
perception of its employees' qualifications, aptitudes, and competence, to move them
around in the various areas of its business operations in order to ascertain where they
will function with maximum benefit to the company. An employee's right to security of
tenure does not give him such a vested right in his position as would deprive the
company of its prerogative to change his assignment or transfer him where he will be
most useful. When his transfer is not unreasonable, nor inconvenient, nor prejudicial to
him, and it does not involve a demotion in rank or diminution of his salaries, benefits,
and other privileges, the employee may not complain that it amounts to a constructive
dismissal.

In this case, the employee (Laplana) had to all intents and purposes resigned from her
position. She had unequivocally asked that she be considered dismissed, herself
suggesting the reason therefor –– retrenchment. When so dismissed, she accepted
separation pay. On the other hand, the employer has not been shown to be acting
otherwise than in good faith, and in the legitimate pursuit of what it considered its best
interests, in deciding to transfer her to another office. There is no showing whatever
that the employer was transferring Laplana to another work place, not because she
would be more useful there, but merely as a subterfuge to rid itself of an undesirable
worker, or to penalize an employee for union activities. The employer was moreover
not unmindful of Laplana's initial plea for reconsideration of the directive for her
transfer to Laoag; in fact, in response to that plea not to be moved to the Laoag Office,
the employer opted instead to transfer her to Manila, the main office, offering at the
same time the normal benefits attendant upon transfers from an office to another.

The situation here presented is of an employer transferring an employee to another


office in the exercise of what it took to be sound business judgment and in accordance
with pre-determined and established office policy and practice, and of the latter having
what was believed to be legitimate reasons for declining that transfer, rooted in
considerations of personal convenience and difficulties for the family. Under these
circumstances, the solution proposed by the employee herself, of her voluntary
termination of her employment and the delivery to her of corresponding separation
pay, would appear to be the most equitable. Certainly, the Court cannot accept the
proposition that when an employee opposes his employer's decision to transfer him to
another work place, there being no bad faith or underhanded motives on the part of
either party, it is the employee's wishes that should be made to prevail.

The termination of services of private respondent is declared legal and proper.

ALLIED BANKING CORPORATION, petitioner, vs. COURT OF APPEALS and


POTENCIANO L. GALANIDA,respondents.

[G.R. No. 144412. November 18, 2003] FIRST DIVISION

CARPIO, J.:

Facts:

Private respondent Potenciano Galanida was hired by petitioner Allied Banking


Corporation on 11 January 1978 and rose from accountant-book keeper to assistant
manager in 1991. His appointment was covered by a “ notice of personnel action”
which provided as one of the conditions of employment the provision on petitioner’s
right to transfer employees as the need arises and in the interest of maintaining smooth
and uninterrupted service to the public.

Private respondent was promoted several times and was transferred to several
branches.

Effecting a rotation/movement of officers assigned in the Cebu homebase, petitioner


listed respondent as second in the order of priority of assistant managers to be assigned
outside of Cebu City having been stationed in Cebu for seven years already. Private
respondent manifested his refusal to be transferred to Bacolod City in a letter citing as
reason parental obligations, expenses, and the anguish that would result if he is away
from his family. He then filed a complaint before the Labor Arbiter for constructive
dismissal.

Subsequently, petitioner bank informed private respondent that he was to report to the
Tagbilaran City Branch effective 23 May 1994. Private respondent refused.
Subsequently, petitioner warned and required of private respondent that tthere is no
discrimination in her transfer. Petitioner also invoked its right to transfer employees as
the need arises and in the interest of maintaining smooth and uninterrupted service to
the public.

Petitioner also warned respondent that her refusal to follow instruction concerning his
transfer and reassignment to Bacolod City and to Tagbilaran City is penalized under
Article XII of the Bank’s Employee Discipline Policy and Procedure. In case of refusal,
the penalty may range from suspension to dismissal as determined by management.

Respondent was also given an opportunity to explain why no disciplinary action


should be meted against him for refusing to follow instructions concerning the transfer
and reassignment.

On 16 June 1994, Galanida replied that whether the bank’s penalty for his refusal be
Suspension or Dismissal it will all the more establish and fortify his complaint now
pending before the NLRC. In the same letter, he charged Allied Bank with
discrimination and favoritism in ordering his transfer.

On 5 October 1994, Galanida received an inter-office communication stating that Allied


Bank had terminated his services effective 1 September 1994 for his continued refusal to
be transferred from the Jakosalem, Cebu City branch

After several hearings, the Labor Arbiter held that Allied Bank had abused its
management prerogative in ordering the transfer of Galanida to its Bacolod and
Tagbilaran branches. In ruling that Galanida’s refusal to transfer did not amount to
insubordination. The Labor Arbiter reasoned that Galanida’s transfer was inconvenient
and prejudicial because Galanida would have to incur additional expenses for board,
lodging and travel. On the other hand, the Labor Arbiter held that Allied Bank failed to
show any business urgency that would justify the transfer.

On appeal, the NLRC likewise ruled that Allied Bank terminated Galanida without just
cause. The NLRC agreed that the transfer order was unreasonable and unjustified,
considering the family considerations mentioned by Galanida. The NLRC characterized
the transfer as a demotion since the Bacolod and Tagbilaran branches were smaller than
the Jakosalem branch, a regional office, and because the bank wanted Galanida, an
assistant manager, to replace an assistant accountant in the Tagbilaran branch. The
NLRC found unlawful discrimination since Allied Bank did not transfer several junior
accountants in Cebu.

Allied Bank filed a motion for reconsideration which the NLRC denied. Dissatisfied,
Allied Bank filed a petition for review questioning the Decision and Resolution of the
NLRC before the Court of Appeals. The Court of Appeals held that Galanida’s refusal
to comply with the transfer orders did not warrant his dismissal. The appellate court
ruled that the transfer from a regional office to the smaller Bacolod or Tagbilaran
branches was effectively a demotion. The appellate court agreed that Allied Bank did
not afford Galanida procedural due process because there was no hearing and no notice
of termination. The Memo merely stated that the bank would issue a notice of
termination but there was no such notice.

Allied Bank filed a motion for reconsideration which the appellate court denied. Hence
this petition.

Issue:

Whether under the facts presented there is legal basis in petitioner’s exercise of its
management prerogative.

Held:

YES. The rule is that the transfer of an employee ordinarily lies within the ambit of the
employer’s prerogatives. The employer exercises the prerogative to transfer an
employee for valid reasons and according to the requirement of its business, provided
the transfer does not result in demotion in rank or diminution of the employee’s salary,
benefits and other privileges. In illegal dismissal cases, the employer has the burden of
showing that the transfer is not unnecessary, inconvenient and prejudicial to the
displaced employee.
The constant transfer of bank officers and personnel with accounting responsibilities
from one branch to another is a standard practice of Allied Bank, which has more than a
hundred branches throughout the country. Allied Bank does this primarily for internal
control. It also enables bank employees to gain the necessary experience for eventual

promotion. The Bangko Sentral ng Pilipinas, in its Manual of Regulations for Banks and
Other Financial Intermediaries, requires the rotation of these personnel. The Manual
directs that the “duties of personnel handling cash, securities and

bookkeeping records should be rotated” and that such rotation “should be irregular,
unannounced and long enough to permit disclosure of any irregularities or
manipulations.”

Galanida was well aware of Allied Bank’s policy of periodically transferring personnel
to different branches. As the Court of Appeals found, assignment to the different
branches of Allied Bank was a condition of Galanida’s employment. Galanida
consented to this condition when he signed the Notice of Personnel Action.

The evidence on record contradicts the charge that Allied Bank discriminated against
Galanida and was in bad faith when it ordered his transfer. Allied Bank’s letter of 13
June 1994 showed that at least 14 accounting officers and personnel from various
branches, including Galanida, were transferred to other branches. Allied Bank did not
single out Galanida. The same letter explained that Galanida was second in line for
assignment outside Cebu because he had been in Cebu for seven years already. The
person first in line, Assistant Manager Roberto Isla, who had been in Cebu for more
than ten years, had already transferred to a branch in Cagayan de Oro City. We note
that none of the other transferees joined Galanida in his complaint or corroborated his
allegations of widespread discrimination and favoritism.

The employer has the prerogative, based on its assessment of the employees’
qualifications and competence, to rotate them in the various areas of its business
operations to ascertain where they will function with maximum benefit to the company.

Neither was Galanida’s transfer in the nature of a demotion. Galanida did not present
evidence showing that the transfer would diminish his salary, benefits or other
privileges. Instead, Allied Bank assured Galanida that he would not suffer any
reduction in rank or grade, and that the transfer would involve the same rank, duties
and obligations.
GENUINE ICE COMPANY INC., Petitioner, - versus – ALFONSO S. MAGPANTAY,
Respondent.
G.R. No. 147790, June 27, 2006, FIRST DIVISION
AUSTRIA-MARTINEZ, J.:

Facts:

Alfonso Magpantay (respondent) was employed as a machine operator with Genuino


Ice Company, Inc. (petitioner) from March 1988 to December 1995. On November 18,
1996, respondent filed against petitioner a complaint for illegal dismissal alleging that
he was dismissed from service effective immediately by virtue of a memorandum, after
which he was not allowed anymore to enter the company premises. Respondent
bewailed that his termination from employment was done without due process.

Petitioner countered that he was not illegally dismissed, since the dismissal was based
on a valid ground, i.e., he led an illegal strike at petitioner’s sister company, Genuino
Agro Industrial Development Corporation, which lasted from November 18 to 22, 1995,
resulting in big operation losses on the latter’s part. Petitioner also maintained that
respondent’s dismissal was made after he was accorded due process.

Respondent replied, however, that assuming that he led such illegal strike, he could not
be liable therefore because it was done in petitioner’s sister company which is a
separate and distinct entity from petitioner.

Petitioner initially claimed that respondent’s acts were tantamount to serious


misconduct or willful disobedience, gross and habitual neglect of duties, and breach of
trust. Subsequently, petitioner amended its position paper to include insubordination
among the grounds for his dismissal, since it came out during respondent’s cross-
examination, and the matter was reported only after the new personnel manager
assumed his position in August 1996.

Issue:

Whether or not the respondent was illegally dismissed from employment.

Held:

No.

Both the Labor Arbiter and the NLRC were one in concluding that petitioner had just
cause for dismissing respondent, as his act of leading a strike at petitioner’s company
for four days, his absence from work during such time, and his failure to perform his
duties during such absence, make up a cause for habitual neglect of duties, while his
failure to comply with petitioner’s order for him to transfer to the GMA, Cavite Plant
constituted insubordination or willful disobedience. The CA, however, differed with
said conclusion and found that respondent’s attitude “has not been proved to be visited
with any wrongdoing”, and that his four-day absence does not appear to be both gross
and habitual.

The Court sustains the CA’s finding that respondent’s four-day absence does not
amount to a habitual neglect of duty; however, the Court finds that respondent was
validly dismissed on ground of willful disobedience or insubordination.

Under Article 282 of the Labor Code, as amended, an employer may terminate an
employment for any of the following causes: (a) serious misconduct or willful
disobedience by the employee of the lawful orders of his employer or representative in
connection with his work; (b) gross and habitual neglect by the employee of his duties;
(c) fraud or willful breach by the employee of the trust reposed in him by his employer
or duly authorized representative; (d) commission of a crime or offense by the
employee against the person of his employer or any immediate member of his family or
his duly authorized representative; and, (e) other causes analogous to the foregoing.
The employer has the burden of proving that the dismissal was for a just cause; failure
to show this would necessarily mean that the dismissal was unjustified and, therefore,
illegal.

Neglect of duty, to be a ground for dismissal, must be both gross and habitual. Gross
negligence connotes want of care in the performance of one’s duties. Habitual neglect
implies repeated failure to perform one’s duties for a period of time, depending upon
the circumstances. On the other hand, fraud and willful neglect of duties imply bad
faith on the part of the employee in failing to perform his job to the detriment of the
employer and the latter’s business. Thus, the single or isolated act of negligence does
not constitute a just cause for the dismissal of the employee.

PHILIPPINE DUPLICATORS, INC., petitioner, vs. NATIONAL LABOR


RELATIONS COMMISSION and PHILIPPINE DUPLICATORS EMPLOYEES
UNION-TUPAS,respondents.
G.R. No. 110068 February 15, 1995, EN BANC
FELICIANO, J.:

Facts:

This Case differentiates Productivity Bonuses and Commissions Productivity bonuses


are generally tied to the productivity or profit generation of the employer corporation.
Productivity bonuses are not directly dependent on the extent an individual employee
exerts himself. A productivity bonus is something extra for which no specific additional
services are rendered by any particular employee and hence not legally demandable,
absent a contractual undertaking to pay it. Sales commissions are intimately related to
or directly proportional to the extent or energy of an employee's endeavours.
Commissions are paid upon the specific results achieved by a salesman-employee. It is
a percentage of the sales closed by a salesman and operates as an integralpart of such
salesman's basic pay.

Issue:

1.WON The commissions received by the salesmen were part of the”wages” to be


considered for their 13thmonth pay. – Yes

2.WON Productivity bonus shall be considered as part of wages in 13th month pay –
No

Held:

1.The commissions were an integral part of the pay of the workers, considering that the
fixed wage was only 30% of what they were normally receiving.

2.Productivity bonuses are generally tied to the productivity, or capacity for revenue
production, of a corporation; such bonuses closely resemble profit-sharing payments
and have no clear director necessary relation to the amount of work actually done by
each individual employee. More generally, a bonus is an amount granted and paid ex
gratia to the employee; its payment constitutes an act of enlightened generosity and
self-interest on the part of the employer, rather than as a demandable or enforceable
obligation. Since productivity bonus is not demandable, then it cannot be considered
part of basic salary when time comes to compute 13th month pay. Additional payments
made to employees, to the extent they partake of the nature of profit-sharing payments,
are properly excluded from the ambit of the term "basic salary" for purposes of
computing the 13th month pay due to employees. Such additional payments are not
"commissions" within the meaning of the second paragraph of Section 5 (a) of the
Revised Guidelines Implementing13th Month Pay. The Supplementary Rules and
Regulations Implementing P.D. No.851 subsequently issued by former Labor Minister
Ople sought to clarify the scope of items excluded in the computation of the 13thmonth
pay; viz .:Sec. 4. Overtime pay, earnings and other remunerations which are not part of
the basic salary shall not be included in the computation of the 13th month pay.
EASTERN TELECOMMUNICATIONS PHILIPPINES, INC., Petitioner,- versus -
EASTERN TELECOMS EMPLOYEES UNION, Respondent.
February 8, 2012, G.R. No. 185665, THIRD DIVISION
MENDOZA, J.:

Facts:

Eastern Telecommunications Phils., Inc. (ETPI) is a corporation engaged in the business


of providing telecommunications facilities, particularly leasing international date lines
or circuits, regular landlines, internet and data services, employing approximately 400
employees.

Eastern Telecoms Employees Union (ETEU) is the certified exclusive bargaining agent
of the company’s rank and file employees with a strong following of 147 regular
members. It has an existing collective bargaining agreement with the company to expire
in the year 2004 with a Side Agreement signed on September 3, 2001.
The Eastern Telecoms Employees Union (ETEU) claimed that Eastern
Telecommunications Philippines, Inc. (ETPI) had consistently and voluntarily been
giving out 14th month bonus during the month of April, and 15th and 16th month
bonuses every December of each year (subject bonuses) to its employees from 1975 to
2002, even when it did not realize any net profits. ETEU posited that by reason of its
long and regular concession, the payment of these monetary benefits had ripened into a
company practice which could no longer be unilaterally withdrawn by ETPI. ETEU
added that this long-standing company practice had been expressly confirmed in the
Side Agreements of the 1998-2001 and 2001-2004 Collective Bargaining
Agreements(CBA) which provided for the continuous grant of these bonuses in no
uncertain terms. ETEU theorized that the grant of the subject bonuses is not only a
company practice but also a contractual obligation of ETPI to the union members.

ETEU contended that the unjustified and malicious refusal of the company to pay the
subject bonuses was a clear violation of the economic provision of the CBA and
constitutes unfair labor practice (ULP). According to ETEU, such refusal was nothing
but a ploy to spite the union for bringing the matter of delay in the payment of the
subject bonuses to the National Conciliation and Mediation Board (NCMB). It prayed
for the award of moral and exemplary damages as well as attorney’s fees for the unfair
labor practice allegedly committed by the company.
The NLRC dismissed ETEU’s complaint and held that ETPI could not be forced to pay
the union members the 14th, 15th and 16th month bonuses for the year 2003 and the
14th month bonus for the year 2004 inasmuch as the payment of these additional
benefits was basically a management prerogative, being an act of generosity and
munificence on the part of the company and contingent upon the realization of profits.
The NLRC pronounced that ETPI may not be obliged to pay these extra compensations
in view of the substantial decline in its financial condition. Likewise, the NLRC found
that ETPI was not guilty of the ULP charge elaborating that no sufficient and substantial
evidence was adduced to attribute malice to the company for its refusal to pay the
subject bonuses.

Respondent ETEU moved for reconsideration but the motion was denied by the NLRC.
Aggrieved, ETEU filed a petition for certiorari before the CA ascribing grave abuse of
discretion on the NLRC for disregarding its evidence which allegedly would prove that
the subject bonuses were part of the union members’ wages, salaries or compensations.
The CA declared that the Side Agreements of the 1998 and 2001 CBA created a
contractual obligation on ETPI to confer the subject bonuses to its employees without
qualification or condition. It also found that the grant of said bonuses has already
ripened into a company practice and their denial would amount to diminution of the
employees’ benefits.

Issue:

1. Whether the members of ETEU are entitled to the payment of 14th, 15th and 16th
month bonuses for the year 2003 and 14th month bonus for year 2004.

2. Whether these bonuses are demandable or not. Stated differently, can these bonuses
be considered part of the wage, salary or compensation making them enforceable
obligations?

Held:

1. YES. From a legal point of view, a bonus is a gratuity or act of liberality of the giver
which the recipient has no right to demand as a matter of right. The grant of a bonus is
basically a management prerogative which cannot be forced upon the employer who
may not be obliged to assume the onerous burden of granting bonuses or other benefits
aside from the employee’s basic salaries or wages.
A bonus, however, becomes a demandable or enforceable obligation when it is made
part of the wage or salary or compensation of the employee. Particularly instructive is
the ruling of the Court in Metro Transit Organization, Inc. v. National Labor Relations
Commission, where it was written:
Whether or not a bonus forms part of wages depends upon the circumstances and conditions for
its payment. If it is additional compensation which the employer promised and agreed to give
without any conditions imposed for its payment, such as success of business or greater
production or output, then it is part of the wage. But if it is paid only if profits are realized or if a
certain level of productivity is achieved, it cannot be considered part of the wage. Where it is not
payable to all but only to some employees and only when their labor becomes more efficient or
more productive, it is only an inducement for efficiency, a prize therefore, not a part of the wage.

2. YES. A reading of the CBA provision reveals that the same provides for the giving of
14th, 15th and 16th month bonuses without qualification. The wording of the provision
does not allow any other interpretation. There were no conditions specified in the CBA
Side Agreements for the grant of the benefits contrary to the claim of ETPI that the same
is justified only when there are profits earned by the company. Terse and clear, the said
provision does not state that the subject bonuses shall be made to depend on the ETPI’s
financial standing or that their payment was contingent upon the realization of profits.
Neither does it state that if the company derives no profits, no bonuses are to be given
to the employees. In fine, the payment of these bonuses was not related to the
profitability of business operations.

The records are also bereft of any showing that the ETPI made it clear before or during
the execution of the Side Agreements that the bonuses shall be subject to any condition.
Indeed, if ETPI and ETEU intended that the subject bonuses would be dependent on the
company earnings, such intention should have been expressly declared in the Side
Agreements or the bonus provision should have been deleted altogether. In the absence
of any proof that ETPI’s consent was vitiated by fraud, mistake or duress, it is
presumed that it entered into the Side Agreements voluntarily, that it had full
knowledge of the contents thereof and that it was aware of its commitment under the
contract.

Verily, by virtue of its incorporation in the CBA Side Agreements, the grant of 14th,
15th and 16th month bonuses has become more than just an act of generosity on the
part of ETPI but a contractual obligation it has undertaken. Moreover, the continuous
conferment of bonuses by ETPI to the union members from 1998 to 2002 by virtue of the
Side Agreements evidently negates its argument that the giving of the subject bonuses
is a management prerogative.

The rule is settled that any benefit and supplement being enjoyed by the employees
cannot be reduced, diminished, discontinued or eliminated by the employer. The
principle of non-diminution of benefits is founded on the constitutional mandate to
protect the rights of workers and to promote their welfare and to afford labor full
protection.
MANILA JOCKEY CLUB EMPLOYEES LABOR UNION- PTGWO, Petitioner,
-versus-
MANILA JOCKEY CLUB, INC., Respondent.
G.R. No. 167760, March 7, 2007, FIRST DIVISION
GARCIA, J.:

Facts:

Petitioner Manila Jockey Club Employees Labor Union-PTGWO and respondent Manila
Jockey Club, Inc., a corporation with a legislative franchise to conduct, operate and
maintain horse races, entered into a Collective Bargaining Agreement (CBA) effective
January 1, 1996 to December 31, 2000. The CBA governed the economic rights and
obligations of respondent’s regular monthly paid rank-and-file employees. In the CBA,
the parties agreed to a 7-hour work schedule from 9:00 a.m. to 12:00 noon and from1:00
p.m. to 5:00 p.m. on a work week of Monday to Saturday.

Accordingly, overtime on an ordinary working day shall be remunerated in an amount


equivalent to the worker's regular basic wage plus twenty five percent (25%) thereof.
Where the employee is permitted or suffered to work on legally mandated holidays or
on his designated rest day which is not a legally mandated holiday, thirty percent (30%)
shall be added to his basic wage for a seven hour work; while work rendered in excess
of seven hours on legally mandated holidays and rest days not falling within the
aforestated categories day shall be additionally compensated for the overtime work
equivalent to his rate for the first seven hours on a legally mandated holiday or rest day
plus thirty percent (30%) thereof.

The CBA likewise reserved in respondent certain management prerogatives, including


the determination of the work schedule.

On April 3, 1999, respondent issued an inter-office memorandum declaring that,


effective April 20, 1999, the hours of work of regular monthly-paid employees shall be
from 1:00 p.m. to 8:00 p.m. when horse races are held, that is, every Tuesday and
Thursday. The memorandum, however, maintained the 9:00 a.m. to 5:00 p.m. schedule
for non-race days.

On October 12, 1999, petitioner and respondent entered into an Amended and
Supplemental CBA and clarified that any conflict arising therefrom shall be referred to a
voluntary arbitrator for resolution.

Subsequently, before a panel of voluntary arbitrators of the National Conciliation and


Mediation Board (NCMB), petitioner questioned the memorandum as violative of the
prohibition against non-diminution of wages and benefits guaranteed under Section 1,
Article IV, of the CBA which specified the work schedule of respondent's employees to
be from 9:00 a.m. to 5:00 p.m. Petitioner claimed that as a result of the memorandum,
the employees are precluded from rendering their usual overtime work from 5:00 p.m.
to 9:00 p.m.

The NCMB’s panel of voluntary arbitrators upheld respondent's prerogative to change


the work schedule of regular monthly-paid employees. Petitioner moved for
reconsideration but the panel denied the motion.

Dissatisfied, petitioner then appealed the panel’s decision to the CA . The CA upheld
that of the panel and denied petitioner’s subsequent motion for reconsideration.
Hence, petitioner’s present recourse,

Issues:

Whether or not the act of the company is changing the work schedule of the employees
is part of management prerogative.

Held:
YES. Respondent, as employer, cites the change in the program of horse races as reason
for the adjustment of the employees’ work schedule. It rationalizes that when the CBA
was signed, the horse races started at 10:00 a.m. When the races were moved to 2:00
p.m., there was no other choice for management but to change the employees' work
schedule as there was no work to be done in the morning. Evidently, the adjustment in
the work schedule of the employees is justified.

We are not unmindful that every business enterprise endeavors to increase profits. As it
is, the Court will not interfere with the business judgment of an employer in the
exercise of its prerogative to devise means to improve its operation, provided that it
does not violate the law, CBAs, and the general principles of justice and fair play. We
have thus held that management is free to regulate, according to its own discretion and
judgment, all aspects of employment, including hiring, work assignments, working
methods, time, place and manner of work, processes to be followed, supervision of
workers, working regulations, transfer of employees, work supervision, layoff of
workers and discipline, dismissal, and recall of workers.
DUNCAN ASSOCIATION OF DETAILMAN-PTGWO and PEDRO A. TECSON,
petitioners, vs. GLAXO WELLCOME PHILIPPINES, INC., Respondent.
G.R. No. 162994, September 17, 2004, SECOND DIVISION
TINGA, J.:

Facts:

Petitioner Pedro A. Tecson was hired by respondent Glaxo Wellcome Philippines, Inc.
as medical representative after Tecson had undergone training and orientation.

Thereafter, Tecson signed a contract of employment which stipulates, among others,


that he agrees to study and abide by existing company rules; to disclose to management
any existing or future relationship by consanguinity or affinity with co-employees or
employees of competing drug companies and should management find that such
relationship poses a possible conflict of interest, to resign from the company.

The Employee Code of Conduct of Glaxo similarly provides that an employee is


expected to inform management of any existing or future relationship by consanguinity
or affinity with co-employees or employees of competing drug companies. If
management perceives a conflict of interest or a potential conflict between such
relationship and the employee’s employment with the company, the management and
the employee will explore the possibility of a "transfer to another department in a non-
counterchecking position" or preparation for employment outside the company after six
months.

Tecson was initially assigned to market Glaxo’s products in the Camarines Sur-
Camarines Norte sales area.
Subsequently, Tecson entered into a romantic relationship with Bettsy, an employee of
Astra Pharmaceuticals (Astra), a competitor of Glaxo. Bettsy was Astra’s Branch
Coordinator in Albay. She supervised the district managers and medical representatives
of her company and prepared marketing strategies for Astra in that area.

Even before they got married, Tecson received several reminders from his District
Manager regarding the conflict of interest which his relationship with Bettsy might
engender. Still, love prevailed, and Tecson married Bettsy in September 1998.

In January 1999, Tecson’s superiors informed him that his marriage to Bettsy gave rise
to a conflict of interest. Tecson’s superiors reminded him that he and Bettsy should
decide which one of them would resign from their jobs, although they told him that
they wanted to retain him as much as possible because he was performing his job well.
Tecson requested for time to comply with the company policy against entering into a
relationship with an employee of a competitor company. He explained that Astra,
Bettsy’s employer, was planning to merge with Zeneca, another drug company; and
Bettsy was planning to avail of the redundancy package to be offered by Astra. With
Bettsy’s separation from her company, the potential conflict of interest would be
eliminated. At the same time, they would be able to avail of the attractive redundancy
package from Astra.

In August 1999, Tecson again requested for more time resolve the problem. In
September 1999, Tecson applied for a transfer in Glaxo’s milk division, thinking that
since Astra did not have a milk division, the potential conflict of interest would be
eliminated. His application was denied in view of Glaxo’s "least-movement-possible"
policy.

In November 1999, Glaxo transferred Tecson to the Butuan City-Surigao City-Agusan


del Sur sales area. Tecson asked Glaxo to reconsider its decision, but his request was
denied.

Tecson sought Glaxo’s reconsideration regarding his transfer and brought the matter to
Glaxo’s Grievance Committee. Glaxo, however, remained firm in its decision and gave
Tescon until February 7, 2000 to comply with the transfer order. Tecson defied the
transfer order and continued acting as medical representative in the Camarines Sur-
Camarines Norte sales area.

During the pendency of the grievance proceedings, Tecson was paid his salary, but was
not issued samples of products which were competing with similar products
manufactured by Astra. He was also not included in product conferences regarding
such products.

Because the parties failed to resolve the issue at the grievance machinery level, they
submitted the matter for voluntary arbitration. Glaxo offered Tecson a separation pay of
one-half (½) month pay for every year of service, or a total of P50,000.00 but he declined
the offer. On November 15, 2000, the National Conciliation and Mediation Board
(NCMB) rendered its Decision declaring as valid Glaxo’s policy on relationships
between its employees and persons employed with competitor companies, and
affirming Glaxo’s right to transfer Tecson to another sales territory.

Aggrieved, Tecson filed a Petition for Review with the Court of Appeals assailing the
NCMB Decision.

The Court of Appeals denied the Petition for Review on the ground that the NCMB did
not err in rendering its Decision. The appellate court held that Glaxo’s policy
prohibiting its employees from having personal relationships with employees of
competitor companies is a valid exercise of its management prerogatives.
Tecson filed a Motion for Reconsideration of the appellate court’s Decision, but the
motion was denied by the appellate court.
Hence this petition.

Issue:

Whether the Glaxo’s policy against its employees marrying employees from competitor
companies is valid;

Held:

YES. No reversible error can be ascribed to the Court of Appeals when it ruled that
Glaxo’s policy prohibiting an employee from having a relationship with an employee of
a competitor company is a valid exercise of management prerogative.

Glaxo has a right to guard its trade secrets, manufacturing formulas, marketing
strategies and other confidential programs and information from competitors,
especially so that it and Astra are rival companies in the highly competitive
pharmaceutical industry.
The prohibition against personal or marital relationships with employees of competitor
companies upon Glaxo’s employees is reasonable under the circumstances because
relationships of that nature might compromise the interests of the company. In laying
down the assailed company policy, Glaxo only aims to protect its interests against the
possibility that a competitor company will gain access to its secrets and procedures.
That Glaxo possesses the right to protect its economic interests cannot be denied. No
less than the Constitution recognizes the right of enterprises to adopt and enforce such a
policy to protect its right to reasonable returns on investments and to expansion and
growth.20 Indeed, while our laws endeavor to give life to the constitutional policy on
social justice and the protection of labor, it does not mean that every labor dispute will
be decided in favor of the workers. The law also recognizes that management has rights
which are also entitled to respect and enforcement in the interest of fair play.
PHILIPPINE TELEGRAPH AND TELEPHONE COMPANY, petitioner, vs.
NATIONAL LABOR RELATIONS COMMISSION and GRACE DE GUZMAN,
respondents.
G.R. No. 118978, May 23, 1997, SECOND DIVISION
REGALADO, J.

Facts:

Grace de Guzman was initially hired by petitioner as a reliever, specifically as a


"Supernumerary Project Worker," for a fixed period from November 21, 1990 until April
20, 1991 vice one C.F. Tenorio who went on maternity leave. Under the Reliever
Agreement which she signed with petitioner company, her employment was to be
immediately terminated upon expiration of the agreed period. Thereafter, from June 10,
1991 to July 1, 1991, and from July 19, 1991 to August 8, 1991, private respondent's
services as reliever were again engaged by petitioner, this time in replacement of one
Erlinda F. Dizon who went on leave during both periods. After August 8, 1991, and
pursuant to their Reliever Agreement, her services were terminated.

On September 2, 1991, private respondent was once more asked to join petitioner
company as a probationary employee, the probationary period to cover 150 days. In the
job application form that was furnished her to be filled up for the purpose, she
indicated in the portion for civil status therein that she was single although she had
contracted marriage a few months earlier, that is, on May 26, 1991.

It now appears that private respondent had made the same representation in the two
successive reliever agreements which she signed on June 10, 1991 and July 8, 1991.
When petitioner supposedly learned about the same later, its branch supervisor in
Baguio City, Delia M. Oficial, sent to private respondent a memorandum requiring her
to explain the discrepancy. In that memorandum, she was reminded about the
company's policy of not accepting married women for employment.

In her reply letter, private respondent stated that she was not aware of PT&T's policy
regarding married women at the time, and that all along she had not deliberately
hidden her true civil status. Petitioner nonetheless remained unconvinced by her
explanations. Private respondent was dismissed from the company which she readily
contested by initiating a complaint for illegal dismissal, coupled with a claim for non-
payment of cost of living allowances (COLA), before the Regional Arbitration Branch of
the National Labor Relations Commission in Baguio City.
The Labor Arbiter declared that private respondent, who had already gained the status
of a regular employee, was illegally dismissed by petitioner. Her reinstatement, plus
payment of the corresponding back wages and COLA, was correspondingly ordered.
On appeal to the National Labor Relations Commission (NLRC), said public respondent
upheld the labor arbiter and it ruled that private respondent had indeed been the
subject of an unjust and unlawful discrimination by her employer, PT & T.

Issue:

Whether the act of the petitioner constituted a marital discrimination.

Held:

YES. It is recognized that regulation of manpower by the company falls within the so-
called management prerogatives, which prescriptions encompass the matter of hiring,
supervision of workers, work assignments, working methods and assignments, as well
as regulations on the transfer of employees, lay-off of workers, and the discipline,
dismissal, and recall of employees. As put in a case, an employer is free to regulate,
according to his discretion and best business judgment, all aspects of employment,
"from hiring to firing," except in cases of unlawful discrimination or those which may
be provided by law.

In the case at bar, petitioner's policy of not accepting or considering as disqualified from
work any woman worker who contracts marriage runs afoul of the test of, and the right
against, discrimination, afforded all women workers by our labor laws and by no less
than the Constitution. Contrary to petitioner's assertion that it dismissed private
respondent from employment on account of her dishonesty, the record discloses clearly
that her ties with the company were dissolved principally because of the company's
policy that married women are not qualified for employment in PT & T, and not merely
because of her supposed acts of dishonesty.

In the present controversy, petitioner's expostulations that it dismissed private


respondent, not because the latter got married but because she concealed that fact, does
have a hollow ring. Her concealment, so it is claimed, bespeaks dishonesty hence the
consequent loss of confidence in her which justified her dismissal.
STAR PAPER CORPORATION, JOSEPHINE ONGSITCO & SEBASTIAN CHUA,
Petitioners, -versus- RONALDO D. SIMBOL, WILFREDA N. COMIA & LORNA E.
ESTRELLA, Respondents.
G.R. No. 164774 , April 12, 2006, SECOND DIVISION
PUNO, J.:

Facts:

Petitioner Star Paper Corporation is a corporation engaged in trading – principally of


paper products. Josephine Ongsitco is its Manager of the Personnel and Administration
Department while Sebastian Chua is its Managing Director.

Respondents Ronaldo D. Simbol , Wilfreda N. Comia and Lorna E. Estrella were all
regular employees of the company.

Simbol was employed by the company on October 27, 1993. He met Alma Dayrit, also
an employee of the company, whom he married on June 27, 1998. Prior to the marriage,
Ongsitco advised the couple that should they decide to get married, one of them should
resign pursuant to a company policy.

Simbol resigned on June 20, 1998 pursuant to the company policy.

Comia was hired by the company on February 5, 1997. She met Howard Comia, a co-
employee, whom she married on June 1, 2000. Ongsitco likewise reminded them that
pursuant to company policy, one must resign should they decide to get married. Comia
resigned on June 30, 2000.

Estrella was hired on July 29, 1994. She met Luisito Zuñiga , also a co-worker.
Petitioners stated that Zuñiga, a married man, got Estrella pregnant. The company
allegedly could have terminated her services due to immorality but she opted to resign
on December 21, 1999.

The respondents each signed a Release and Confirmation Agreement. They stated
therein that they have no money and property accountabilities in the company and that
they release the latter of any claim or demand of whatever nature.

Respondents offer a different version of their dismissal. Simbol and Comia allege that
they did not resign voluntarily; they were compelled to resign in view of an illegal
company policy. As to respondent Estrella, she alleges that she had a relationship with
co-worker Zuñiga who misrepresented himself as a married but separated man. After
he got her pregnant, she discovered that he was not separated. Thus, she severed her
relationship with him to avoid dismissal due to the company policy. On November 30,
1999, she met an accident and was advised by the doctor at the Orthopedic Hospital to
recuperate for twenty-one (21) days. She returned to work on December 21, 1999 but
she found out that her name was on-hold at the gate. She was denied entry. She was
directed to proceed to the personnel office where one of the staff handed her a
memorandum. The memorandum stated that she was being dismissed for immoral
conduct. She refused to sign the memorandum because she was on leave for twenty-one
(21) days and has not been given a chance to explain. The management asked her to
write an explanation. However, after submission of the explanation, she was
nonetheless dismissed by the company. Due to her urgent need for money, she later
submitted a letter of resignation in exchange for her thirteenth month pay.

Respondents later filed a complaint for unfair labor practice, constructive dismissal,
separation pay and attorney’s fees. They averred that the aforementioned company
policy is illegal and contravenes Article 136 of the Labor Code.

The Labor Arbiter dismissed the complaint for lack of merit. On appeal to the NLRC,
the Commission affirmed the decision of the Labor Arbiter.

Respondents filed a Motion for Reconsideration but was denied by the NLRC. They
appealed to the Court of Appeals which reversed the NLRC decision declaring illegal,
the petitioners’ dismissal from employment and ordering private respondents to
reinstate petitioners to their former positions without loss of seniority rights with full
backwages from the time of their dismissal until actual reinstatement;

Issue:

Whether the policy of the employer banning spouses from working in the same
company is a valid exercise of management prerogative.

Held:

NO. The Labor Code is the most comprehensive piece of legislation protecting labor.
The case at bar involves Article 136 of the Labor Code which provides:
Art. 136. It shall be unlawful for an employer to require as a condition of employment
or continuation of employment that a woman employee shall not get married, or to
stipulate expressly or tacitly that upon getting married a woman employee shall be
deemed resigned or separated, or to actually dismiss, discharge, discriminate or
otherwise prejudice a woman employee merely by reason of her marriage.

It is true that the policy of petitioners prohibiting close relatives from working in the
same company takes the nature of an anti-nepotism employment policy. Companies
adopt these policies to prevent the hiring of unqualified persons based on their status as
a relative, rather than upon their ability.

These policies focus upon the potential employment problems arising from the
perception of favoritism exhibited towards relatives.

With more women entering the workforce, employers are also enacting employment
policies specifically prohibiting spouses from working for the same company. We note
that two types of employment policies involve spouses: policies banning only spouses
from working in the same company (no-spouse employment policies), and those
banning all immediate family members, including spouses, from working in the same
company (anti-nepotism employment policies).

A requirement that a woman employee must remain unmarried could be justified as a


“bona fide occupational qualification,” or BFOQ, where the particular requirements of
the job would justify the same, but not on the ground of a general principle, such as the
desirability of spreading work in the workplace. A requirement of that nature would be
valid provided it reflects an inherent quality reasonably necessary for satisfactory job
performance.

We do not find a reasonable business necessity in the case at bar. It is significant to note
that in the case at bar, respondents were hired after they were found fit for the job, but
were asked to resign when they married a co-employee. Petitioners failed to show how
the marriage of Simbol, then a Sheeting Machine Operator, to Alma Dayrit, then an
employee of the Repacking Section, could be detrimental to its business operations.
Neither did petitioners explain how this detriment will happen in the case of Wilfreda
Comia, then a Production Helper in the Selecting Department, who married Howard
Comia, then a helper in the cutter-machine. The policy is premised on the mere fear that
employees married to each other will be less efficient. If we uphold the questioned rule
without valid justification, the employer can create policies based on an unproven
presumption of a perceived danger at the expense of an employee’s right to security of
tenure.
Lastly, the absence of a statute expressly prohibiting marital discrimination in our
jurisdiction cannot benefit the petitioners. The protection given to labor in our
jurisdiction is vast and extensive that we cannot prudently draw inferences from the
legislature’s silence that married persons are not protected under our Constitution and
declare valid a policy based on a prejudice or stereotype. Thus, for failure of petitioners
to present undisputed proof of a reasonable business necessity, we rule that the
questioned policy is an invalid exercise of management prerogative.

ANSELMO FERRAZZINI, plaintiff-appellee, vs. CARLOS GSELL, defendant-


appellant.
G.R. No. L-10712, August 10, 1916, EN BANC
TRENT, J.:

Facts:
The plaintiff had been employed by the defendant for an indefinite time to work in the
latter's industrial enterprises involving the manufacture of umbrellas in the city of
Manila. The defendant admitted that he discharged the plaintiff without giving him the
"written advice of six months in advance" as provided in the contract, but alleged that
the discharge was lawful on account of absence, unfaithfulness, and disobedience of
orders. The defendant sought affirmative relief for a further alleged breach of the
contract by the plaintiff after his discharge.

That portion of the contract upon which the defendant's counterclaimed is based reads
as follows:
That during the term of this contract, and for the period of five years after the
termination of the employment of the said party of the second part, whether this
contract continue in force for the period of one, two, three or more years, or be sooner
terminated, the said party of the second party shall not engage or interest himself in any
business enterprises similar to or in competition with those conducted, maintained or
operated by the said party of the first day in the Philippines, and shall not assist, aid or
encourage any such enterprise by the furnishing of information, advice or suggestions
of any kind, and shall not enter into the employ of any enterprises in the Philippine
Islands, whatever, save and except after obtaining special written permission therefor
from the said party of the first part. It is further stipulated and agreed that the said
party of the second part is hereby obligated and bound to pay unto the party of the first
part the sum of ten thousand pesos, Philippine currency (P10,000) as liquidated
damages for each and every breach of the present clause of this contract, whether such
breach occurred during the employment of the said party of the second part or at any
time during the period of five years from and after the termination of said employment,
and without regard to the cause of the termination of said employment.

The plaintiff admits that he entered the employment of Mr. Whalen in the Philippine
Islands as a foreman on some construction work for a cement factory within a few days
after his discharge and without the consent, either written or verbal, of the defendant.
This work was entirely different and disassociated from that engaged in by the
defendant Gsell, yet this act of the plaintiff was a technical violation of the above-
quoted provisions of the contract wherein he expressly agreed and obligated himself
"not to enter into the employment of any enterprise in the Philippine Islands, whatever,
save and except after obtaining special written permission therefor" from the defendant.

Issue:
Whether the provisions of the contract are valid and binding upon the plaintiff.

Held:

No.

The contract under consideration, tested by the law, rules and principlest forth is clearly
one in undue or unreasonable restraint of trade and therefore against public policy. It is
limited as to time and space but not as to trade. It is not necessary for the protection of
the defendant, as this is provided for in another part of the clause. It would force the
plaintiff to leave the Philippine Islands in order to obtain a livelihood in case the
defendant declined to give him the written permission to work elsewhere in this
country.

ALFONSO DEL CASTILLO, plaintiff-appellant, vs. SHANNON RICHMOND,


defendant-appellee.
G.R. No. L-21127, February 9, 1924, EN BANC
JOHNSON, J.:

Facts:
Shannon Richmond hired Alfonso del Castillo as pharmacist of the former’s drugstore
known as the Botica Americana situated in the district of Legaspi of the municipality
and Province of Albay. In their agreement, the following stipulation appeared:
3. That in consideration of the fact that the said Alfonso del Castillo has just graduated
as a pharmacist and up to the present time has not been employed in the capacity of a
pharmacist and in consideration of this employment and the monthly salary mentioned
in this contract, the said Alfonso del Castillo also agrees not to open, nor own nor have
any interest directly or indirectly in any other drugstore either in his own name or in
the name of another; nor have any connection with or be employed by any other
drugstore situated within a radius of our miles from the district of Legaspi,
municipality and Province of Albay, while the said Shannon Richmond or his heirs may
own or have open a drugstore, or have an interest in any other one within the limits of
the districts of Legaspi, Albay, and Daraga of the municipality of Albay, Province of
Albay.

The said contract was acknowledge before a notary on the same day of its execution.
The plaintiff alleges that the provisions and conditions contained in the third paragraph
of said contract constitute an illegal and unreasonable restriction upon his liberty to
contract, are contrary to public policy, and are unnecessary in order to constitute a just
and reasonable protection to the defendant; and asked that the same be declared null
and void and of no effect. The defendant interposed a general and special defense. In
his special defense he alleges "that during the time the plaintiff was in the defendant's
employ he obtained knowledge of his trade and professional secrets and came to know
and became acquainted and established friendly relations with his customers so that to
now annul the contract and permit plaintiff to establish a competing drugstore in the
town of Legaspi, as plaintiff has announced his intention to do, would be extremely
prejudicial to defendant's interest."

Issue:

Whether or not the qestioned stipulation is valid.


Held:

YES.

A contract by which an employee agrees to refrain for a given lenght of time, after the
expiration of the term of his employment, from engaging in a business, competitive
with that of his employer, is not void as being in restraint of trade if the restraint
imposed is not greater than that which is necessary to afford a reasonable protection. In
all cases like the present, the question is whether, under the particular circumstances of
the case and the nature of the particular contract involved in it, the contract is, or is not,
unreasonable. Of course in establishing whether the contract is a reasonable or
unreasonable one, the nature of the business must also be considered. What would be a
reasonable restriction as to time and place upon the manufacture of railway locomotive
engines might be a very unreasonable restriction when imposed upon the employment
of a day laborer.

Considering the nature of the business in which the defendant is engaged, in relation
with the limitation placed upon the plaintiff both as to time and place, we are of the
opinion, and so decide, that such limitation is legal and reasonable and not contrary to
public policy.

ROLANDO C. RIVERA, Petitioner, vs. SOLIDBANK CORPORATION, Respondent.


G.R. No. 163269 , April 19, 2006, FIRST DIVISION
CALLEJO, SR., J.:

Facts:

Petitioner had been working for Solidbank Corporation since July 1, 1977. He was
initially employed as an Audit Clerk, then as Credit Investigator, Senior Clerk,
Assistant Accountant, and Assistant Manager. Prior to his retirement, he became the
Manager of the Credit Investigation and Appraisal Division of the Consumer’s Banking
Group.

In December 1994, Solidbank offered two retirement programs to its employees. Since
Rivera was only 45 years old, he was not qualified for retirement under the ORP. Under
the SRP, he was entitled to receive P1,045,258.95 by way of benefits.

Deciding to devote his time and attention to his poultry business in Cavite, Rivera
applied for retirement under the SRP. Solidbank approved the application and Rivera
was entitled to receive the net amount of P963,619.28. Rivera received the amount and
confirmed his separation from Solidbank on February 25, 1995.

Subsequently, Solidbank required Rivera to sign an undated Release, Waiver and


Quitclaim, which contains the following provisions:
I will not, at any time, in any manner whatsoever, directly or indirectly engage in any
unlawful activity prejudicial to the interest of the BANK, its parent, affiliate or
subsidiary companies, their stockholders, officers, directors, agents or employees, and
their successors-in-interest and will not disclose any information concerning the
business of the BANK, its manner or operation, its plans, processes or data of any kind.

The second undertaking is incorporated in the Undertaking following petitioner’s


execution of the Release, Waiver and Quitclaim which reads:
That as a supplement to the Release and Quitclaim, I executed in favor of Solidbank on
FEBRUARY 28, 1995, I hereby expressly undertake that I will not seek employment with
any competitor bank or financial institution within one (1) year from February 28, 1995.
On May 1, 1995, the Equitable Banking Corporation employed Rivera as Manager of its
Credit Investigation and Appraisal Division of its Consumers’ Banking Group. Upon
discovering this, Solidbank wrote a letter informing Rivera that he had violated the
Undertaking. She likewise demanded the return of all the monetary benefits he received
in consideration of the SRP within five (5) days from receipt; otherwise, appropriate
legal action would be taken against him.

When Rivera refused to return the amount demanded within the given period,
Solidbank filed a complaint for Sum of Money with Prayer for Writ of Preliminary
Attachment before the Regional Trial Court.

Issue:
Whether the employment ban incorporated in the Undertaking which petitioner
executed upon his retirement is unreasonable, oppressive, hence, contrary to public
policy;

Held:

Yes.

On the face of the Undertaking, the post-retirement competitive employment ban is


unreasonable because it has no geographical limits; respondent is barred from accepting
any kind of employment in any competitive bank within the proscribed period.
Although the period of one year may appear reasonable, the matter of whether the
restriction is reasonable or unreasonable cannot be ascertained with finality solely from
the terms and conditions of the Undertaking, or even in tandem with the Release,
Waiver and Quitclaim.

RENATO S. GATBONTON, Petitioner, - versus - NATIONAL LABOR RELATIONS


COMMISSION, MAPUA INSTITUTE OF TECHNOLOGY and JOSE CALDERON,
Respondents.
G.R. NO. 146779, January 23, 2006, FIRST DIVISION
AUSTRIA-MARTINEZ, J.:

Facts:
Petitioner Renato S. Gatbonton is an associate professor of respondent Mapua Institute
of Technology (MIT), Faculty of Civil Engineering. Some time in November 1998, a civil
engineering student of respondent MIT filed a letter-complaint against petitioner for
unfair/unjust grading system, sexual harassment and conduct unbecoming of an
academician. Pending investigation of the complaint, respondent MIT placed petitioner
under a 30-day preventive suspension effective January 11, 1999. The committee
believed that petitioner’s continued stay during the investigation affects his
performance as a faculty member, as well as the students’ learning; and that the
suspension will allow petitioner to “prepare himself for the investigation and will
prevent his influences to other members of the community.”

Thus, petitioner filed with the NLRC a complaint for illegal suspension. Petitioner
questioned the validity of the administrative proceedings with the Regional Trial Court
in a petition for certiorari but the case was terminated on May 21, 1999 when the parties
entered into a compromise agreement wherein respondent MIT agreed to publish in the
school organ the rules and regulations implementing Republic Act No. 7877 (R.A. No.
7877) or the Anti-SexualHarassment Act; disregard the previous administrative
proceedings and conduct anew an investigation on the charges againstpetitioner.
Petitioner agreed to recognize the validity of the published rules and regulations, as
well as the authority of respondent to investigate, hear and decide the administrative
case against him.

Issue:

Whether or not the preventive suspension is valid.

Held:

NO. Preventive suspension is a disciplinary measure for the protection of the


company’s property pending investigation of any alleged malfeasance or misfeasance
committed by the employee. The employer may place the worker concerned under
preventive suspension if his continued employment poses a serious and imminent
threat to the life or property of the employer or of his co-workers. However, when it is
determined that there is no sufficient basis to justify an employee’s preventive
suspension, the latter is entitled to the payment of salaries during the time of preventive
suspension.
ALFREDO ROCO and CANDELARIA ROCO, respondents.
[G.R. No. 150660. July 30, 2002] FIRST DIVISION
KAPUNAN, J.:

Facts:

CALS Poultry Supply Corporation is engaged in the business of selling dressed chicken
and other related products and managed by Danilo Yap.

On May 16, 1995, it hired Candelaria Roco as helper, at its chicken dressing plant on a
probationary basis.

With respect to Candelaria Roco, there is no dispute that she was employed on
probationary basis. She was hired on May 16, 1995 and her services were terminated on
November 15, 1995 due to poor work performance. She did not measure up to the work
standards on the dressing of chicken. The Labor Arbiter sustained CALS in terminating
her employment. The NLRC affirmed the Labor Arbiter’s ruling.

The Court of Appeals did not disagree with the NLRC’s finding that Candelaria was
dismissed because she did not qualify as a regular employee in accordance with the
reasonable standards made known by the company to her at the time of her
employment.

However, the Court of Appeals set aside the NLRC ruling on the ground that at the
time Candelaria’s services were terminated, she had attained the status of a regular
employee as the termination on November 15, 1995 was effected four (4) days after the
6-month probationary period had expired, hence, she is entitled to security of tenure in
accordance with Article 281 of the Labor Code.

Issue:

What is the proper computation of the probationary employment period.

Held:

In Cebu Royal v. Deputy Minister of Labor, our computation of the 6-month


probationary period is reckoned from the date of appointment up to the same calendar
date of the 6th month.
[G.R. No. 149859. June 9, 2004]

RADIN C. ALCIRA, petitioner, vs. NATIONAL LABOR RELATIONS


COMMISSION, MIDDLEBY PHILIPPINES CORPORATION/FRANK THOMAS,
XAVIER G. PEÑA and TRIFONA F. MAMARADLO, respondents.
CORONA, J.:

Facts:

The petitioner, Radin Alcira, was hired by the respondent Middleby Philippines
Corporation as engineering support services supervisor under probationary status for 6
months. Afterwards, the service of the petitioner was terminated by the respondent on
the ground that the latter was not satisfied on the performance of the former. As a
result, the petitioner filed a complaint foe illegal dismissal in the National Labor
Relations Commission (NLRC) against the respondent.

Petitioner contended that his termination in the service tantamount to illegal dismissal
since he attained the status of a regular employee as of the time of dismissal. He
presented the appointment paper showing that he was hired on May 20, 1996,
consequently, his dismissal on November 20, 1996 was illegal because at that time, he
was already a regular employee since the 6-month probationary period ended on
November 16, 1996.

The respondent, on the other hand, asserted that during the petitioner’s probationary
period, he showed poor performance on his assigned tasks, was late couple of times and
violated the company’s rule. Thus, the petitioner was terminated and his application to
become a regular employment was disapproved. The respondent also insisted that the
removal of the petitioner from office was within the probationary period.

The Labor Arbiter dismissed the complaint on the ground that the dismissal of the
petitioner was done before his regularization because the 6- month probationary period,
counting from May 20, 1996 shall end on November 20, 1996. The NLRC affirmed the
decision of the Labor Arbiter. The Court of Appeals affirmed the decision of NLRC.
Hence, the present recourse.

Issue:

Whether the petitioner was already a regular employee in respondent’s company at the
time of his dismissal from the service

Held:

The Supreme Court ruled in the negative. The status of the petitioner at the time of his
termination was still probationary. His dismissal on November 20, 1996 was within the
6- month probationary period. Article 13 of the Civil Code provides that when the law
speaks of years, months, and days and nights, it shall be understood that years are of
365 days, months of 30 days, days of 24 hours and nights are from sunset to sunrise.
Since, one month is composed of 30 days, then, 6 months shall be understood to be
composed of 180 days. And the computation of the 6- month period is reckoned from
the date of appointment up to the same calendar date of the 6th month following. Since,
the number of days of a particular month is irrelevant, petitioner was still a
probationary employee at the time of his dismissal. Wherefore, the petition is
dismissed.
G.R. No. 148738
MITSUBISHI MOTORS PHILIPPINES CORPORATION, Petitioner,
- versus -
CHRYSLER PHILIPPINES LABOR UNION and NELSON PARAS,
Respondents.
PUNO, J.,

FACTS

Private respondent Nelson Paras first worked with Mitsubishi Philippines as a shuttle
bus driver on March 19, 1976. He resigned on June 16, 1982 because he went to Saudi
Arabia and worked there as a diesel mechanic and heavy machine operator from 1982
to 1993. Upon his return, Mitsubishi Philippines re-hired him as a welder-fabricator at a
tooling shop from November 1, 1994 to March 3, 1995.

On May 1996, Paras was re-hired again, this time as a probationary manufacturing
trainee at the Plant Engineering Maintenance Department. He had an orientation on
May 15, 1996 and afterwhich, with respect to the company’s rules and guidelines,
started reporting for work on May 27, 1996.
Paras was evaluated by his immediate supervisors after six months of working. The
supervisors rating Paras’ performance were Lito R. Lacambacal and Wilfredo J. Lopez,
as part of the MMPC’s company policies. Upon this evaluation, Paras garnered an
average rating.

Later, respondent Paras was informed by his supervisor, Lacambacal, that he received
an average performance rating but it is a rate which would still qualify him to be
regularized. But as part of the company protocols, the Division Managers namely A.C.
Velando, H.T. Victoria and Dante Ong reviewed the performance evaluation made on
Paras. Despite the recommendations of the supervisors, they unanimously agreed that
the performance was unsatisfactory. As a consequence, Paras was not considered for
regularization.

Paras received a Notice of Termination on November 26, 1996 which was dated
November 25, 1996. This letter’s intent is to formally relieve him off of his services and
position effective the date since he failed to meet the company’s standards.

ISSUE:

Whether or not respondent Paras’ termination was legal or not.

HELD:

The Court holds that a company employer may indeed hire an employee on a
probationary basis in order to determine his fitness to perform work. The Court stresses
the existence of the statements under Article 281 of the Labor Code which specifies that
the employer must inform the employee of the standards they were to meet in order to
be granted regularization and that such probationary period shall not exceed six (6)
months from the date the employee started working, unless specified in the
apprenticeship agreement.

Respondent Paras was employed on a probationary basis and was apprised of the
standards upon which his regularization would be based during the orientation. His
first day to report for work was on May 27, 1996. As per the company's policy, the
probationary period was from three (3) months to a maximum of six (6) months.
Applying Article 13 of the Civil Code, the probationary period of six (6) months consists
of one hundred eighty (180) days. The Court conforms with paragraph one, Article 13 of
the Civil Code providing that the months which are not designated by their names shall
be understood as consisting of thirty (30) days each. This case, the Labor Code pertains
to 180 days. Also, as clearly provided for in the last paragraph of Article 13, it is said
that in computing a period, the first day shall be excluded and the last day included.
Thus, the one hundred eighty (180) days commenced on May 27, 1996, and ended on
November 23, 1996. The termination letter dated November 25, 1996 was served on
respondent Paras only at 3:00 a.m. of November 26, 1996. The Court held that by that
time, he was actually already a regular employee of the petitioner under Article 281 of
the Labor Code. His position as a regularized employee is thus secured until further
notice.

[G.R. No. 149371. April 13, 2005]

ABERDEEN COURT, INC., and RICHARD NG, petitioners, vs. MATEO C.


AGUSTIN JR., respondent.

AZCUNA, J.:

FACTS:

On September 16, 1996, Aberdeen Court, Inc. (Aberdeen), one of the petitioners,
employed Mateo C. Agustin (Agustin), herein respondent, for the purpose of trouble
shooting the electrical problems in said petitioner’s establishment. Agustin was
engaged on a six-month probationary basis. The employment contract provided, inter
alia, that:

Should my performance be considered unsatisfactory at any time by management


during my probationary period, I understand and agree that the management can
terminate my services at any time, even before the termination of the agreed six-month
period.

On January 12 and 13, 1997 the personnel of Centigrade Industries, Inc. performed a
reading of the exhaust air balancing at the fifth and sixth floors of Aberdeen’s premises.
Petitioners claim that Agustin was placed in charge of the undertaking. On the other
hand, Agustin asserts that Engr. Abad merely requested him to accompany the
aforesaid personnel to show the location of the exhaust air outlet at the fifth and sixth
floors of the premises. He avers that:

The request of Engr. Abad is actually the responsibility of the company’s mechanical
engineers. Despite the fact that the request of Engr. Abad is not a part of his job since he
is not a mechanical engineer and there were three (3) other mechanical engineers on
duty in the company premises, petitioner [herein respondent], being a subordinate of
Engr. Abad, obliged and accompanied the aforementioned personnel to the location.
There were no other specific instructions from Engr. Abad to petitioner with respect to
the conduct or actual reading to be made by the Centigrade personnel.

It must be noted that the reading of exhaust air balancing is under the category of
heating, ventilating and air conditioning (HVAC) which are within the realm of field of
work of mechanical engineers. Being an electrical engineer, petitioner obviously has no
knowledge of the procedure and the equipment used by mechanical engineers in the
conduct of the reading of the exhaust air balancing.

After the Centigrade personnel finished their job, they submitted their report to
Agustin. Petitioners allege that Agustin accepted and signed the report, without
verifying its correctness. Engineer Abad later checked the work of the Centigrade
employees only to find out that four rooms in the fifth floor and five rooms in the sixth
floor were incorrectly done. In contrast, Agustin states that after the report was handed
to him, he took the same to Engr. Abad, who he claims was responsible for evaluating
and confirming the said report. Allegedly, instead of signing it himself, Engr. Abad
directed respondent to sign it, giving the reason that Agustin was present when the
reading was conducted. Respondent Agustin complied, but he now points out that his
signature was not accompanied by any qualification that he accepted the report on
behalf of Aberdeen. He claims that he signed merely to evidence that he received a
copy of the report.

The parties also differ on the occurrences two days after the signing of the report or on
January 15, 1997. According to petitioners, Aberdeen management confronted Agustin
with his failure to check the job and asked him to explain his side. Agustin allegedly
ignored management and left the company, which made it impossible for Aberdeen to
transmit any further notice to him.

However, Agustin claims that:


On January 15, 1997 or two days after the report was submitted by Centigrade
Industries, petitioner [herein respondent] was summarily dismissed. In the afternoon
of that day, he received a telephone call from the personnel office of respondent
company ordering him to report to that office after his tour of duty. At about seven
p.m. at the personnel office, Ms. Lani Carlos of the Personnel Department, informed
him that Aberdeen Court is terminating his services as electrical engineer. Petitioner
was flabbergasted. Ms. Carlos then informed him that he could get his two (2) weeks
salary in the amount of P4,000, more or less, on the condition that he will sign some
documents which provides that the company has no more liability and that he is
voluntarily resigning from Aberdeen Court. Aware of his rights, petitioner did not
sign the offered documents. He was then hurriedly led to the door by Ms. Carlos.

The following day or on January 16, 1997, petitioner requested assistance from the
Department of Labor and Employment (DOLE). A DOLE personnel told him to report
for work since private respondents did not serve him a notice of termination. As
instructed, petitioner reported for work on the same day. Upon arriving at the company
premises, petitioner asked Ms. Carlos if he could still report for work but private
respondent’s personnel officer told him that he cannot do so.

ISSUE:

Whether or not Agustin qualified as a regular employee

Ruling:

It can be gleaned from Article 281 of the Labor Code that there are two grounds to
legally terminate a probationary employee. It may be done either: a) for a just cause or
b) when employee fails to qualify as a regular employee in accordance with reasonable
standards made known by the employer to the employee at the start of the
employment.

Petitioners say that Agustin was terminated because he failed to qualify as a regular
employee. Petitioners, however, allegedly did not show that respondent was apprised
of these reasonable standards at the start of the employment.

In Servidad v. NLRC et al., where effectively the probationary period was for one year,
the Court stated:
If the nature of the job did actually necessitate at least one year for the employee to
acquire the requisite training and experience, still, the same could not be a valid
probationary employment as it falls short of the requirement of Article 281 of the Labor
Code. It was not brought to light that the petitioner was duly informed at the start of
his employment, of the reasonable standards under which he could qualify as a regular
employee. The rudiments of due process demand that an employee should be apprised
beforehand of the conditions of his employment and the basis for his advancement.

Similarly, in Secon Philippines Ltd. v. NLRC, the dismissal of the employee was
declared illegal by the Court because the employer did not prove that the employee was
properly apprised of the standards of the job at the time of his engagement and,
naturally, the employer could not show that the employee failed to meet such
standards.
The Implementing Rules of the Labor Code in Book VI, Rule I, Section 6, also provides:
Probationary employment. -- There is probationary employment where the employee,
upon his engagement, is made to undergo a trial period during which the employer
determines his fitness to qualify for regular employment, based on reasonable
standards made known to him at the time of engagement.
Probationary employment shall be governed by the following rules:

(c) The services of an employee who has been engaged on probationary basis may be
terminated only for a just cause, when he fails to qualify as a regular employee in
accordance with the reasonable standards prescribed by the employer.

(d) In all cases of probationary employment, the employer shall make known to the
employee the standards under which he will qualify as a regular employee at the time
of his engagement. Where no standards are made known to the employee at that time,
he shall be deemed a regular employee.

The above rule, however, should not be used to exculpate a probationary employee
who acts in a manner contrary to basic knowledge and common sense, in regard to
which there is no need to spell out a policy or standard to be met. This is what the
NLRC found to be the fact in this case. Said the NLRC:

It bears stressing that even if technically the reading of air exhaust balancing is not
within the realm of expertise of the complainant, still it ought not to be missed that
prudence and due diligence imposed upon him not to readily accept the report handed
to him by the workers of Centigrade Industries. Required of the complainant was that
he himself proceed to the work area, inquire from the workers as to any difficulties
encountered, problems fixed and otherwise observe for himself the progress and/or
condition/quality of the work performed.

As it is, We find it hard to believe that complainant would just have been made to sign
the report to signify his presence. By saying so, complainant is inadvertently degrading
himself from an electrical engineer to a mere watchdog. It is in this regard that We
concur with the respondents that by his omission, lack of concern and grasp of basic
knowledge and common sense, complainant has shown himself to be undeserving of
continued employment from probationary employee to regular employee.
[G.R. No. 145417. December 11, 2003]

FLORENCIO M. DE LA CRUZ, JR., petitioner, vs. NATIONAL LABOR RELATIONS


COMMISSION (4th Division) SHEMBERG MARKETING CORPORATION and
ERNESTO U. DACAY, JR., respondents.
CORONA, J.:

FACTS:

On May 27, 1996, petitioner Florencio M. de la Cruz, Jr. was hired by private respondent
Shemberg Marketing Corporation (Shemberg) as senior sales manager with a monthly
salary of P40,500. Shemberg was engaged in the business of manufacturing, trading,
distributing and importing various consumer products. The position of senior sales
manager was then newly created in line with Shemberg’s objective of product
positioning in the consumer market. Its duties included, among others, the supervision
and control of the sales force of the company. The senior sales manager was also vested
with some discretion to decide on matters within the scope of his functions, including
the appointment of district sales representatives and the reshuffling of salesmen to
achieve sales targets.

However, on September 14, 1996, Shemberg’s human resource department manager,


Ms. Lilybeth Y. Llanto, summoned petitioner and informed him of the management’s
decision to terminate his services. Petitioner asked Llanto for the reason but the latter
merely informed him that it had something to do with the drop in the company’s sales.
Petitioner then requested a meeting with Shemberg’s vice president, Ernesto U. Dacay,
Jr., but was told that the decision of the management was final. His request to be
furnished a 30-day written notice was also denied by the management. Hence,
petitioner filed a complaint for illegal dismissal, non-payment of salary, backwages,
13th month pay and damages against Shemberg, Ernesto Dacay, Jr. and Lilybeth Llanto.
Respondents answered that petitioner’s dismissal was premised on the following: (1)
his poor performance as evidenced by the steady and substantial drop in company sales
since his assumption as senior sales manager; (2) the dissatisfaction of his subordinates
over his management style and dealings with the company’s distributors which
resulted in the low morale of Shemberg’s sales force, as evidenced by the joint affidavit
of two of his subordinates, Ruel O. Salgado and Joel D. Sol; (3) his unauthorized use of
company cellular phone for overseas personal calls and (4) the unauthorized
reimbursement of the plane tickets of his wife and child. In short, petitioner was
terminated for his failure to meet the required company standards and for loss of trust
and confidence.

ISSUES:

Whether or not De la Cruz is a probationary employee

RULING:

Petitioner was holding a managerial position in which he was tasked to perform key
functions in accordance with an exacting work ethic. His position required the full trust
and confidence of his employer. While petitioner could exercise some discretion, this
obviously did not cover acts for his own personal benefit. As found by the court a quo,
he committed a transgression that betrayed the trust and confidence of his employer ―
reimbursing his family’s personal travel expenses out of company funds. Petitioner
failed to present any persuasive evidence or argument to prove otherwise. His act
amounted to fraud or deceit which led to the loss of trust and confidence of his
employer.

We reiterate the well-established rule that findings of fact of the Court of Appeals are
conclusive on the parties and are not generally reviewable by this Court when
supported by substantial evidence.
Petitioner was hired by respondent Shemberg Marketing Corporation on May 27, 1996
and was terminated on September 14, 1996. Article 281 of the Labor Code provides:
Probationary employment – Probationary employment shall not exceed six (6) months
from the date the employee started working, unless it is covered by an apprenticeship
agreement stipulating a longer period. The services of an employee who has been
engaged on a probationary basis may be terminated for a just cause or when he fails to
qualify as a regular employee in accordance with reasonable standards, made known by
the employer to the employee at the time of his engagement. An employee who is
allowed to work after a probationary period shall be considered a regular employee.
Petitioner vigorously contends that he was not a probationary employee since
Shemberg failed to disclose to him the reasonable standards for qualifying as a regular
employee.

This Court notes, however, the evidence on record clearly showing that petitioner was
well informed of the standards to be met before he could qualify as a regular employee.
G.R. No. 148931 September 12, 2006

CATHAY PACIFIC AIRWAYS, LIMITED, petitioner, vs.


PHILIP LUIS F. MARIN and THE HON. COURT OF APPEALS (Former First
Division), respondents.

CALLEJO, SR., J.:

FACTS:

Marin used to work for Saudia Airlines as a ticketing agent. When he applied for
employment as a Reservation Officer in Cathay Pacific Airways, Ltd. (Cathay), he was
interviewed by the following: Senior Supervisor Nenita Montallana, Reservations
Manager Elizabeth Leviste, Staff and Administrative Supervisor M.A. Canizares, and
Country Manager (Philippines) Peter W. Foster.

In a letter dated March 30, 1992, Foster confirmed Marin's appointment as Reservations
Officer effective April 6, 1992 for a probationary period of six months. He was to receive
a monthly salary of P5,334.00, including holidays and rest days, with a promise of a
salary review upon satisfactory completion of the probationary period. The letter also
stated that Cathay reserved the right to "terminate [Marin's] services during the
probationary period if [his] performance proves to be unsatisfactory, in which case, [he]
will receive the salary due [him] at the time of the termination of [his] services." It was
also understood that Marin "had accepted the [recognized] terms of employment," and
that he would be "reconfirmed as a member of [the] regular staff upon completion of
the probationary period."

On October 15, 1992, Marin filed a complaint6 for illegal dismissal against Cathay and
Foster before the NLRC. The complaint was later amended to include claims for 13th
month pay, moral and exemplary damages, and attorney's fees.
ISSUE:

Whether or not Marin is a probationary employee

RULING

It is settled that a probationary employee enjoys only a temporary employment status,


not a permanent status. In general terms, he is terminable anytime as long as such
termination is made before the expiration of the six-month probationary period. The
employment of a probationary employee may only be terminated either (1) for a just
cause; or (2) when the employee fails to qualify as a regular employee in accordance
with the reasonable standards made known to him by the employer at the start of his
employment. The power of the employer to terminate an employee on probation is thus
subject to the following conditions: (1) it must be exercised in accordance with the
specific requirements of the contract; (2) the dissatisfaction on the part of the employer
must be real and in good faith, not prejudicial so as to violate the contract or the law;
and (3) there must be no unlawful discrimination in the dismissal. The burden of
proving just or valid cause for dismissing an employee rests on the employer.

In Secon Philippines, Ltd. v. NLRC, this Court held that the probationary employment
of an employee may be terminated when he fails to qualify as regular employee in
accordance with reasonable standards made known to him by his employer at the time
of employment and after due process; in Manlimos v. National Labor Relations
Commission, it was held that the constitutional protection on the probationary
employee ends upon the expiration of the period provided for in the probationary
contract of employment. Thus, a probationary employee remains secure in his or her
employment during the time that the employment contract remains in effect, but the
moment the probationary employment period expires, the employee can no longer
invoke the constitutional protection. Thereafter, the parties are free to renew the
contract or not; or for the employer to extend to such employee a regular or permanent
employment. If the employee is not given a permanent or regular employment contract
on account of his unsatisfactory work performance, it cannot be said that he was
illegally dismissed. In such case, the contract merely expired.
We agree with the rulings of the Labor Arbiter and NLRC that respondent's
employment was not terminated during the period of his probationary employment,
and that he was not extended a regular employment by petitioner Cathay on account of
his unsatisfactory work performance during the probationary period.

G.R. No. 164582 March 28, 2007

PILAR ESPINA, ELEANOR G. AQUINO, LORENE C. BARNUEVO, MARICRIS S. J.


BANDINO, JULIO M. PETALIO, JR., NOEL T. DE BORJA, REMEGIO P. BASCO,
MATEO D. DEOCAREZA, EMILIANO A. EBREO, BENJAMIN PAZ, LEONORA
PAZ, CLAUDIO DE LOS REYES, LEANDRO R. CELIS, PATERNO FERNANDEZ,
ANICETO M. RODRIGUEZ, DONATO M. PUNZALAN, LOURDES ALFONSO Q.,
ALLAN PANLILIO, DAISY V. ARCEO, ALEJANDRO D. PASCUAL, MA.
CORAZON T. BAJO, ARNOLD M. BLANCO, CRISTITO S. ABELA, DIOSCORO
FAJANILAG, and AGUSTIN WONG, Petitioners,
vs.
HON. COURT OF APPEALS, MONDE M.Y. SAN BISCUIT CORP., M.Y. SAN
BISCUIT INC., MRS. MHEW WHA LIM and MR. KENG SUN MAR, Respondents.

CHICO-NAZARIO, J.:

FACTS:

Respondent M.Y. San Biscuits, Inc. (M.Y. San) was previously engaged in the business
of manufacturing biscuits and other related products.

On 27 December 2000, in a conciliation proceeding before the Department of Labor and


Employment (DOLE) NCMB-NCR Director Leopoldo de Jesus, the duly authorized
representative of M.Y. San Worker’s Union-PTGWO and M.Y. San Sales Force Union-
PTGWO was informed of the closure or cessation of business operations of respondent
M.Y. San as a result of the intended sale of the business and all the assets of respondent
M.Y. San to respondent Monde M.Y. San Corporation (Monde) and was notified of their
termination, effective 31 January 2001.

On 28 December 2000, the written notice of the sale and purchase of the assets of
respondent M.Y San to respondent Monde and of the termination of all the employees
of respondent M.Y. San were filed before the DOLE Regional Office No. IV.5
On 22 January 2001, respondent M.Y. San and the Union signed a Memorandum of
Agreement (MOA) embodying the agreements set forth in the Minutes/Agreement,
dated 27 December 2000. Embodied in the MOA is an agreement that the existing
Collective Bargaining Agreement shall cease to be effective on 31 January 2001 and shall
in no way be binding upon the buyer, respondent Monde, and that respondent M.Y.
San shall provide respondent Monde a list of all its present employees who shall be
given preference in employment by the latter. Pertinent provisions of the Agreement:

The Company agrees to submit the list of all its present employees to the new
corporation for purposes of rehiring if said employee applies and qualifies, subject to
such criteria as the new corporation may impose. In the rehiring, the covered employees
shall be given hiring preference, if qualified. The corresponding Notice as to whom of
the covered employees have been hired by the new corporation shall be issued
immediately after January 31, 2001. During the entire rehiring process and until the
election and qualification of the new officers, the PTGWO, through its National
President, or his authorized representative, shall act as the TRUSTEE of the UNION.

All employees hired by MONDE M.Y. SAN CORPORATION and/or the new owner of
the COMPANY, shall upon hiring, subject to the terms and conditions of their
probationary employment, become members of the UNION. The continued existence of
the UNION in the company and/or MONDE M.Y. SAN CORPORATION shall not be
interrupted by the payment of the Company’s employees of their separation package or
the temporary closure of the Company’s operations.6

On 31 January 2001, all the employees of respondent M.Y. San received their separation
pay and the cash equivalent of their vacation and sick leaves. Thereafter, they signed
their respective Quitclaims.

On 1 February 2001, an Asset Purchase Agreement was executed between respondents


M.Y. San and Monde.

On 2 February 2001, respondent Monde commenced its operations. All the former
employees of respondent M.Y. San who were terminated upon its closure and who
applied and qualified for probationary employment, including petitioners herein,
started working for respondent Monde on a contractual basis for a period of six months.

Subsequently, petitioners were terminated on various dates.


ISSUE:

Whether or not petitioners were illegally terminated


HELD:

Work is a necessity that has economic significance deserving legal protection. The
provisions on social justice and protection to labor in the Constitution dictate so.

However, employers are also accorded rights and privileges to assure their self-
determination and independence and reasonable return of capital. This mass of
privileges comprises the so-called management prerogatives. Although they may be
broad and unlimited in scope, the State has the right to determine whether an
employer’s privilege is exercised in a manner that complies with the legal requirements
and does not offend the protected rights of labor. One of the rights accorded an
employer is the right to close an establishment or undertaking. Just as no law forces
anyone to go into business, no law can compel anybody to continue the same.

The right to close the operations of an establishment or undertaking is explicitly


recognized under the Labor Code as one of the authorized causes in terminating
employment of workers, the only limitation being that the closure must not be for the
purpose of circumventing the provisions on terminations of employment embodied in
the Labor Code. Article 283 of the Labor Code reads:

ART. 283. Closure of establishment and reduction of personnel.—The employer may


also terminate the employment of any employee due to the installation of labor saving
devices, redundancy, retrenchment to prevent losses or the closing or cessation of
operation of the establishment or undertaking unless the closing is for the purpose of
circumventing the provisions of this Title, by serving a written notice on the worker and
the Ministry of Labor and Employment at least one (1) month before the intended date
thereof. x x x. In case of retrenchment to prevent losses and in cases of closures or
cessation of operations of establishment or undertaking not due to serious business
losses or financial reverses, the separation pay shall be equivalent to one (1) month pay
or at least one-half (1/2) month pay for every year of service, whichever is higher. A
fraction of at least six (6) months shall be considered as one (1) whole year. (Emphasis
supplied.)

The phrase "closure or cessation of operations of establishment or undertaking"


includes a partial or total closure or cessation.
The closure, therefore, of the business operation of respondent M.Y. San was not tainted
with bad faith or other circumstance that would give rise to suspicions of malicious
intent. Other than their mere allegations, petitioners failed to present independent
evidence that would otherwise show that the closure of M.Y. San was without factual
basis and done in utter bad faith. Mere allegation is not evidence. It is a basic rule in
evidence that each party must prove his affirmative allegation.

G.R. No. 128682 March 18, 1999


JOAQUIN T. SERVIDAD, petitioner,
vs.
NATIONAL LABOR RELATIONS COMMISSION, INNODATA PHILIPPINES,
INC./ INNODATA CORPORATION, TODD SOLOMON, respondents.

PURISIMA, J.:

FACTS:

Petitioner Joaquin T. Servidad was employed on May 9, 1994 by respondent


INNODATA as a "Data Control Clerk", under a contract of employment Section 2 of
which, reads:

Sec. 2. This Contract shall be effective for a period of 1 years commencing on May 10,
1994, until May 10, 1995 unless sooner terminated pursuant to the provisions hereof.

From May 10, 1994 to November 10, 1994, or for a period of six (6) months, the
EMPLOYEE shall be contractual during which the EMPLOYER can terminate the
EMPLOYEE's services by serving written notice to that effect. Such termination shall be
immediate, or at whatever date within the six-month period, as the EMPLOYER may
determine. Should the EMPLOYEE continue his employment beyond November 10,
1994, he shall become a regular employee upon demonstration of sufficient skill in the
terms of his ability to meet the standards set by the EMPLOYER. If the EMPLOYEE fails
to demonstrate the ability to master his task during the first six months he can be placed
on probation for another six (6) months after which he will be evaluated for promotion
as a regular employee.

On November 9, 1995, or after working for six (6) months, he was made to sign a three-
month probationary employment and later, an extended three-month probationary
employment good until May 9, 1995.

On July 7, 1994, the petitioner was given an overall rating of 100% and 98% in the work
evaluations conducted by the company. In another evaluation, petitioner received a
rating of 98.5% given by the private respondent.

On May 9, 1995, petitioner was dismissed from the service on the ground of alleged
termination of contract of employment.
ISSUE:

Whether or not Servidad is a probationary employee?

RULING:

The private respondent sought to alternatively avail of probationary employment and


employment for a fixed term so as to preclude the regularization of the status of
petitioner. The utter disregard of public policy by the contract in question negates the
ruling of NLRC that said contract is the law between the parties. The private agreement
of the parties cannot prevail over Article 1700 of the Civil Code, which provides:

Art. 1700. The relation between capital and labor are not merely contractual. They
are so impressed with public interest that labor contracts must yield to the common
good. Therefore, such contracts are subject to special laws on labor unions, collective
bargaining, strikes and lockouts, closed shops, wages, working conditions, hours of
labor and similar subjects.

Similarly telling is the case of Pakistan Airlines Corporation vs. Pole, et al. 20 There, it
was said:

. . . provisions of applicable law, especially provisions relating to matters affected with


public policy, are deemed written into the contract. Put a little differently, the
governing principle is that the parties may not contract away applicable provisions of
law especially peremptory provisions dealing with matters heavily impressed with
public interest. The law relating to labor and employment is clearly such an area and
parties are not at liberty to insulate themselves and their relationships from the impact
of labor laws and regulations by simply contracting with each other. . . .

On the averment that NLRC gravely abused its discretion in finding that petitioner
failed to meet the standards of the company, we find for petitioner. The decision at
NLRC on the matter simply stated that the petitioner fell short of the expectations of the
company without specifying factual basis therefor. The public respondent overlooked
the undisputed satisfactory ratings of the performance of petitioner in the two job
evaluations conducted by the respondent company. Even granting, therefore, that the
contract litigated upon is valid; still, the petitioner, who was permitted to work beyond
six months could not be dismissed on the ground of failure to meet the standards of
Innodata. By the provisions of the very contract itself, petitioner has become a regular
employee of private respondent. Therein, it is stipulated that: ". . . Should the
EMPLOYEE continue employment beyond November 10, 1994, he shall become a
regular employee upon demonstration of sufficient skill in the terms of his ability to
meet the standards set by the EMPLOYER. . . ."

[G.R. No. 119253. April 10, 1997]

AMOR CONTI and LEOPOLDO CRUZ, petitioners, vs. NATIONAL LABOR


RELATIONS COMMISSION (Third Division), CORFARM HOLDINGS
CORPORATION, CARLITO J. RABANG and CIPRIANO Q. BARAYANG,
respondents.
PADILLA, J.:

FACTS:

Petitioner Amor Conti was employed by respondent Corfarm as cashier on 2 February


1991. Petitioner Leopoldo Cruz was employed by the same respondent corporation as a
warehouseman on 16 May 1991. Both Amor Conti and Leopoldo Cruz were
subsequently promoted to the positions of Head of Commissary and Store Supervisor,
respectively. In their respective employment contracts with Corfarm, it was stipulated
that their employment shall be coterminous with the effectivity of the contract executed
by and between Corfarm and MERALCO for the management of the latter's
commissary (hereinafter referred to as the "management contract").

On 31 December 1992, said management contract between Corfarm and MERALCO


expired. However, Corfarm continued to operate the MERALCO commissary despite
the non-renewal of said contract.

On 13 January 1993, petitioners received a memorandum, dated 12 January 1993, from


private respondents terminating their services effective on said date, allegedly for two
reasons: 1) the expiration of their employment contracts, these being coterminous with
the management contract between Corfarm and MERALCO, and; 2) the on-going
evaluation of their past performances, and investigation of the internal auditor of
Corfarm of certain anomalous transactions involving them (petitioners).

ISSUE:

Whether or not petitioners were illegally dismissed?

RULING:

At the outset, it will be noted that the Office of the Solicitor General (OSG), in its
"Manifestation and Motion in lieu of Comment", dated 19 June 1995, agreed with the
findings of the labor arbiter that the petitioners were illegally dismissed, and prayed of
this Court that the questioned NLRC decision dated 24 November 1994 and resolution
dated 26 January 1995, be set aside.

Petitioners contend that they were denied due process when they were dismissed
without a written notice (specifying the particular charges constituting the grounds for
their dismissal), and a hearing, as required by law. They further contend that the
memorandum dated 11 January 1993, supposedly issued by Corfarm to petitioners
directing them "to explain why they should not be dismissed for alleged acts of
negligence and carelessness" was never received by them. Besides, said memorandum
did not specify the particular acts or omissions of petitioners. It merely stated that
based on the results of the investigation conducted by Corfarm's internal audit staff,
petitioners were found to have been negligent in the performance of their duties.

Petitioners' contentions are meritorious.

This Court has consistently held that the twin requirements of notice and hearing
constitute essential elements of due process in the dismissal of employees.As to the
requirement of notice, it has been held that the employer must furnish the worker with
two written notices before termination of employment can be legally effected: (a) notice
which apprises the employee of the particular acts or omissions for which his dismissal
is sought, and; (b) subsequent notice which informs the employee of the employer's
decision to dismiss him.

With regard to the requirement of a hearing, this Court has held that the essence of due
process is simply an opportunity to be heard, and not that an actual hearing should
always and indispensably be held.

FABELA vs SAN MIGUEL CORPORATION Case Digest


[G.R. No. 150658 February 9, 2007] NOELITO FABELA, MARCELO DELA CRUZ III,
ROGELIO LASAT, HENRY MALIWANAG, MANUEL DELOS SANTOS, and
ROMMEL QUINES, Petitioners, vs. SAN MIGUEL CORPORATION and ARMAN
HICARTE, Respondents.

FACTS:
Petitioners were hired by respondent San Miguel
Corporation (SMC) as “Relief Salesmen” for the Greater Manila Area (GMA) under
separate but almost similarly worded “ Contracts of Employment with Fixed Period.”
After having entered into successive contracts of the same nature with SMC, the
services of petitioners were terminated after SMC no longer agreed to forge another
contract with them. Respondent SMC claimed that the hiring of petitioners was not
intended to be permanent, as the same was merely occasioned by the need to fill in a
vacuum arising from SMC’s gradual transition to a new system of selling and
delivering its products. Respondents explained that SMC previously operated under
the “Route System,” but began implementing in 1993 the “Pre-Selling System” in which
the salesmen under the earlier system would be replaced by Accounts Specialists which
called for upgraded qualifications. While some of the qualified regular salesmen were
readily upgraded to the position of Accounts Specialist, respondents claimed that SMC
still had to sell its beer products using the conventional routing system during the
transition stage, thus giving rise to the need for temporary employees; and the members
of the regular Route Crew then existing were required to undergo a training program to
determine whether they possessed or could be trained for the necessary attitude and
aptitude required of an Accounts Specialist, hence, the hiring of petitioners and others
for a fixed period, co-terminus with the completion of the transition period and
Training Program for all prospective Accounts Specialists. Claiming that they were
illegally dismissed, petitioners filed complaints for illegal dismissal against
respondents.

ISSUE:

Whether or not petitioners were validly hired for a fixed period.

HELD:

The SC held that under article 280 of the Labor Code, there are two kinds of regular
employees, namely: (1) those who are engaged to perform activities which are necessary
or desirable in the usual business or trade of the employer, and (2) those casual
employees who have rendered at least one year of service, whether continuous or
broken, with respect to the activity in which they are employed. Article 280 also
recognizes project employees, those whose “employment has been fixed for a specific
project or undertaking.” Project employment is distinct from casual employment
referred to in the second paragraph of Article 280 for the proviso that “any employee
who has rendered at least one year of service . . . shall be considered a regular
employee” does not apply to project employees, but only to casual employees.
Although Article 280 does not expressly recognize employment for a fixed period,
which is distinct from employment which has been fixed for a specific project or
undertaking, it has been clarified that employment for a fixed period is not in itself
illegal. Even if the duties of an employee consist of activities usually necessary or
desirable in the usual business of the employer, it does not necessarily follow that the
parties are forbidden from agreeing on a period of time for the performance of such
activities through a contract of employment for a fixed term.

Unfortunately, respondents’ contention that there are fixed periods stated in the
contracts of employment does not lie. Brent instructs that a contract of employment
stipulating a fixed-term, even if clear as regards the existence of a period, is invalid if it
can be shown that the same was executed with the intention of circumventing security
of tenure, and should thus be ignored. Indeed, substantial evidence exists in the present
case showing that the subject contracts were utilized to deprive petitioners of their
security of tenure.The contract of employment of petitioner Fabela, for instance, states
that the transition period from the Route System to the Pre-Selling System would be
twelve (12) months from April 4, 1995. It bears noting, however, that petitioner Fabela,
besides being hired again for another fixed period of four (4) months after the lapse in
April 1996 of the one-year contract, had already been working for respondent SMC on a
fixed-term basis as early as 1992, or one year before respondent SMC even began its
shift to the Pre-selling System in 1993. Thus, there is sufficient basis to believe that the
shift of SMC to the Pre-Selling System was not the real basis for the forging of fixed-
term contracts of employment with petitioners and that the periods were fixed only as a
means to preclude petitioners from acquiring security of tenure. A fixed-term
employment is valid only under certain circumstances, such as when the employee
himself insists upon the period, or where the nature of the engagement is such that,
without being seasonal or for a specific project, a definite date of termination is a sine
qua non. Petition is granted.

G.R. No. 150478. April 15, 2005

HACIENDA BINO/HORTENCIA STARKE, INC./HORTENCIA L. STARKE,


Petitioners,
vs.
CANDIDO CUENCA, FRANCISCO ACULIT, ANGELINA ALMONIA, DONALD
ALPUERTO, NIDA BANGALISAN, ROGELIO CHAVEZ, ELMO DULINGGIS,
MERCEDES EMPERADO, TORIBIO EMPERADO, JULIANA ENCARNADO,
REYNALDO ENCARNADO, GENE FERNANDO, JOVEN FERNANDO, HERNANI
FERNANDO, TERESITA FERNANDO, BONIFACIO GADON, JOSE GALLADA,
RAMONITO KILAYKO, ROLANDO KILAYKO, ALFREDO LASTIMOSO,
ANTONIO LOMBO, ELIAS LOMBO, EMMA LOMBO, LAURENCIA LOMBO,
LUCIA LOMBO, JOEL MALACAPAY, ADELA MOJELLO, ERNESTO MOJELLO,
FRUCTOSO MOJELLO, JESSICA MOJELLO, JOSE MOJELLO, MARITESS
MOJELLO, MERLITA MOJELLO, ROMEO MOJELLO, RONALDO MOJELLO,
VALERIANA MOJELLO, JAIME NEMENZO, RODOLFO NAPABLE, SEGUNDIA
OCDEN, JARDIOLINA PABALINAS, LAURO PABALINAS, NOLI PABALINAS,
RUBEN PABALINAS, ZALDY PABALINAS, ALFREDO PANOLINO, JOAQUIN
PEDUHAN, JOHN PEDUHAN, REYNALDO PEDUHAN, ROGELIO PEDUHAN,
JOSEPHINE PEDUHAN, ANTONIO PORRAS, JR., LORNA PORRAS, JIMMY
REYES, ALICIA ROBERTO, MARCOS ROBERTO, JR., MARIA SANGGA,
RODRIGO SANGGA, ARGENE SERON, SAMUEL SERON, SR., ANGELINO
SENELONG, ARMANDO SENELONG, DIOLITO SENELONG, REYNALDO
SENELONG, VICENTE SENELONG, FEDERICO STA. ANA, ROGELIO SUASIM,
EDNA TADLAS, ARTURO TITONG, JR., JOSE TITONG, JR., NANCY VINGNO,
ALMA YANSON, JIMMY YANSON, MYRNA VILLANUEVA BELENARIO,
SALVADOR MALACAPAY, and RAMELO TIONGCO, Respondents.

CALLEJO, SR., J.:

FACTS:

The petitioner disfavored the fact that the private respondent employees have formed a
union. When the union became the collective bargaining representative in the
certification election, the petitioner refused to sit down to negotiate a CBA. Moreover,
the respondents were not given work for a month amounting to unjustified dismissal.
As a result, the complainants staged a strike to protest but was settled through a
memorandum of agreement which contained a list of those considered as regular
employees for the payroll.

The NLRC held that there was illegal dismissal and this was affirmed by the Court of
Appeals.

ISSUE:

W/N the employees are regular workers

RULING:

Yes, they are regular and not seasonal employees. For them to be excluded as regulars,
it is not enough that they perform work that is seasonal in nature but they also are
employed for the duration of one season. The evidence only proved the first but not the
second requirement.

The ruling in Mercado v. NLRC is not applicable since in that case, the workers were
merely required to perform phases of agricultural work for a definite period of time,
after which, their services are available to other employers. The management's sudden
change of assignment reeks of bad faith, it is likewise guilty of ULP.

G.R. No. 167045 August 29, 2008

COCOMANGAS HOTEL BEACH RESORT and/or SUSAN MUNRO, petitioners,


vs.
FEDERICO F. VISCA, JOHNNY G. BAREDO, RONALD Q. TIBUS, RICHARD G.
VISCA and RAFFIE G. VISCA, respondents.

AUSTRIA-MARTINEZ, J.:

Facts:

The respondents alleged that they were regular employees of petitioners tasked with
the maintenance and repair of the resort facilities. They did not report for work after
they were informed that the ongoing constructions and repairs would be temporarily
suspended because they caused irritation and annoyance to the resort's guests and due
to budgetary constraints. They later on discovered that not less than ten workers were
subsequently hired by petitioners to do repairs in two cottages of the resort and two
workers were retained after the completion without respondents being allowed to
resume work.

Issue:

Whether respondents are regular or project employees

Held:

They are regular employees, not project employees. A project employee is one whose
"employment has been fixed for a specific project or undertaking, the completion or
termination of which has been determined at the time of the engagement of the
employee or where the work or service to be performed is seasonal in nature and the
employment is for the duration of the season." Before an employee hired on a per-
project basis can be dismissed, a report must be made to the nearest employment office,
of the termination of the services of the workers every time completes a project.

In the present case, respondents cannot be classified as project employees, since they
worked continuously for petitioners from three to twelve years without any mention of
a "project" to which they were specifically assigned. While they had designations as
"foreman," "carpenter" and "mason," they performed work other than carpentry or
masonry. They were tasked with the maintenance and repair of the furniture, motor
boats, cottages, and windbreakers and other resort facilities.

There is likewise no evidence of the project employment contracts covering


respondents' alleged periods of employment. More importantly, there is no evidence
that petitioners reported the termination of respondents' supposed project employment
to the DOLE as project employees.

Department Order No. 19, as well as the old Policy Instructions No. 20, requires
employers to submit a report of an employee’s termination to the nearest public
employment office every time his employment is terminated due to a completion of a
project. Petitioners' failure to file termination reports is an indication that the
respondents were not project employees but regular employees.
This Court has held that an employment ceases to be coterminous with specific projects
when the employee is continuously rehired due to the demands of employer’s business
and re-engaged for many more projects without interruption.

The repeated and continuing need for their services is sufficient evidence of the
necessity, if not indispensability, of their services to petitioners' resort business.

That respondents were regular employees is further bolstered by the following


evidence:
(a) the SSS Quarterly Summary of Contribution Payments listing respondents as
employees of
petitioners;
(b) the Service Record Certificates stating that respondents were employees of
petitioners for
periods ranging from three to twelve years and all have given "very satisfactory
performance";
(c) petty cash vouchers showing payment of respondents' salaries and holiday and
overtime pays.

G.R. No. 157680 October 8, 2008

EQUIPMENT TECHNICAL SERVICES or JOSEPH JAMES DEQUITO, petitioners,


vs.
COURT OF APPEALS, ALEX ALBINO, REY ALBINO, JULIUS ABANES, MIGUEL
ALINAB, CHRISTOPHER BIOL, NELSON CATONG, RENATO DULOT, FLORO
PACUNDO, MARCELITO GAMAS, REYNALDO LIMA, SAMMY MESAGAL,
ERNESTO PADILLA, and CONRADO SULIBAGA, respondents.

VELASCO, JR., J.:

FACTS:

Petitioner Equipment Technical Services (ETS) is primarily engaged in the business of


sub-contracting plumbing works of on-going building construction. Among its clients
was Uniwide Sales, Inc. (Uniwide). Petitioner Joseph James Dequito was, during the
period material, occupying the position of manager of ETS,1 albeit the CA referred to
him as ETS’ president. On various occasions involving different projects, ETS hired the
services of private respondents as pipe fitters, plumbers, or threaders.

In December 1998, ETS experienced financial difficulties when Uniwide, its client at the
time, failed to pay for the plumbing work being done at its Coastal Mall. As a result,
ETS was only able to pay its employees 13th month pay equivalent to two weeks’
salary.

Unhappy over what they thought was ETS’ failure to release the balance of their 13th
month pay, private respondents brought their case before the Arbitration Branch of the
NLRC, docketed as NLRC NCR Case No. 00-01-00571-99 and entitled as Alex Albino,
Renato Dulot, Miguel Alinab, Marcelito Gamas, Julius Abanes, Christopher Biol,
Sammy Mesagal, Conrado Sulibaga, Floro Pacundo v. Equipment Technical Services or
Joseph James Dequito.

Later, two other cases were filed against ETS for illegal dismissal and payment of
money claims when the complainants thereat were refused work in another ETS project,
i.e., Richville project, allegedly because they refused to sign individual employment
contracts with ETS. These two other cases were Nelson Catong, Roger Lamayon,
Christopher Lamayon v. Equipment Technical Services or Joseph James Dequito,
docketed as NLRC NCR Case No. 00-02-01429-99; and Rey Albino, Ernesto Padilla,
Reynaldo Lima v. Equipment Technical Services or Joseph James Dequito, docketed as
NLRC NCR Case No. 00-02-01615-99.

The three cases were consolidated before the labor arbiter. Following failed conciliation
efforts, all concerned, except Roger and Christopher Lamayon, submitted, as the labor
arbiter directed, their respective position papers.

Private respondents’ position2 is summed up as follows: (1) they are regular employees
of ETS; (2) ETS dismissed them without cause and without due process after they filed
cases for money claims against ETS in the arbitration branch of the NLRC; (3) ETS has
not paid them their salaries, 13th month pay, service incentive leave pay, overtime pay,
and premium pay for holidays and rest days; and (4) they are entitled to reinstatement
to their former positions with paid backwages in addition to their money claims and
payment of attorney’s fees.

ETS’ position may be summed up as follows: (1) private respondents were its
contractual/project employees engaged for different projects of the company; (2) they
were not illegally dismissed, having been hired on a per project basis; (3) ETS was
unable to fully release private respondents’ 13th month pay because Uniwide failed to
pay for its contracted plumbing project; (4) ETS was forced to abandon the Uniwide
project and undertake another project, the Richville project, because the chances of
being paid by Uniwide were dim; (5) ETS asked private respondents to sign
employment contracts to formalize their previous agreement but said private
respondents refused; and (6) as a result, ETS was constrained to deny employment to
private respondents as it considered the execution of employment contracts part of
management prerogative before employment commences.

ISSUE:

Whether or not employees were project employees?

RULING:

The principal test for determining whether an employee is properly characterized as


"project employee," as distinguished from "regular employee," is whether or not "the
project employee" was assigned to carry out "a specific project or undertaking," the
duration and scope of which were specified at the time the employees were engaged for
that project.8 And as Article 280 of the Labor Code, defining a regular employee vis-à-
vis a project employee, would have it:

Art. 280. Regular and casual employment. – The provisions of written agreement to the
contrary notwithstanding and regardless of the oral agreement of the parties, an
employment shall be deemed to be regular where the employee has been engaged to
perform activities which are usually necessary or desirable in the usual business or
trade of the employer, except where the employment has been fixed for a specific
project or undertaking the completion or termination of which has been determined at
the time of the engagement of the employee x x x.

It bears to stress at the outset that ETS admits hiring or employing private respondents
to perform plumbing works for various projects. Given this postulate, regular
employment may reasonably be presumed and it behooves ETS to prove otherwise, that
is, that the employment in question was contractual in nature ending upon the
expiration of the term fixed in the contract or for a specific project or undertaking. But
the categorical finding of the CA, confirmatory for the most part of that of the labor
arbiter, is that not a single written contract of employment fixing the terms of
employment for the duration of the Uniwide project, or any other project, was
submitted by ETS despite the latter’s allegations that private respondents were merely
contractual employees. Records of payroll and other pertinent documents, such as job
contracts secured by ETS showing that private respondents were hired for specific
projects, were also not submitted by ETS.

Moreover, if private respondents were indeed employed as project employees,


petitioners should have had submitted a report of termination every time their
employment was terminated owing to the completion of each plumbing project. As
correctly held by the CA in its Amended Decision, citing Tomas Lao Construction v.
NLRC, ETS’ failure to report the employment termination and file the necessary papers
after every project completion tends to support the claim of private respondents about
their not being project employees. Under Policy Instruction No. 20, Series of 1977, the
report must be made to the nearest public office employment. The decision in Violeta v.
NLRC is also apropos, particularly when it held:

[The employer] should have filed as many reports of termination as there were
construction projects actually finished if petitioners [employees] were indeed project
employees, considering that petitioners were hired and again [hired] for various
projects or phases of work therein. Its failure to submit reports of termination cannot
but sufficiently convince us further that petitioners are truly regular employees. Just as
important, the fact that petitioners had rendered more than one year of service at the
time of their dismissal overturns private respondent’s allegations that petitioners were
hired for a specific or fixed undertaking for a limited period of time.
CACERES VS. UNIVERSAL ROBINA SUGAR MILLING CORPORATION
G.R. No. 159343, September 28, 2007

FACTS:

Universal Robina Sugar Milling Corporation (respondent) is a corporation engaged in


the cane sugar milling business. Pedy Caseres (petitioner Caseres) started working for
respondent in 1989, while Andito Pael (petitioner Pael) in 1993. At the start of their
respective employments, they were made to sign a Contract of Employment for Specific
Project or Undertaking. Petitioners' contracts were renewed from time to time; until
May 1999 when they were informed that their contracts will not be renewed anymore.
Petitioners filed a complaint for illegal dismissal, regularization, incentive leave pay,
13th month pay, damages and attorney‘s fees.

Issue:

Whether or not the petitioners are seasonal/project/term employees and not regular
employees of respondents

Ruling:

Article 280 of the Labor Code provides:


ART. 280. Regular and Casual Employees. – The provision of written agreement to the
contrary notwithstanding and regardless of the oral agreement of the parties, an
employment shall be deemed to be regular where the employee has been engaged to
perform activities which are usually necessary or desirable in the usual business or
trade of the employer, except where the employment has been fixed for a specific
project or undertaking the completion or termination of which has been determined at
the time of the engagement of the employee or where the work or services to be
performed is seasonal in nature and the employment is for the duration of the season.

An employment shall be deemed to be casual if it is not covered by the preceding


paragraph: Provided, That, any employee who has rendered at least one year of service,
whether such service is continuous or broken, shall be considered a regular employee
with respect to the activity in which he is employed and his employment shall continue
while such actually exists.

The foregoing provision provides for three kinds of employees: (a) regular employees
or those who have been ―engaged to perform activities which are usually necessary or
desirable in the usual business or trade of the employer‖; (b) project employees or those
―whose employment has been fixed for a specific project or undertaking, the
completion or termination of which has been determined at the time of the engagement
of the employee or where the work or services to be performed is seasonal in nature and
the employment is for the duration of the season‖; and (c) casual employees or those
who are neither regular nor project employees. The principal test for determining
whether an employee is a project employee or a regular employee is whether the
employment has been fixed for a specific project or undertaking, the completion or
termination of which has been determined at the time of the engagement of the
employee. A project employee is one whose employment has been fixed for a specific
project or undertaking, the completion or termination of which has been determined at
the time of the engagement of the employee or where the work or service to be
performed is seasonal in nature and the employment is for the duration of the season. A
true project employee should be assigned to a project which begins and ends at
determined or determinable times, and be informed thereof at the time of hiring.
Petitioners contend that respondent's repeated hiring of their services qualifies them to
the status of regular employees.

On this score, the LA ruled:


This is further buttressed by the fact that the relationship between complainants and the
respondent URSUMCO, would clearly reveal that the very nature of the terms and
conditions of their hiring would show that complainants were required to perform
phases of special projects which are not related to the main operation of the respondent
for a definite period, after which their services are available to any farm owner.
The NLRC, agreeing with the LA, further ruled that:
In the case at bar, We note that complainants never bothered to deny that they
voluntarily, knowingly and willfully executed the contracts of employment. Neither
was there any showing that respondents exercised moral dominance on the
complainants, x x x it is clear that the contracts of employment are valid and binding on
the complainants.

The execution of these contracts in the case at bar is necessitated by the peculiar nature
of the work in the sugar industry which has an off milling season. The very nature of
the terms and conditions of complainants' hiring reveals that they were required to
perform phases of special projects for a definite period after, their services are available
to other farm owners. This is so because the planting of sugar does not entail a whole
year operation, and utility works are comparatively small during the off-milling season.

Finally, the CA noted:


Petitioner Pedy Caseres first applied with private respondent URSUMCO on January 9,
1989 as a worker assisting the crane operator at the transloading station. Upon
application, Caseres was interviewed and made to understand that his employment
would be co-terminus with the phase of work to which he would be then assigned, that
is until February 5, 1989 and thereafter he would be free to seek employment elsewhere.
Caseres agreed and signed the contract of employment for specific project or
undertaking. After an absence of more than five (5) months, Caseres re-applied with
respondent as a seasonal project worker assisting in the general underchassis
reconditioning to transport units on July 17, 1989. Like his first assignment, Caseres was
made to understand that his services would be co-terminus with the work to which he
would be then assigned that is from July 17, 1989 to July 20, 1989 and that thereafter he
is free to seek employment elsewhere to which Caseres agreed and readily signed the
contract of employment for specific project or undertaking issued to him. Thereafter
Caseres voluntarily signed several other employment contracts for various
undertakings with a determinable period. As in the first contract, Caseres' services were
co-terminus with the work to which he was assigned, and that thereafter, he was free to
seek employment with other sugar millers or elsewhere.

The nature and terms and conditions of employment of petitioner Andito Pael were the
same as that of his co-petitioner Caseres. It must be noted that there were intervals in
petitioners' respective employment contracts, and that their work depended on the
availability of such contracts or projects. Consequently, the employment of
URSUMCO's work force was not permanent but co-terminous with the projects to
which the
employees were assigned and from whose payrolls they were paid (Palomares vs.
NLRC, 277 SCRA 439).

Petitioners' repeated and successive re-employment on the basis of a contract of


employment for more than one year cannot and does not make
them regular employees. Length of service is not the controlling determinant of the
employment tenure of a project employee (Rada vs. NLRC,)

G.R. No. 112629 July 7, 1995

PHILIPPINE NATIONAL CONSTRUCTION CORPORATION (PNCC), petitioner,


vs.
NATIONAL LABOR RELATIONS COMMISSION, PHILIPPINE OVERSEAS
EMPLOYMENT ADMINISTRATION, BONIFACIO M. ROQUERO, and ALFREDO
I. DAVILA, respondents.

DAVIDE, JR., J.:

FACTS:

Petitioner Philippine National Construction Corporation (PNCC) asks that we set aside
the resolution 1 of public respondent National Labor Relations Commission (NLRC) in
NLRC NCR CA No. 003767-92 dismissing for lack of merit the appeal from the decision
2 of the Philippine Overseas and Employment Administration (POEA) in POEA Case
No. 90-10-1183 entitled "Alfredo Davila and Bonifacio Roquero vs. Philippine National
Construction Corporation (PNCC), et al.."

In the complaint filed with the POEA, private respondents Alfredo Davila and
Bonifacio Roquero sought to recover from the petitioner salary, overtime pay, vacation
and sick leave, and completion bonus differentials; Davila further asked for payment of
his salary corresponding to the unexpired portion of his contract. They therein alleged
that they had been working as security guards of PNCC since 1980. Having passed the
criteria set by PNCC for overseas workers, they were assigned as company security
guards at PNCC Iraq Expressway Project with a salary of US$350.00 a month each.
Their contracts are evidenced by master employment contracts approved by the POEA
which explicitly state:
This is to confirm your employment with the Philippine National Construction
Corporation-Iraq Expressway Project (Employer/Principal)
. . . other relevant data are as follows:

Position :Company Guard


Salary : US$350.00/month
Jobsite: Samawah, Iraq
Commencement of contract: Upon Departure

They departed for Iraq on 14 May 1985; however, before they left they were made to
sign printed forms in blank. The necessary papers for their overseas assignment were
not given to them not until they were already at the Manila International Airport. They
found out to their disgust that contrary to the master employment plan, the printed
forms they had earlier signed in blank already contain an entry that their salary rate is
US$260.00 a month. Thus, private respondent Roquero received only US$260.00 as
monthly salary during his entire two-year assignment in Iraq and three-week extended
period of assignment therein. Private respondent Davila received the same salary until
he was repatriated prior to the expiration of his contract due to a reduction of work
force. For their four-hour daily overtime work, they were paid only two-hour overtime
pay at the rate of US$260.00 per month.

The PNCC resisted the complaint by claiming that the so-called Master Employment
Contracts relied upon by the private respondents were but notices or offers for overseas
employment, and mere offers without acceptance by them do not constitute contracts of
employment. The contracts which bound them were those providing for a salary of
US$260.00 per month.

ISSUE:

Whether or not the private respondents are employees of PNCC

RULING:

It is true that the only way by which a labor case may reach this Court is through a
petition for certiorari under Rule 65 of the Rules of Court. 8 It must, however, be shown
that the NLRC has acted without or in excess of jurisdiction, or with grave abuse of
discretion, and there is no appeal, nor any plain, speedy, and adequate remedy in the
ordinary course of law. 9
Section 14, Rule VII of the New Rules of Procedure of the NLRC allows an aggrieved
party to file a motion for the reconsideration of any order, resolution, or decision of the
Commission based on palpable or patent errors. Such a motion constitutes a plain,
speedy, and adequate remedy which the aggrieved party may avail of.

It is settled that before certiorari may be availed of, the petitioner must have filed a
motion for the reconsideration of the order or act complained of to enable the tribunal,
board, or office concerned to pass upon and correct its mistakes without the
intervention of the higher court. 10 The petitioner has not endeavored to show any
justifiable reason why it did not file a motion for reconsideration to give the NLRC an
opportunity to re-examine its resolution.

At any rate, at the bottom of the petitioner's grievance is an issue of fact. It is doctrinally
entrenched that the factual findings of labor officials are conclusive and binding on this
Court when supported by substantial evidence. 11 An examination of the decision of
the POEA, which was affirmed by the NLRC, discloses that the findings of facts therein
are supported by substantial evidence. Hence, they can no longer be disturbed by this
Court.

Besides, in an earlier case brought by the petitioner and involving the same issue but
with other employees similarly situated as the private respondents, 12 this Court
upheld the resolution of the NLRC affirming the POEA findings as follows:

. . . The only dispute which remains unsolved is whether or not the monthly salary of
herein complainants is US$350.00 a month or US$260.00.

As correctly invoked by complainants paragraph (i) of Article 34 of the Labor Code


prohibits the substitution or alteration of employment contracts approved and verified
by the Department of Labor from the time (of) the actual signing thereof by the parties
up to and including the period of expiration of the same without the approval of the
Department of Labor.

With regard to the first issue in this case the approved contract of employment of the
herein complainants with the respondent is US$350.00 a month. This can be inferred
from the POEA approved contract of employment and by the certification issued by
respondent's chief recruiting officer. This being so, herein complainants have the right
to be paid as monthly salaries the aforementioned amount.
Complainants having been granted voluntarily by the respondent a two-hour daily
overtime (Exh. "G", "G-1") during the durations of their contract, are also entitled to be
paid thereto based on the monthly salaries of US$350.00 and not US$260.00.

The petitioner's contention that the private respondents' claims are barred by laches do
not deserve even a short shrift.
G.R. No. 106090 February 28, 1994

RICARDO FERNANDEZ, petitioner,


vs.
NATIONAL LABOR RELATIONS COMMISSION and D. M. CONSUNJI, INC.,
respondents.

NOCON, J.:

FACTS:

Petitioner was hired as a laborer at the D.M. Consunji, Inc., a construction firm, on
November 5, 1974. He became a skilled welder and worked for private respondent until
March 23, 1986 when his employment was terminated on the ground that the project
petitioner had been assigned to was already completed and there was no more work for
him to do.

Skeptic of private respondent's reason, petitioner brought his plight before the Labor
Arbiter who consolidated the same with three (3) other separate complaints for illegal
dismissal and various money claims against private respondent. After filing their
respective position papers and other documents pertinent to their causes/defenses, the
parties agreed to submit the case for decision based on record.

On May 12, 1988, Labor Arbiter Fernando V. Cinco rendered a decision, finding that
complainants worked continuously in various projects ranging from five (5) to twenty
(20) years and belonged to a work pool.

ISSUE:

Whether or not petitioner belonged in a work pool?

RULING:
Noteworthy in this case is the fact that herein private respondent's lay-off reports and
the termination reports were duly submitted to the then Ministry of Labor and
Employment everytime a project was completed in accordance with Policy Instruction
No. 20, which provides:

Project employees are not entitled to termination pay if they are terminated as a result
of the completion of the project or any phase thereof in which they are employed,
regardless of the number of projects in which they have been employed by a particular
construction company. Moreover, the company is not required to obtain a clearance
from the Secretary of Labor in connection with such termination. What is required of
the company is a report to the nearest Public Employment Office for statistical
purposes.

The presence of this factor makes this case different from the cases decided by the Court
where the employees were deemed regular employees. The cases of Ochoco v. National
Labor Relations Commission, 5 Philippine National Construction Corporation v.
National Labor Relations Commission, 6 Magante v. National Labor Relations
Commission, 7 and Philippine National Construction Corporation v. National Labor
Relations, et al., 8 uniformly held that the failure of the employer to report to the nearest
employment office the termination of workers everytime a project is completed proves
that the employees are not project employees. Contrariwise, the faithful and regular
effort of private respondent in reporting every completion of its project and submitting
the lay-off list of its employees proves the nature of employment of the workers
involved therein as project employees. Given this added circumstance behind
petitioner's employment, it is clear that he does not belong to the work pool from which
the private respondent would draw workers for assignment to other projects at its
discretion.
ALEJANDRO MARAGUINOT, JR. AND PAUILINO ENERO v. NLRC, VIC DEL
ROSARIO, VIVA FILMS
GR No. 120969

Facts:

Maraguinot and Enero were separately hired by Vic Del Rosario under Viva Films as
part of the filming crew. Sometime in May 1992, sought the assistance of their
supervisor to facilitate their request that their salary be adjusted in accordance with the
minimum wage law.

On June 1992, Mrs. Cesario, their supervisor, told them that Mr. Vic Del Rosario would
agree to their request only if they sign a blank employment contract. Petitioners refused
to sign such document. After which, the Mr. Enero was forced to go on leave on the
same month and refused to take him back when he reported for work. Mr. Maraguinot
on the other hand was dropped from the payroll but was returned days after. He was
again asked to sign a blank employment contract but when he refused, he was
terminated.

Consequently, the petitioners sued for illegal dismissal before the Labor Arbiter. The
private respondents claim the following: (a) that VIVA FILMS is the trade name of
VIVA PRODUCTIONS, INC. and that it was primarily engaged in the distribution &
exhibition of movies- but not then making of movies; (b) That they hire contractors
called “producers” who act as independent contractors as that of Vic Del Rosario; and
(c) As such, there is no employee-employer relation between petitioners and private
respondents.

The Labor Arbiter held that the complainants are employees of the private respondents.
That the producers are not independent contractor but should be considered as labor-
only contractors and as such act as mere agent of the real employer. Thus, the said
employees are illegally dismissed.

The private respondents appealed to the NLRC which reversed the decision of the
Labor Arbiter declaring that the complainants were project employees due to the ff.
reasons: (a) Complainants were hired for specific movie projects and their employment
was co-terminus with each movie project; (b)The work is dependent on the availability
of projects. As a result, the total working hours logged extremely varied; (c) The
extremely irregular working days and hours of complainants work explains the lump
sum payment for their service; and (d) The respondents alleged that the complainants
are not prohibited from working with other movie companies whenever they are not
working for the independent movie producers engaged by the respondents.

A motion for reconsideration was filed by the complainants but was denied by NLRC.
In effect, they filed an instant petition claiming that NLRC committed a grave abuse of
discretion in: (a) Finding that petitioners were project employees; (b) Ruling that
petitioners were not illegally dismissed; and (c) Reversing the decision of the Labor
Arbiter.

In the instant case, the petitioners allege that the NLRC acted in total disregard of
evidence material or decisive of the controversy.

Issues:

(a) W/N there exist an employee- employer relationship between the petitioners and
the private respondents.

(b) W/N the private respondents are engaged in the business of making movies.

(c) W/N the producer is a job contractor.

Held:

There exist an employee- employer relationship between the petitioners and the private
respondents because of the ff. reasons that nowhere in the appointment slip does it
appear that it was the producer who hired the crew members. Moreover, it was VIVA’s
corporate name appearing on heading of the slip. It can likewise be said that it was
VIVA who paid for the petitioners’ salaries.

Respondents also admit that the petitioners were part of a work pool wherein they
attained the status of regular employees because of the ff. requisites: (a) There is a
continuous rehiring of project employees even after cessation of a project; (b) The tasks
performed by the alleged “project employees” are vital, necessary and indispensable to
the usual business or trade of the employer; and (c) However, the length of time which
the employees are continually re-hired is not controlling but merely serves as a badge of
regular employment.
Since the producer and the crew members are employees of VIVA and that these
employees’ works deal with the making of movies. It can be said that VIVA is engaged
of making movies and not on the mere distribution of such.

The producer is not a job contractor because of the ff. reasons: (Sec. Rule VII, Book III of
the Omnibus Rules Implementing the Labor Code.)

a. A contractor carries on an independent business and undertakes the contract work on


his own account under his own responsibility according to his own manner and
method, free from the control and direction of his employer or principal in all matters
connected with the performance of the work except as to the results thereof. The said
producer has a fix time frame and budget to make the movies.

b. The contractor should have substantial capital and materials necessary to conduct his
business. The said producer, Del Rosario, does not have his own tools, equipment,
machinery, work premises and other materials to make motion pictures. Such materials
were provided by VIVA.

It can be said that the producers are labor-only contractors. Under Article 106 of the
Labor Code (reworded) where the contractor does not have the requisites as that of the
job contractors.

G.R. No. 121605 February 2, 2000


PAZ MARTIN JO and CESAR JO, petitioners,
vs.
NATIONAL LABOR RELATIONS COMMISSION and PETER MEJILA,
respondents.

QUISUMBING, J.:

FACTS:

Private respondent Peter Mejila worked as barber on a piece rate basis at Dina's Barber
Shop. In 1970, the owner, Dina Tan, sold the barbershop to petitioners Paz Martin Jo
and Cesar Jo. All the employees, including private respondent, were absorbed by the
new owners. The name of the barbershop was changed to Windfield Barber Shop.

The owners and the barbers shared in the earnings of the barber shop. The barbers got
two-thirds (2/3) of the fee paid for every haircut or shaving job done, while one-third
(1/3) went to the owners of the shop.

In 1977, petitioners designated private respondent as caretaker of the shop because the
former caretaker became physically unfit. Private respondent's duties as caretaker, in
addition to his being a barber, were: (1) to report to the owners of the barbershop
whenever the airconditioning units malfunctioned and/or whenever water or electric
power supply was interrupted, (2) to call the laundry woman to wash dirty linen; (3) to
recommend applicants for interview and hiring; (4) to attend to other needs of the shop.
For this additional job, he was given an honorarium equivalent to one-third (1/3) of the
net income of the shop.1âwphi1.nêt

When the building occupied by the shop was demolished in 1986, the barbershop
closed. But soon a place nearby was rented by petitioners and the barbershop resumed
operations as Cesar's Palace Barbershop and Massage Clinic. In this new location,
private respondent continued to be a barber and caretaker, but with a fixed monthly
honorarium as caretaker, to wit: from February 1986 to 1990 — P700; from February
1990 to March 1991 — P800; and from July 1992 P1,300.

In November 1992, private respondent had an altercation with his co-barber, Jorge
Tinoy. The bickerings, characterized by constant exchange of personal insults during
working hours, became serious so that private respondent reported the matter to Atty.
Allan Macaraya of the labor department. The labor official immediately summoned
private respondent and petitioners to a conference. Upon investigation, it was found
out that the dispute was not between private respondent and petitioners; rather, it was
between the former and his fellow barber. Accordingly, Atty. Macaraya directed
petitioners' counsel, Atty. Prudencio Abragan, to thresh out the problem.

During the mediation meeting held at Atty. Abragan's office a new twist was added.
Despite the assurance that he was not being driven out as caretaker-barber, private
respondent demanded payment for several thousand pesos as his separation pay and
other monetary benefits. In order to give the parties enough time to cool off, Atty.
Abragan set another conference but private respondent did not appear in such meeting
anymore.

Meanwhile, private respondent continued reporting for work at the barbershop. But, on
January 2, 1993, he turned over the duplicate keys of the shop to the cashier and took
away all his belongings therefrom. On January 8, 1993, he began working as a regular
barber at the newly opened Goldilocks Barbershop also in Iligan City.

On January 12, 1993, private respondent filed a complaint2 for illegal dismissal with
prayer for payment of separation pay, other monetary benefits, attorney's fees and
damages. Significantly, the complaint did not seek reinstatement as a positive relief.

ISSUE:

Whether or not Magcalas is a regular employee, not just a project employee

RULING:

Absent a clear showing that petitioners and private respondent had intended to pursue
a relationship of industrial partnership, we entertain no doubt that private respondent
was employed by petitioners as caretaker-barber. Initially, petitioners, as new owners of
the barbershop, hired private respondent as barber by absorbing the latter in their
employ. Undoubtedly, the services performed by private respondent as barber is
related to, and in the pursuit of the principal business activity of petitioners. Later on,
petitioners tapped private respondent to serve concurrently as caretaker of the shop.
Certainly, petitioners had the power to dismiss private respondent being the ones who
engaged the services of the latter. In fact, private respondent sued petitioners for illegal
dismissal, albeit contested by the latter. As a caretaker, private respondent was paid by
petitioners wages in the form of honorarium, originally, at the rate of one-third (1/3) of
the shop's net income but subsequently pegged at a fixed amount per month. As a
barber, private respondent earned two-thirds (2/3) of the fee paid per haircut or
shaving job done. Furthermore, the following facts indubitably reveal that petitioners
controlled private respondent's work performance, in that: (1) private respondent had
to inform petitioners of the things needed in the shop; (2) he could only recommend the
hiring of barbers and masseuses, with petitioners having the final decision; (3) he had to
be at the shop at 9:00 a.m. and could leave only at 9:00 p.m. because he was the one who
opened and closed it, being the one entrusted with the key.7 These duties were
complied with by the private respondent upon instructions of petitioners. Moreover,
such task was far from being negligible as claimed by petitioners. On the contrary, it
was crucial to the business operation of petitioners as shown in the preceding
discussion. Hence, there was enough basis to declare private respondent an employee
of petitioners. Accordingly, there is no cogent reason to disturb the findings of the labor
arbiter and NLRC on the existence of employer-employee relationship between herein
private parties.

With regard to the second issue, jurisprudence has laid out the rules and valid ground
for termination of employment. To constitute abandonment, there must be concurrence
of the intention to abandon and some overt acts from which it may be inferred that the
employee concerned has no more interest in working.8 In other words, there must be a
clear, deliberate and unjustified refusal to resume employment and a clear intention to
sever the employer-employee relationship on the part of the employee.

BRENT SCHOOL, INC.DIMACHE vs. RONALDO ZAMORA and DOROTEO R.


ALEGRE
G.R. No. L-48494 February 5, 1990 en banc

FACTS:
Private respondent Doroteo R. Alegre was engaged as athletic director by petitioner
Brent School, Inc. at a yearly compensation of P20,000.00. The contract fixed a specific
term for its existence, five (5) years, i.e., from July 18, 1971, the date of execution of the
agreement, to July 17, 1976. Subsequent subsidiary agreements dated March 15, 1973,
August 28, 1973, and September 14, 1974 reiterated the same terms and conditions,
including the expiry date, as those contained in the original contract of July 18, 1971.

On April 20,1976, Alegre was given a copy of the report filed by Brent School with the
Department of Labor advising of the termination of his services effective on July 16,
1976. The stated ground for the termination was "completion of contract, expiration of
the definite period of employment." Although protesting the announced termination
stating that his services were necessary and desirable in the usual business of his
employer, and his employment lasted for 5 years - therefore he had acquired the status
of regular employee - Alegre accepted the amount of P3,177.71, and signed a receipt
therefor containing the phrase, "in full payment of services for the period May 16, to
July 17, 1976 as full payment of contract."

The Regional Director considered Brent School's report as an application for clearance
to terminate employment (not a report of termination), and accepting the
recommendation of the Labor Conciliator, refused to give such clearance and instead
required the reinstatement of Alegre, as a "permanent employee," to his former position
without loss of seniority rights and with full back wages.

ISSUE:

Whether or not the provisions of the Labor Code, as amended, have anathematized
"fixed period employment" or employment for a term.

RULING:

Respondent Alegre's contract of employment with Brent School having lawfully


terminated with and by reason of the expiration of the agreed term of period thereof, he
is declared not entitled to reinstatement.

The employment contract between Brent School and Alegre was executed on July 18,
1971, at a time when the Labor Code of the Philippines (P.D. 442) had not yet been
promulgated. At that time, the validity of term employment was impliedly recognized
by the Termination Pay Law, R.A. 1052, as amended by R.A. 1787. Prior, thereto, it was
the Code of Commerce (Article 302) which governed employment without a fixed
period, and also implicitly acknowledged the propriety of employment with a fixed
period. The Civil Code of the Philippines, which was approved on June 18, 1949 and
became effective on August 30,1950, itself deals with obligations with a period. No
prohibition against term-or fixed-period employment is contained in any of its articles
or is otherwise deducible therefrom.

It is plain then that when the employment contract was signed between Brent School
and Alegre, it was perfectly legitimate for them to include in it a stipulation fixing the
duration thereof Stipulations for a term were explicitly recognized as valid by this
Court.

The status of legitimacy continued to be enjoyed by fixed-period employment contracts


under the Labor Code (PD 442), which went into effect on November 1, 1974. The Code
contained explicit references to fixed period employment, or employment with a fixed
or definite period. Nevertheless, obscuration of the principle of licitness of term
employment began to take place at about this time.

Article 320 originally stated that the "termination of employment of probationary


employees and those employed WITH A FIXED PERIOD shall be subject to such
regulations as the Secretary of Labor may prescribe." Article 321 prescribed the just
causes for which an employer could terminate "an employment without a definite
period." And Article 319 undertook to define "employment without a fixed period" in
the following manner: …where the employee has been engaged to perform activities
which are usually necessary or desirable in the usual business or trade of the employer,
except where the employment has been fixed for a specific project or undertaking the
completion or termination of which has been determined at the time of the engagement
of the employee or where the work or service to be performed is seasonal in nature and
the employment is for the duration of the season.

Subsequently, the foregoing articles regarding employment with "a definite period" and
"regular" employment were amended by Presidential Decree No. 850, effective
December 16, 1975.

Article 320, dealing with "Probationary and fixed period employment," was altered by
eliminating the reference to persons "employed with a fixed period," and was
renumbered (becoming Article 271).

As it is evident that Article 280 of the Labor Code, under a narrow and literal
interpretation, not only fails to exhaust the gamut of employment contracts to which the
lack of a fixed period would be an anomaly, but would also appear to restrict, without
reasonable distinctions, the right of an employee to freely stipulate with his employer
the duration of his engagement, it logically follows that such a literal interpretation
should be eschewed or avoided. The law must be given a reasonable interpretation, to
preclude absurdity in its application. Outlawing the whole concept of term employment
and subverting to boot the principle of freedom of contract to remedy the evil of
employer's using it as a means to prevent their employees from obtaining security of
tenure is like cutting off the nose to spite the face or, more relevantly, curing a headache
by lopping off the head.

Such interpretation puts the seal on Bibiso upon the effect of the expiry of an agreed
period of employment as still good rule—a rule reaffirmed in the recent case of
Escudero vs. Office of the President (G.R. No. 57822, April 26, 1989) where, in the fairly
analogous case of a teacher being served by her school a notice of termination following
the expiration of the last of three successive fixed-term employment contracts, the Court
held:
Reyes (the teacher's) argument is not persuasive. It loses sight of the fact that her
employment was probationary, contractual in nature, and one with a definitive period.
At the expiration of the period stipulated in the contract, her appointment was deemed
terminated and the letter informing her of the non-renewal of her contract is not a
condition sine qua non before Reyes may be deemed to have ceased in the employ of
petitioner UST. The notice is a mere reminder that Reyes' contract of employment was
due to expire and that the contract would no longer be renewed. It is not a letter of
termination.

Paraphrasing Escudero, respondent Alegre's employment was terminated upon the


expiration of his last contract with Brent School on July 16, 1976 without the necessity of
any notice. The advance written advice given the Department of Labor with copy to
said petitioner was a mere reminder of the impending expiration of his contract, not a
letter of termination, nor an application for clearance to terminate which needed the
approval of the Department of Labor to make the termination of his services effective.
In any case, such clearance should properly have been given, not denied.
G.R. No. 79182 September 11, 1991

PNOC-ENERGY DEVELOPMENT CORPORATION, petitioner,


vs.
NATIONAL LABOR RELATIONS COMMISSION (Third Division) and DANILO
MERCADO, respondents.

PARAS, J.

FACTS:

In June 1985, Danilo Mercado was dismissed by PNOC-Energy Development


Corporation (PNOC-EDC) due to serious acts of dishonesty allegedly committed by
Mercado. Mercado then filed a complaint for illegal dismissal against PNOC-EDC.
PNOC-EDC filed a motion to dismiss on the ground that the Labor arbiter and/or the
National Labor Relations Commission (NLRC) has no jurisdiction over PNOC-EDC
because it is a subsidiary of the Philippine National Oil Company (PNOC), a
government owned or controlled corporation, and as a subsidiary, it is also a GOCC
and as such, the proper forum for Mercado’s suit is the Civil Service Commission.

ISSUE:

Whether or not PBOC-EDC is correct.

HELD:

No. The issue in this case has been decided already in the case of PNOC-EDC vs
Leogardo. It is true that PNOC is a GOCC and that PNOC-EDC, being a subsidiary of
PNOC, is likewise a GOCC. It is also true that under the 1973 Constitution, all GOCCs
are under the jurisdiction of the CSC. However, the 1987 Constitution change all this as
it now provides:
The Civil Service embraces all branches, subdivisions, instrumentalities and agencies of
the Government, including government-owned or controlled corporations with original
charters. (Article IX-B, Section 2 [1])

Hence, the above provision sets the rule that the mere fact that a corporation is a GOCC
does not automatically place it under the CSC. Under this provision, the test in
determining whether a GOCC is subject to the Civil Service Law is the manner of its
creation such that government corporations created by special charter are subject to its
provisions while those incorporated under the general Corporation Law are not within
its coverage.

In the case at bar, PNOC-EDC, even though it is a GOCC, was incorporated under the
general Corporation Law – it does not have its own charter, hence, it is under the
jurisdiction of the MOLE.

Even though the facts of this case occurred while the 1973 Constitution was still in force,
the provisions of the 1987 Constitution regarding the legal matters [procedural aspect]
are applicable because it is the law in force at the time of the decision.
G.R. No. 101013 February 2, 1993

ABRAHAM B. BLANCAFLOR, ANASTACIO T. MERCADO, LEONARDO


DANTES, ANA B. AGAIN, MARVIN B. VICENTE, ROBERTO Z. CALICA,
MARYLYN M. KARGANILLA and LYDIA S. YUSAY, petitioners,
vs.
NATIONAL LABOR RELATIONS COMMISSION, GREGORIO ARANETA
UNIVERSITY FOUNDATION and ILUMINADO G. VALENCIA, respondents.

REGALADO, J.:

FACTS:

On March 15, 1983, Cesar A. Mijares, the former president of respondent Gregorio
Araneta University Foundation (GAUF), sent a letter to the then Minister of Labor and
Employment requesting approval of the Reorganization, Retrenchment and
Restructuring (hereinafter referred to as RRR) Program of the GAUF on the ground of
serious business losses and financial reverses being experienced by the university. 1
In a letter dated March 29, 1983, Minister Blas F. Ople approved the RRR Program
without any serious objection, but with the requirement that the implementation
thereof shall be instituted without prejudice to whatever benefits may have accrued in
favor of the employees concerned. 2

Petitioners in the case at bar are regular members of the faculty of respondent
university and were concurrently holding administrative positions as dean, department
heads and institute secretaries therein. In the implementation of the RRR Program
effective January 1, 1984, herein petitioners were retired but subsequently rehired. Their
appointment to their administrative positions as dean, department heads and institute
secretaries, respectively, had been extended by private respondent from time to time
until the expiration of their last appointment on May 31, 1988.

Private respondent GAUF appealed the decision to the National Labor Relations
Commission (NLRC) which rendered its decision dated May 24, 1991, reversing the
labor arbiter's aforestated decision and dismissing petitioners' complaint for lack of
merit. 4 Petitioners motion for reconsideration and related reliefs was denied in the
resolution of the NLRC dated July 23, 1991.

ISSUE:

Whether the termination of Blancaflor is valid?

RULING:

Yes, there was no illegal dismissal. Petitioners herein were dismissed by reason of the
expiration of their contracts of employment. Petitioners' appointments as dean,
department heads and institute secretaries were for fixed terms of definite periods as
shown by their respective contracts of employment, which all expired on the same date,
Herein petitioners voluntarily signed the appointments extended to them as attested by
their signature over the word "conforme" in their contracts of employment. As we
observed in Brent, it is the practice and policy of educational institutions that
appointment to the positions of department heads and other high administrative offices
are held by faculty members only on a temporary or non-permanent basis either within
a specified term or at the pleasure of the school head or board of regents. There is
nothing whatever amiss in said practice of having teachers serve as administrative
officials for a fixed term or in a non-permanent capacity in order to accord to as many of
the teaching staff as possible the opportunity to serve as dean, principal, or
administrative officer of one type or another.

The alleged lack of notice of termination to petitioners is of no consequence. Petitioners


were lawfully terminated upon the expiration of their contracts with respondent
without the necessity of any notice. When the contract specifies the period of its
duration it terminates on the expiration of such period. A contract for employment for a
definite period terminates by its own term at the end of such period. 14 The general
notice of termination given by respondent university to petitioners was a mere
reminder that their contracts of employment were due to expire and that the contract
would no longer be renewed. 15 Further, it must be noted that after the employment
contracts of herein petitioners as administrative officers expired, they were retained as
faculty members by private respondent.

On the claims of herein petitioners for separation or retirement pay by reason of the
RRR Program of 1984, the contention of private respondents that petitioners are not
entitled to the same, since they were not separated, is not well-taken. The right of herein
petitioners to claim the said benefits under the 1984 RRR Program of respondent
university is unquestionably evident.
Philips Semiconductor vs. Fardiquela, G.R. No. 141717, April 14, 2004

Facts:

The petitioner Philips Semiconductors (Phils.), Inc. is a domestic corporation engaged in


the production and assembly of semiconductors such as power devices, RF modules,
CATV modules, RF and metal transistors and glass diods. It caters to domestic and
foreign corporations that manufacture computers, telecommunications equipment and
cars. Aside from contractual employees, the petitioner employed 1,029 regular workers.
The employees were subjected to periodic performance appraisal based on output,
quality, attendance and work attitude. One was required to obtain a performance rating
of at least 3.0 for the period covered by the performance appraisal to maintain good
standing as an employee. On May 8, 1992, respondent Eloisa Fadriquela executed a
Contract of Employment with the petitioner in which she was hired as a production
operator with a daily salary of P118. Her initial contract was for a period of three
months up to August 8, 1992, but was extended for two months when she garnered a
performance rating of 3.15. Her contract was again renewed for two months or up to
December 16, 1992, when she received a performance rating of 3.8.After the expiration
of her third contract, it was extended anew, for three months, that is, from January 4,
1993 to April 4, 1993. After garnering a performance rating of 3.4, the respondent‘s
contract was extended for another three months, that is, from April 5, 1993 to June 4,
1993. She, however, incurred five absences in the month of April, three absences in the
month of May and four absences in the month of June. Line supervisor Shirley F.
Velayo asked the respondent why she incurred the said absences, but the latter failed to
explain her side. The respondent was warned that if she offered no valid justification
for her absences, Velayo would have no other recourse but to recommend the non-
renewal of her contract. The respondent still failed to respond, as a consequence of
which her performance rating declined to 2.8. Velayo recommended to the petitioner
that the respondent‘s employment be terminated due to habitual absenteeism, in
accordance with the Company Rules and Regulations. Thus, the respondent‘s contract
of employment was no longer renewed.

Issues and Rulings:

(a) whether or not the respondent was still a contractual employee of the petitioner as of
June 4, 1993;

The two kinds of regular employees under the law are (1) those engaged to perform
activities which are necessary or desirable in the usual business or trade of the
employer; and (2) those casual employees who have rendered at least one year of
service, whether continuous or broken, with respect to the activities in which they are
employed. The primary standard to determine a regular employment is the reasonable
connection between the particular activity performed by the employee in relation to the
business or trade of the employer. The test is whether the former is usually necessary or
desirable in the usual business or trade of the employer. If the employee has been
performing the job for at least one year, even if the performance is not continuous or
merely intermittent, the law deems the repeated and continuing need for its
performance as sufficient evidence of the necessity, if not indispensability of that
activity to the business of the employer. Hence, the employment is also considered
regular, but only with respect to such activity and while such activity exists.[22] The
law does not provide the qualification that the employee must first be issued a regular
appointment or must be declared as such before he can acquire a regular employee
status. In this case, the respondent was employed by the petitioner on May 8, 1992 as
production operator. She was assigned to wire building at the transistor division. There
is no dispute that the work of the respondent was necessary or desirable in the business
or trade of the petitioner.

She remained under the employ of the petitioner without any interruption since May 8,
1992 to June 4, 1993 or for one (1) year and twenty-eight (28) days. The original contract
of employment had been extended or renewed for four times, to the same position, with
the same chores. Such a continuing need for the services of the respondent is sufficient
evidence of the necessity and indispensability of her services to the petitioner‘s
business. By operation of law, then, the respondent had attained the regular status of
her employment with the petitioner, and is thus entitled to security of tenure as
provided for in Article 279 of the Labor Code which reads:

(b) whether or not the petitioner dismissed the respondent from her employment;
if so, whether or not she was accorded the requisite notice and investigation prior to her
dismissal; and

On the second and third issues, we agree with the appellate court that the respondent
was dismissed by the petitioner without the requisite notice and without any formal
investigation. Given the factual milieu in this case, the respondent‘s dismissal from
employment for incurring five (5) absences in April 1993, three (3) absences in May 1993
and four (4) absences in June 1993, even if true, is too harsh a penalty. We do agree that
an employee may be dismissed for violation of reasonable regulations/rules
promulgated by the employer. Dismissal is the ultimate penalty that can be meted to an
employee. Where a penalty less punitive would suffice, whatever missteps may have
been committed by the worker ought not to be visited with a consequence so severe
such as dismissal from employment. For, the Constitution guarantees the right of
workers to ―security of tenure.‖ The misery and pain attendant to the loss of jobs then
could be avoided if there be acceptance of the view that under certain circumstances of
the case the workers should not be deprived of their means of livelihood.
Price, et al., v Innodata Phils., G.R. No. 178505, September 30, 2008

Facts:

INNODATA had since ceased operations due to business losses in June 2002.
Petitioners Cherry J. Price, Stephanie G. Domingo, and Lolita Arbilera were employed
as formatters by INNODATA. The parties executed an employment contract
denominated as a Contract of Employment for a Fixed Period, stipulating that the
contract shall be effective from FEB. 16, 1999 to FEB. 16, 2000 a period of ONE YEAR.
On 16 February 2000, the HRAD Manager of INNODATA wrote petitioners informing
them of their last day of work, at the end of the close of business hours On February 16,
2000. According to INNODATA, petitioners employment already ceased due to the end
of their contract.

On 22 May 2000, petitioners filed a Complaint for illegal dismissal and damages against
respondents. Petitioners claimed that they should be considered regular employees
since their positions as formatters were necessary and desirable to the usual business of
INNODATA as an encoding, conversion and data processing company. Petitioners
finally argued that they could not be considered project employees considering that
their employment was not coterminous with any project or undertaking, the
termination of which was predetermined. Respondents asserted that petitioners were
not illegally dismissed, for their employment was terminated due to the expiration of
their terms of employment.
The Labor Arbiter issued its Decision finding petitioners complaint for illegal dismissal
and damages meritorious.

Respondent INNODATA appealed the Labor Arbiters Decision to the NLRC. The
NLRC reversed the Labor Arbiters Decision dated 17 October 2000, and absolved
INNODATA of the charge of illegal dismissal.

On 25 September 2006, the Court of Appeals promulgated its Decision sustaining the
ruling of the NLRC that petitioners were not illegally dismissed. Hence, this petition.

Issues:

Whether petitioners were illegally dismissed by respondents


Whether petitioners were hired by INNODATA under valid fixed-term employment
contracts

Ruling:

The Court finds merit in the present Petition. There were no valid fixed-term contracts
and petitioners were regular employees of the INNODATA who could not be dismissed
except for just or authorized cause.

The employment status of a person is defined and prescribed by law and not by what
the parties say it should be. Equally important to consider is that a contract of
employment is impressed with public interest such that labor contracts must yield to
the common good. Thus, provisions of applicable statutes are deemed written into the
contract, and the parties are not at liberty to insulate themselves and their relationships
from the impact of labor laws and regulations by simply contracting with each other.
Regular employment has been defined by Article 280 of the Labor
Code, as amended, which reads: Art. 280. Regular and Casual Employment. The
provisions of written agreement to the contrary notwithstanding and regardless of the
oral agreement of the parties, an employment shall be deemed to be regular where the
employee has been engaged to perform activities which are usually necessary or
desirable in the usual business or trade of the employer, except where the employment
has been fixed for a specific project or undertaking the completion or termination of
which has been determined at the time of engagement of the employee or where the
work or services to be performed is seasonal in nature and employment is for the
duration of the season.
An employment shall be deemed to be casual if it is not covered by the preceding
paragraph. Provided, That, any employee who has rendered at least one year of service,
whether such service is continuous or broken, shall be considered a regular employee
with respect to the activity in which he is employed and his employment shall continue
while such activity exists.

Based on the afore-quoted provision, the following employees are accorded regular
status: (1) those who are engaged to perform activities which are necessary or desirable
in the usual business or trade of the employer, regardless of the length of their
employment; and (2) those who were initially hired as casual employees, but have
rendered at least one year of service, whether continuous or broken, with respect to the
activity in which they are employed.

Undoubtedly, petitioners belong to the first type of regular employees.

Under Article 280 of the Labor Code, the applicable test to determine whether an
employment should be considered regular or non-regular is the reasonable connection
between the particular activity performed by the employee in relation to the usual
business or trade of the employer.

In the case at bar, petitioners were employed by INNODATA on 17 February 1999 as


formatters. The primary business of INNODATA is data encoding, and the formatting
of the data entered into the computers is an essential part of the process of data
encoding. Formatting organizes the data encoded, making it easier to understand for
the clients and/or the intended end users thereof. Undeniably, the work performed by
petitioners was necessary or desirable in the business or trade of INNODATA.

However, it is also true that while certain forms of employment require the
performance of usual or desirable functions and exceed one year, these do not
necessarily result in regular employment under Article 280 of the Labor Code. Under
the Civil Code, fixed-term employment contracts are not limited, as they are under the
present Labor Code, to those by nature seasonal or for specific projects with
predetermined dates of completion; they also include those to which the parties by free
choice have assigned a specific date of termination.

The decisive determinant in term employment is the day certain agreed upon by the
parties for the commencement and termination of their employment relationship, a day
certain being understood to be that which must necessarily come, although it may not
be known when. Seasonal employment and employment for a particular project are
instances of employment in which a period, where not expressly set down, is
necessarily implied.

While this Court has recognized the validity of fixed-term employment contracts, it has
consistently held that this is the exception rather than the general rule. More
importantly, a fixed-term employment is valid only under certain circumstances. In
Brent, the very same case invoked by respondents, the Court identified several
circumstances wherein a fixed-term is an essential and natural appurtenance, to wit:

Some familiar examples may be cited of employment contracts which may be neither
for seasonal work nor for specific projects, but to which a fixed term is an essential and
natural appurtenance: overseas employment contracts, for one, to which, whatever the
nature of the engagement, the concept of regular employment with all that it implies
does not appear ever to have been applied, Article 280 of the Labor Code
notwithstanding; also appointments to the positions of dean, assistant dean, college
secretary, principal, and other administrative offices in educational institutions, which
are by practice or tradition rotated among the faculty members, and where fixed terms
are a necessity without which no reasonable rotation would be possible. Similarly,
despite the provisions of Article 280, Policy Instructions No. 8 of the Minister of Labor
implicitly recognize that certain company officials may be elected for what would
amount to fixed periods, at the expiration of which they would have to stand down, in
providing that these officials, "x x may lose their jobs as president, executive vice-
president or vice president, etc. because the stockholders or the board of directors for
one reason or another did not reelect them."
[G.R. No. 122178. February 25, 1999]

DANILO DIMABAYAO, petitioner, vs. NATIONAL LABOR RELATIONS


COMMISSION, ISLAND BISCUIT INC. and CHENG SUY EH, respondents.
BELLOSILLO, J.:

FACTS:

DANILO DIMABAYAO seeks to set aside through this petition for certiorari under
Rule 65 of the 1997 Rules of Civil Procedure the 15 March 1995 Decision and 23 June
1995 Resolution of the National Labor Relations Commission (NLRC) which modified
the Decision of the Labor Arbiter finding private respondents guilty of having illegally
dismissed petitioner from their employ.

Private respondent Island Biscuit, Inc., is engaged in the manufacture of biscuits with
private respondent Cheng Suy Eh as its General Manager. On 5 April 1983 it employed
petitioner with the specific task of operating the roller, cutting biscuits, sorting out
rejects, mashing flour and feeding the flour mass into its thinning machine.

On 30 July 1992, while petitioner was assigned to sort out rejects, with prior permission
first obtained from his checker, he went to the comfort room to answer the call of nature
and relieve himself, afterwhich he returned to his work place. But private respondent
Cheng Suy Eh was unhappy seeing petitioner away from his work station and
immediately demanded from him a written explanation allegedly for abandoning his
work. As a matter of policy, respondent company discourages its employees from
going to the comfort room during working hours for sanitary or hygienic purposes as
the company is engaged in the food business.[1]

The following day, 31 July 1992, Marcela Lok, respondent company's Personnel
Manager, handed petitioner a letter asking him to explain in writing why he left his
work station on 17 and 30 July 1992. Petitioner verbally explained that he never left his
station on 17 July while on 30 July he only went to the comfort room for a short while to
answer the call of nature.[2] Believing that this denial was enough he did not anymore
submit any written explanation. But, for his inability to submit a written explanation,
petitioner was suspended for fifteen (15) days which he contested before the Arbitration
Branch of the NLRC.

On 20 October 1992 petitioner requested a fellow worker to replace him in his work
station so he could go to the comfort room to relieve himself. Again private respondent
Cheng Suy Eh noticed petitioner's brief absence and so, upon his return, his manager
berated him again and required him to submit once more a written explanation for
allegedly abandoning his work. Petitioner complied.

Finding petitioner's explanation not satisfactory, respondent company through its


Personnel Officer Marcela Lok served petitioner a notice of termination.

Petitioner thereafter amended his complaint before the NLRC to include illegal
dismissal among his causes of action in view of his termination from the service.

On 21 September 1994 the Labor Arbiter declared the suspension of petitioner valid and
legal not because he left his production area to relieve himself but for his utter disregard
of the directive of the manager to submit his written explanation. His dismissal
however was found illegal, but because of the strained relationship between the parties,
the Labor Arbiter further held that reinstatement was no longer feasible and thereafter
awarded petitioner a limited back wages for six (6) months without reinstatement.
Thus private respondents were in addition required to pay petitioner service incentive
leave pay of P615.00, proportionate thirteenth month pay of P2,132.00, separation pay of
P14,391.00, and 10% attorney's fees of P3,632.60.

ISSUE:

Whether or not the acts of Dimabayao constitute willful neglect of duties.

RULING:

No. While it may be true that complainant has been leaving his work area without
permission, this Arbitration Board finds that complainant's habit of going to the toilet in
the morning during production is merely a call of nature and by force of habit he had to
relieve himself. Whether or not the complainant relieved himself is not the issue. The
call of nature is a reasonable reason for him to leave his work area. Although
complainant is not entirely without fault since he has been leaving his workplace
without permission from his supervisor and his disrespect towards his superiors as
borne out by the reports of his supervisor and guards, the infraction committed by the
complainant is not so grave that would warrant the ultimate penalty of dismissal . . .

The Labor Arbiter, in effect, opined that a grave injustice would be committed against
the employee if the penalty imposed was grossly disproportionate to the wrong he
committed.[12] At most, a 7-day suspension without pay - for not asking permission
from his supervisor before answering a call of nature, if that be considered an infraction
at all! - should have been sufficient penalty for petitioner.

The NLRC also endeavored to justify its decision by taking into account offenses
allegedly committed by petitioner way back in 1990. These offenses as enumerated in
the NLRC decision were infractions imputed to petitioner prior to the 17 July, 30 July
and 20 October 1992 incidents. As such, they should have been outrightly ignored by
the NLRC in determining and upholding the validity of petitioner's dismissal since, as
may be gleaned from the termination letter, petitioner's dismissal was based merely on
the 17 July, 30 July and 20 October 1992 alleged incidents, without reference to any
infraction committed before then. This only shows that the offenses attributed to
petitioner before 17 July 1992 were mere afterthoughts conceived in the course of the
trial to further justify his dismissal. To refer to those alleged earlier violations as further
grounds for dismissal is undoubtedly prejudicial to petitioner. Significantly, it would
also be doubly prejudicial to him to penalize him for those committed on 17 and 30 July
1992 as he was already suspended for fifteen (15) days for those infractions. This,
obviously, denied petitioner procedural due process and deprived him of his right to be
heard, to refute and present evidence to controvert such accusations prior to his actual
dismissal from employment.

As a consequence, petitioner is entitled to reinstatement.[13] The postulate advanced by


the Labor Arbiter that there existed "strained relationship" between the parties, thus
barring reinstatement of petitioner, does not hold water. Strained relationship may be
invoked only against employees whose positions demand trust and confidence, or
whose differences with their employer are of such nature or degree as to preclude
reinstatement. In the instant case, however, the relationship between petitioner, an
ordinary employee, and management was clearly on an impersonal level. Petitioner
did not occupy such a sensitive position as would require complete trust and
confidence, and where personal ill will would foreclose his reinstatement.[14] But,
interestingly, petitioner himself was praying for his reinstatement.
[G.R. No. 144899. February 5, 2004]

ELIZABETH C. BASCON and NOEMI V. COLE, petitioners, vs. HONORABLE


COURT OF APPEALS, METRO CEBU COMMUNITY HOSPITAL, INC., and
GREGORIO IYOY, respondents.
QUISUMBING, J.:

FACTS:

The petitioners in the instant case were employees of private respondent Metro Cebu
Community Hospital, Inc. (MCCH) and members of the Nagkahiusang Mamumuo sa
Metro Cebu Community Hospital (NAMA-MCCH), a labor union of MCCH employees.
Petitioner Elizabeth C. Bascon had been employed as a nurse by respondent MCCH
since May 1984. At the time of her termination from employment in April 1996, she
already held the position of Head Nurse. The other petitioner, Noemi V. Cole, had been
working as a nursing aide with MCCH since August 1974. Both petitioners were
dismissed by the respondent hospital for allegedly participating in an illegal strike.

The instant controversy arose from an intra-union conflict between the NAMA-MCCH
and the National Labor Federation (NFL), the mother federation of NAMA-MCCH. In
November 1995, NAMA-MCCH asked MCCH to renew their Collective Bargaining
Agreement (CBA), which was set to expire on December 31, 1995. NFL, however,
opposed this move by its local affiliate. Mindful of the apparent intra-union dispute,
MCCH decided to defer the CBA negotiations until there was a determination as to
which of said unions had the right to negotiate a new CBA.

Believing that their union was the certified collective bargaining agent, the members
and officers of NAMA-MCCH staged a series of mass actions inside MCCH’s premises
starting February 27, 1996. They marched around the hospital putting up streamers,
placards and posters.

On March 13 and 19, 1996, the Department of Labor and Employment (DOLE) office in
Region 7 issued two (2) certifications stating that NAMA-MCCH was not a registered
labor organization. This finding, however, did not deter NAMA-MCCH from filing a
notice of strike with the Region 7 Office of the National Conciliation and Mediation
Board (NCMB). Said notice was, however, disregarded by the NCMB for want of legal
personality of the union.
Meanwhile, the MCCH management received reports that petitioners participated in
NAMA-MCCH’s mass actions. Consequently, notices were served on all union
members, petitioners included, asking them to explain in writing why they were
wearing red and black ribbons and roaming around the hospital with placards. In their
collective response dated March 18, 1996, the union members, including petitioners,
explained that wearing armbands and putting up placards was their answer to MCCH’s
illegal refusal to negotiate with NAMA-MCCH.

Subsequently, on March 28, 1996, MCCH notified the petitioners that they were to be
investigated for their activities in the mass actions, with the hearings being scheduled
on March 28, 1996 and April 1, 1996. Petitioners, however, denied receiving said notices.
In a notice dated April 8, 1996, MCCH ordered petitioners to desist from participating
in the mass actions conducted in the hospital premises with a warning that non-
compliance therewith would result in the imposition of disciplinary measures.
Petitioners again claimed they did not receive said order. Petitioners Bascon and Cole
were then served notices terminating their employment effective April 12, 1996 and
April 19, 1996, respectively.

ISSUE:

Whether or not the acts of the Petitioners were just cause for their termination?

RULING:

No, the acts of the did not constitute petitioners were terminated for allegedly
participating in an illegal strike and gross insubordination to the order prohibiting them
from wearing armbands and putting up placards, not for ipso facto failing to show up
in the scheduled investigation. Thus, the real issue is whether or not petitioners were
validly terminated for (1) allegedly participating in an illegal strike and/or (2) gross
insubordination to the order to stop wearing armbands and putting up placards.

As to the first ground, Article 264 (a) of the Labor Code provides in part that:

…Any union officer who knowingly participates in illegal strike and any worker or
union officer who knowingly participates in the commission of illegal acts during a
strike may be declared to have lost his employment status… (Emphasis ours)

Thus, while a union officer can be terminated for mere participation in an illegal strike,
an ordinary striking employee, like petitioners herein, must have participated in the
commission of illegal acts during the strike (underscoring supplied). There must be
proof that they committed illegal acts during the strike.[14] But proof beyond
reasonable doubt is not required. Substantial evidence, which may justify the
imposition of the penalty of dismissal, may suffice.

In this case, the Court of Appeals found that petitioners’ actual participation in the
illegal strike was limited to wearing armbands and putting up placards. There was no
finding that the armbands or the placards contained offensive words or symbols. Thus,
neither such wearing of armbands nor said putting up of placards can be construed as
an illegal act. In fact, per se, they are within the mantle of constitutional protection
under freedom of speech.

Evidence on record shows that various illegal acts were committed by unidentified
union members in the course of the protracted mass action. And we commiserate with
MCCH, patients, and third parties for the damage they suffered. But we cannot hold
petitioners responsible for acts they did not commit. The law, obviously solicitous of the
welfare of the common worker, requires, before termination may be considered, that an
ordinary union member must have knowingly participated in the commission of illegal
acts during a strike.

As regards the appellate court’s finding that petitioners were justly terminated for gross
insubordination or willful disobedience, Article 282 of the Labor Code provides in part:

An employer may terminate an employment for any of the following causes:

(a) Serious misconduct or willful disobedience by the employee of the lawful orders of
his employer or representative in connection with his work.

However, willful disobedience of the employer’s lawful orders, as a just cause for
dismissal of an employee, envisages the concurrence of at least two requisites: (1) the
employee's assailed conduct must have been willful, that is, characterized by a
wrongful and perverse attitude; and (2) the order violated must have been reasonable,
lawful, made known to the employee and must pertain to the duties which he had been
engaged to discharge.[15]

In this case, we find lacking the element of willfulness characterized by a perverse


mental attitude on the part of petitioners in disobeying their employer’s order as to
warrant the ultimate penalty of dismissal. Wearing armbands and putting up placards
to express one’s views without violating the rights of third parties, are legal per se and
even constitutionally protected. Thus, MCCH could have done well to respect
petitioners’ right to freedom of speech instead of threatening them with disciplinary
action and eventually terminating them.

NAGKAKAISANG LAKAS NG MANGGAGAWA SA KEIHIN (NLMK-OLALIA-


KMU)
and HELEN VALENZUELA vs. KEIHIN PHILIPPINES CORPORATION,
G.R. No. 171115, August 9, 2010
FACTS:

Petitioner Helen Valenzuela (Helen) was a production associate in respondent Keihin


Philippines Corporation (Keihin), a company engaged in the production of intake
manifold and throttle body used in motor vehicles manufactured by Honda.

It is a standard operating procedure of Keihin to subject all its employees to


reasonable search before they leave the company premises.[4] On September 5, 2003,
while Helen was about to leave the company premises, she saw a packing tape near her
work area and placed it inside her bag because it would be useful in her transfer of
residence. When the lady guard on duty inspected Helen’s bag, she found the packing
tape inside her bag. The guard confiscated it and submitted an incident report[5] dated
September 5, 2003 to the Guard-in-Charge, who, in turn, submitted a memorandum[6]
regarding the incident to the Human Resources and Administration Department on the
same date.

The following day, or on September 6, 2003, respondent company issued a show cause
notice[7] to Helen accusing her of violating F.2 of the company’s Code of Conduct,
which says, “Any act constituting theft or robbery, or any attempt to commit theft or
robbery, of any company property or other associate’s property. Penalty: D
(dismissal).”[8] Paul Cupon, Helen’s supervisor, called her to his office and directed
her to explain in writing why no disciplinary action should be taken against her.

Helen, in her explanation,[9] admitted the offense and even manifested that she would
accept whatever penalty would be imposed upon her. She, however, did not reckon
that respondent company would terminate her services for her admitted offense.[10]

On September 26, 2003, Helen received a notice[11] of disciplinary action informing her
that Keihin has decided to terminate her services.

On October 15, 2003, petitioners filed a complaint[12] against respondent for illegal
dismissal, non-payment of 13th month pay, with a prayer for reinstatement and
payment of full backwages, as well as moral and exemplary damages. Petitioners
alleged that Helen’s act of taking the packing tape did not constitute serious
misconduct, because the same was done with no malicious intent.[13] They believed
that the tape was not of great value and of no further use to respondent company since
it was already half used. Although Helen admitted that she took the packing tape,
petitioners claimed that her punishment was disproportionate to her infraction.
Keihin, on the other hand, maintained that Helen was guilty of serious misconduct
because there was a deliberate act of stealing from the company. Respondent company
also claimed that motive and value of the thing stolen are irrelevant in this case.

ISSUE:

Whether or not the dismissal of Helen was valid based on serious misconduct

RULING:

It is clear that petitioners failed to include the name of the dismissed employee Helen
Valenzuela in the caption of their petition for certiorari filed with the CA as well as in
the body of the said petition. Instead, they only indicated the name of the labor union
Nagkakaisang Lakas ng Manggagawa sa Keihin (NLMK-OLALIA) as the party acting
on behalf of Helen. As a result, the CA rightly dismissed the petition based on a formal
defect.

Under Section 7, Rule 3 of the Rules of Court, “parties in interest without whom no final
determination can be had of an action shall be joined as plaintiffs or defendants.” If
there is a failure to implead an indispensable party, any judgment rendered would have
no effectiveness. It is “precisely ‘when an indispensable party is not before the court
(that) an action should be dismissed.’ The absence of an indispensable party renders all
subsequent actions of the court null and void for want of authority to act, not only as to
the absent parties but even to those present.” The purpose of the rules on joinder of
indispensable parties is a complete determination of all issues not only between the
parties themselves, but also as regards other persons who may be affected by the
judgment. A decision valid on its face cannot attain real finality where there is want of
indispensable parties.

At any rate, we are aware that it is the policy of courts to encourage full adjudication of
the merits of an appeal. Dismissal of appeals purely on technical grounds, especially an
appeal by a worker who was terminated and whose livelihood depends on the speedy
disposition of her case, is frowned upon. Thus, while we affirm the CA’s dismissal of
the petition for certiorari, we shall still discuss the substantive aspect of the case and go
into the merits.
[G.R. No. 158232. March 31, 2005]

FUJITSU COMPUTER PRODUCTS CORPORATION OF THE PHILIPPINES and


ERNESTO ESPINOSA, petitioners, vs. THE HONORABLE COURT OF APPEALS,
VICTOR DE GUZMAN and ANTHONY P. ALVAREZ, respondents.
CALLEJO, SR., J.:

FACTS:

Petitioner Fujitsu Computer Products Corporation of the Philippines (FCPP) is a


corporation organized and existing under Philippine laws with business address at the
Special Export Processing Zone, Carmelray, Canlubang, Calamba, Laguna. It is
engaged in the manufacture of hard disc drives, MR heads and other computer storage
devices for export.[2]

Respondent Victor de Guzman began working for FCPP on September 21, 1997 as
Facilities Section Manager. As of 1999, he was also holding in a concurrent capacity the
position of Coordinator ISO 14000 Secretariat and was receiving a monthly salary of
P43,100.00[3]

Respondent Allan Alvarez, on the other hand, was employed as a Senior Engineer on
April 21, 1998. He was assigned at the Facilities Department under the supervision of
respondent De Guzman, and was then earning P16,800.00.[4]

The garbage and scrap materials of FCPP were collected and bought by the Saro’s
Trucking Services and Enterprises (Saro’s). On January 15, 1999, respondent De
Guzman as Facilities Section Manager, for and in behalf of FCPP, signed a Garbage
Collection Agreement[5] with Saro’s, and the latter’s signatory therein was its owner
and general manager, Larry Manaig.

Sometime in the third week of July 1999, petitioner Ernesto Espinosa, HRD and General
Affairs Director of FCPP, received a disturbing report from Manaig. Manaig reported
that respondent De Guzman had caused the “anomalous disposal of steel [purlins][6]
owned by FCPP.”[7] Two of Manaig’s employees, Roberto Pumarez[8] and Ma. Theresa
S. Felipe,[9] executed written statements detailing how respondent De Guzman had
ordered the steel purlins to be brought out.

ISSUE:

Whether or not the acts of Victor De Guzman constitute serious misconduct?

RULING:

After a careful and painstaking study of the records of the case, the Court rules that the
respondents’ dismissal from employment was not grounded on any of the just causes
enumerated under Article 282 of the Labor Code.

The term “trust and confidence” is restricted to managerial employees.[33] In this case,
it is undisputed that respondent De Guzman, as the Facilities Section Manager,
occupied a position of responsibility, a position imbued with trust and confidence.
Among others, it was his responsibility to see to it that the garbage and scrap materials
of petitioner FCPP were adequately managed and disposed of. Thus, respondent De
Guzman was entrusted with the duty of handling or taking care of the property of his
employer, i.e., the steel purlins which the petitioners allege the respondent prematurely
declared as scrap materials.

However, to be valid ground for dismissal, loss of trust and confidence must be based
on a willful breach of trust and founded on clearly established facts. A breach is willful
if it is done intentionally, knowingly and purposely, without justifiable excuse, as
distinguished from an act done carelessly, thoughtlessly, heedlessly, or inadvertently.
It must rest on substantial grounds and not on the employer’s arbitrariness, whims,
caprices or suspicion; otherwise, the employee would eternally remain at the mercy of
the employer.[34] Loss of confidence must not be indiscriminately used as a shield by
the employer against a claim that the dismissal of an employee was arbitrary. And, in
order to constitute a just cause for dismissal, the act complained of must be work-
related and shows that the employee concerned is unfit to continue working for the
employer.[35]

The Court had the occasion to reiterate in Nokom v. National Labor Relations
Commission[36] the guidelines for the application of the doctrine of loss of confidence-

a. loss of confidence should not be simulated;


b. it should not be used as a subterfuge for causes which are improper, illegal or
unjustified;
c. it may not be arbitrarily asserted in the face of overwhelming evidence to the
contrary; and
d. it must be genuine, not a mere afterthought to justify earlier action taken in bad faith.
[37]
In the case at bar, the grounds relied upon by petitioner FCPP in terminating the
employment of respondent De Guzman are contained in the Inter-Office Memorandum
dated August 23, 1999 which effectively terminated the latter’s employment:

We have carefully evaluated your case and we are convinced that you have committed
grave abuse of authority amounting to serious misconduct and willful breach of trust
and confidence.
Based on our findings, as supported by strong and competent evidences, and contrary
to your explanation per your Letter dated July 26, 1999, the following facts were
satisfactorily established:

1. That sometime in the first week of July 1999, you intimated to Mr. Roberto
Pumarez, Supervisor of Saro’s Trucking Services, your intention to buy from Saro’s the
metals which were then piled up and kept inside the Fuji Electric Philippines’
compound;

2. Thereafter, you ordered the metals to be sold to Saro’s Trucking Services so that
you can buy them (metals) later from Saro’s at the price of P3.00 per kg., which price
you yourself imposed on them;

3. However, it turned out later some pieces of metals which you have earlier
declared as scraps and ordered to be sold to Saro’s were still to be used in the
construction of FCPP’s Building B. Thus, on July 10, 1999, while Saro’s employees were
initially loading the metals, an Engineer of SNK Philippines, Inc., FCPP’s building
contractor, stopped them. It was only later after they were prevented from further
loading the metals that you checked with the SNK personnel if the metals can already
be disposed of as scraps which prove that you have prematurely declared the metals as
scrap;

4. That through Mr. [Adrian] Camcaman, your subordinate Technician, you


instructed the personnel of Saro’s to deliver the metals to Sta. Rosa Baptist Church,
where you are an active Church member;

5. That, as of this date, you have not yet settled/paid your obligation to Saro’s. That
immediately after you were placed under preventive suspension and to support your
explanation that the transaction was between Saro’s and Sta. Rosa Baptist Church, you
caused, through some people representing to be members of the Baptist Church and
who are unknown to Saro’s, to issue a check in favor of Saro’s. When this failed,
another person, representing to be a member of the Baptist church and who appeared
for the first time, went to the office of Saro’s and tried to serve a letter addressed to Mr.
Larry Manaig, Saro’s Proprietor, allegedly inquiring about the total obligation of the
Baptist Church to Saro’s but, which was again not accepted as, in truth and in fact, there
was really no transaction between Saro’s and the Sta. Rosa Baptist Church. All along, it
was you and Mr. Camcaman who dealt directly with Saro’s.
6. That in previous occasions, it was reported by Mr. Manaig that you solicited from
him empty drums, pails and corrugated cartons which were all part of those scraps
picked up from FCPP and you never paid any of them, a fact which you never denied in
your explanation which is tantamount to admission.

Based on the foregoing, it is our well-discerned view that the transaction was
exclusively limited between you and Saro’s. Except for your self-serving explanation,
you failed miserably to present direct evidence that it was the Sta. Rosa Baptist church
which bought the subject metals from Saro’s, as what you want us to believe. At best,
your explanation is a mere afterthought desperately concocted to exculpate yourself.

As Facilities Manager, a very sensitive and confidential position, the nature of your
work demands of you that your actions should not be tainted with any suspicion or
impropriety. However, you failed in this regard and abused your position to advance
your self-interest.

G.R. No. 166039 June 26, 2006

DIGITEL* TELECOMMUNICATIONS PHILIPPINES, INC., JOHNSON ROBERT L.


GO** and ERIC J. SEVERINO,*** Petitioners,
vs.
MARIQUIT SORIANO, Respondent.

CARPIO MORALES, J.:

FACTS:

Mariquit Soriano (Soriano) was hired as Director of Marketing by Digitel


Telecommunications Philippines, Inc. (Digitel). Soriano worked under Vice President
for Business Division Eric J. Severino (Severino) and Senior Executive Vice President
Johnson Robert L. Go (Go). Following a professional dispute against Severino and Go,
Soriano filed a resignation letter which was accepted by her superiors.
After her resignation, Soriano filed a suit for illegal termination alleging that she was
forced to resign due to professional and sexual harassment. She alleged that her
superiors are preventing her former colleagues in testifying to the sexual harassment.
She produced an affidavit by one of the persons involved with Digitel stating that the
employees of the company were being forced not to testify against Go and Severino. In
defense, Go and Severino provided witnesses that testified that the acts alleged by
Soriano din not happen.

The Labor Arbiter held that Mariquit voluntarily resigned, thus dismissing the
complaint. On appeal, the NLRC affirmed the findings of the Labor Arbiter. The Court
of Appeals reversed the decision of NLRC. Hence,this petition.

ISSUE:
Whether or not the Soriano was forced to resign, due to professional and sexual
harassment, thus amounting to constructive dismissal.

HELD:
Soriano’s own allegation, although they are so detailed, appear incredible if not
downright puny. An analysis of her statements shows that her own conclusion that she
was being sexually and professionally harassed was on the basis of her own
suppositions, conjectures, and surmises.

She could not satisfactorily explain her allegation that she was consistently
professionally harassed by respondent Severino. The latter’s alleged words: “How come
you claim you know so much yet nothing ever gets done in your department?” do not
jurisprudentially constitute nor clearly establish “professional harassment.” Aside from
these words, the complainant could only venture to allege instances in general and
vague terms. As to the facts allegedly constituting “sexual harassment” advanced by Go
and Severino, after an objective analysis over their assertions as stated in their
respective counter-affidavits and further considering the other supporting documents
attached to the respondents’ pleadings, it is found that these far out weigh the Soriano’s
own evidence

A reading of the affidavit of the witness, who was never an employee nor present at the
party of Digitel, reveals, however, that she merely “concluded” that the employees of
Digitel were instructed or harassed not to testify in favor of Soriano when they failed to
meet one Matet Ruiz, a Digitel employee “who kept avoiding to meet with such
tendency to threaten resignation every time higher management would refuse her
demand to transfer subordinates who had administrative differences with her, we
therefore have no doubt that complainant voluntarily resigned when respondent
Severino refused to heed her demand that Ms. Arnedo and Ms. Inductivo, her
subordinates, be transferred to other departments. We also have no doubt that such
resignation does not constitute constructive dismissal, much less an illegal one.

[G.R. No. 124617. April 28, 2000]

PHILIPPINE AEOLUS AUTOMOTIVE UNITED CORPORATION and/or FRANCIS


CHUA, petitioners, vs. NATIONAL LABOR RELATIONS COMMISSION and
ROSALINDA C. CORTEZ, respondents.

BELLOSILLO, J.:

FACTS:

Petitioner Philippine Aeolus Automotive United Corporation (PAAUC) is a corporation


duly organized and existing under Philippine laws, petitioner Francis Chua is its
President while private respondent Rosalinda C. Cortez was a company nurse[1] of
petitioner corporation until her termination on 7 November 1994. Jlexj

On 5 October 1994 a memorandum was issued by Ms. Myrna Palomares, Personnel


Manager of petitioner corporation, addressed to private respondent Rosalinda C. Cortez
requiring her to explain within forty-eight (48) hours why no disciplinary action should
be taken against her (a) for throwing a stapler at Plant Manager William Chua, her
superior, and uttering invectives against him on 2 August 1994; (b) for losing the
amount of P1,488.00 entrusted to her by Plant Manager Chua to be given to Mr. Fang of
the CLMC Department on 23 August 1994; and, (c) for asking a co-employee to punch-
in her time card thus making it appear that she was in the office in the morning of 6
September 1994 when in fact she was not. The memorandum however was refused by
private respondent although it was read to her and discussed with her by a co-
employee. She did not also submit the required explanation, so that while her case was
pending investigation the company placed her under preventive suspension for thirty
(30) days effective 9 October 1994 to 7 November 1994. Lexjuris

On 20 October 1994, while Cortez was still under preventive suspension, another
memorandum was issued by petitioner corporation giving her seventy-two (72) hours
to explain why no disciplinary action should be taken against her for allegedly failing to
process the ATM applications of her nine (9) co-employees with the Allied Banking
Corporation. On 21 October 1994 private respondent also refused to receive the second
memorandum although it was read to her by a co-employee. A copy of the
memorandum was also sent by the Personnel Manager to private respondent at her last
known address by registered mail. Jurismis

Meanwhile, private respondent submitted a written explanation with respect to the loss
of the P1,488.00 and the punching-in of her time card by a co-employee.

On 3 November 1994 a third memorandum was issued to private respondent, this time
informing her of her termination from the service effective 7 November 1994 on
grounds of gross and habitual neglect of duties, serious misconduct and fraud or willful
breach of trust.

ISSUE:

Whether or not Rosalinda Cortez was sexually harassed?

RULING:

We are not persuaded. The gravamen of the offense in sexual harassment is not the
violation of the employee's sexuality but the abuse of power by the employer. Any
employee, male or female, may rightfully cry "foul" provided the claim is well
substantiated. Strictly speaking, there is no time period within which he or she is
expected to complain through the proper channels. The time to do so may vary
depending upon the needs, circumstances, and more importantly, the emotional
threshold of the employee. Esmsoâ
Private respondent admittedly allowed four (4) years to pass before finally coming out
with her employer's sexual impositions. Not many women, especially in this country,
are made of the stuff that can endure the agony and trauma of a public, even corporate,
scandal. If petitioner corporation had not issued the third memorandum that
terminated the services of private respondent, we could only speculate how much
longer she would keep her silence. Moreover, few persons are privileged indeed to
transfer from one employer to another. The dearth of quality employment has become a
daily "monster" roaming the streets that one may not be expected to give up one's
employment easily but to hang on to it, so to speak, by all tolerable means. Perhaps, to
private respondent's mind, for as long as she could outwit her employer's ploys she
would continue on her job and consider them as mere occupational hazards. This
uneasiness in her place of work thrived in an atmosphere of tolerance for four (4) years,
and one could only imagine the prevailing anxiety and resentment, if not bitterness,
that beset her all that time. But William Chua faced reality soon enough. Since he had
no place in private respondent's heart, so must she have no place in his office. So, he
provoked her, harassed her, and finally dislodged her; and for finally venting her pent-
up anger for years, he "found" the perfect reason to terminate her. Mseä sm

In determining entitlement to moral and exemplary damages, we restate the bases


therefor. In moral damages, it suffices to prove that the claimant has suffered anxiety,
sleepless nights, besmirched reputation and social humiliation by reason of the act
complained of.[22] Exemplary damages, on the other hand, are granted in addition to,
inter alia, moral damages "by way of example or correction for the public good"[23] if
the employer "acted in a wanton, fraudulent, reckless, oppressive or malevolent
manner."
[A.M. No. CTA-01-1. April 2, 2002]

ATTY. SUSAN M. AQUINO, complainant, vs. HON. ERNESTO D. ACOSTA,


Presiding Judge, Court of Tax Appeals, respondent.

SANDOVAL-GUTIERREZ, J.:

Facts: On November 21, 2000, she reported for work after her vacation in the U.S.,
bringing giftsfor the three judges of the CTA, including respondent. In the afternoon of
the same day, heentered her room and greeted her by shaking her hand. Suddenly, he
pulled her towards him andkissed her on her cheek.On December 28, 2000, while
respondent was on official leave, he called complainant by phone,saying he will get
something in her office. Shortly thereafter, he entered her room, shook her hand and
greeted her, "Merry Christmas." Thereupon, he embraced her and kissed her. She
wasable to free herself by slightly pushing him away.On the first working day in
January, 2001, respondent phoned complainant, asking if she couldsee him in his
chambers in order to discuss some matters. When complainant arrived
there,respondent tried to kiss her but she was able to evade his sexual attempt.Weeks
later, after the Senate approved the proposed bill expanding the jurisdiction of the
CTA,while complainant and her companions were congratulating and kissing each
other, respondentsuddenly placed his arms around her shoulders and kissed her.In the
morning of February 14, 2001, respondent called complainant, requesting her to go to
hisoffice. She then asked Ruby Lanuza, a clerk in the Records Section, to accompany
her.Fortunately, when they reached his chambers, respondent had left.The last incident
happened the next day. At around 8:30 a.m., respondent called complainant andasked
her to see him in his office to discuss the Senate bill on the CTA. She again
requestedRuby to accompany her. The latter agreed but suggested that they should act
as if they met byaccident in respondents office. Ruby then approached the secretarys
table which was separatedfrom respondents office by a transparent glass. For her part,
complainant sat in front of respondent's table and asked him what he wanted to know
about the Senate bill. Respondentseemed to be at a loss for words and kept glancing at
Ruby who was searching for something atthe secretary's desk. Forthwith, respondent
approached Ruby, asked her what she was looking for and stepped out of the office.
When he returned, Ruby said she found what she was looking for and left. Respondent
then approached complainant saying, me gusto akong gawin sa iyo kahapon pa.
Thereupon, he tried to grab her. Complainant instinctively raised her hands to protect
herself but respondent held her arms tightly, pulled her towards him and kissed her.
She pushed himaway, then slumped on a chair trembling. Meantime, respondent sat on
his chair and covered hisface with his hands. Thereafter, complainant left crying and
locked herself inside a comfortroom. After that incident, respondent went to her office
and tossed a note stating, sorry, it wonthappen again.Issue: Whether or not Judge
Acosta is guilty of sexually harassment.Held: No, Judge Acosta is not guilty of sexual
harassment. He is exonerated of the chargesagainst him and is advised to be more
circumspect in his deportment. “A mere casual buss on the cheek is not a sexual
conduct or favor and does not fallwithin the purview of sexual harassment under R.A.
No. 7877. Section 3 (a) thereof provides, towit:'Sec. 3.
Work, Education or Training - related Sexual Harassment Defined
. - Work, education or training-related sexual harassment is committed by an employer,
employee, manager, supervisor,agent of the employer, teacher, instructor, professor,
coach, trainor, or any other person who,having authority, influence or moral
ascendancy over another in a work or training or educationenvironment, demands,
requests or otherwise requires any sexual favor from the other, regardlessof whether
the demand, request or requirement for submission is accepted by the object of
saidAct.a)In a work-related or employment environment, sexual harassment is
committed when:1)The sexual favor is made as a condition in the hiring or in the
employment, re-employment or continued employment of said individual, or in
granting said individualfavorable compensation, terms, conditions, promotions or
privileges; or the refusal togrant sexual favor results in limiting, segregating or
classifying the employee which inanyway would discriminate, deprive or diminish
employment opportunities or otherwiseadversely affect said employees;2) The above
acts would impair the employee's right or privileges under existing labor laws;or 3) The
above acts would result in an intimidating, hostile, or offensive environment for
theemployee.'"Clearly, under the foregoing provisions, the elements of sexual
harassment are as follows:1)The employer, employee, manager, supervisor, agent of the
employer, teacher, instructor, professor, coach, trainor, or any other person has
authority, influence or moralascendancy over another;2)The authority, influence or
moral ascendancy exists in a working environment;
3)
The employer, employee, manager, supervisor, agent of the employer, teacher,
instructor, professor, coach, or any other person having authority, influence or moral
ascendancymakes a demand, request or requirement of a sexual favor.”Indeed, from the
records on hand, there is no showing that respondent judge demanded,requested or
required any sexual favor from complainant in exchange for favorablecompensation,
terms, conditions, promotion or privileges specified under Section 3 of R.A. 7877.

G.R. No. 155831 February 18, 2008

MA. LOURDES T. DOMINGO, petitioner,


vs.
ROGELIO I. RAYALA, respondent.

NACHURA, J.:

FACTS:

On November 16, 1998, Ma. Lourdes T. Domingo (Domingo), then Stenographic


Reporter III at the NLRC, filed a Complaint for sexual harassment against Rayala before
Secretary Bienvenido Laguesma of the Department of Labor and Employment
(DOLE).The committee constituted found Rayala guilty of the offense charged.
Secretary Laguesma submitted a copy of the Committee Report and Recommendation
to the OP, but with the recommendation that the penalty should be suspension for six
(6) months and one (1) day, in accordance with AO 250.On May 8, 2000, the OP issued
AO 119, disagreeing with there commendation that respondent be meted only the
penalty of suspension for six (6) months and one (1) day considering the circumstances
of the case because of the nature of the position of Reyala as occupying the highest
position in the NLRC, being its Chairman. Long digest by Ernani Tadili.It was ordered
that Rayala be dismissed from service for being found guilty of grave offense of
disgraceful and immoral conduct. Rayala filed Motions for Reconsideration until the
case was finally referred to the Court of Appeals for appropriate action. The CA found
Reyala guilty and imposed the penalty of suspension of service for th emaximum
period of one (1) year. Domingo filed a Petition for Review before the SC. Rayala
likewise filed a Petition for Review with this Court essentially arguing that he is not
guilty of any act of sexual harassment. The Republic then filed its own Petition for
Review

ISSUE:

Whether or not there was sexual harassment?

RULING:

Rayala asserts that Domingo has failed to allege and establish any sexual favor,
demand, or request from petitioner in exchange for her continued employment or for
her promotion. According to Rayala, the acts imputed to him are without malice or
ulterior motive. It was merely Domingo¶s perception of malice in his alleged acts ± a
"product of her own imagination" that led her to file the sexual harassment complaint
.Likewise, Rayala assails the OP¶s interpretation, as upheld by the CA, that RA 7877 is
malum prohibitum such that the defense of absence of malice is unavailing. He argues
that sexual harassment is considered an offense against a particular person, not against
society as a whole. Rayala next argues that AO 250 expands the acts proscribed in
RA7877. In particular, he assails the definition of the forms of sexual harassment:
FORMS OFSEXUAL HARASSMENT
Section 1.
Forms of Sexual Harassment.
Sexual harassment may be committed in any of the following forms: a) Overt sexual
advances ;b) Unwelcome or improper gestures of affection; c) Request or demand for
sexual favors including but not limited to going out on dates, outings or the like for the
same purpose; d) Any other act or conduct of a sexual nature or for purposes of sexual
gratification which is generally annoying, disgusting or offensive to the victim.

He posits that these acts alone without corresponding demand, request, or requirement
do not constitute sexual harassment as contemplated by the law.
He alleges that the rule-making power granted to the employer in Section 4(a) of RA
7877 is limited only to procedural matters. The law did not delegate to the employer the
power to promulgate rules which would provide other or additional forms of sexual
harassment, or to come up with its own definition of sexual harassment.
[G.R. No. 132164. October 19, 2004]

CIVIL SERVICE COMMISSION, petitioner, vs. ALLYSON BELAGAN, respondent.


SANDOVAL-GUTIERREZ, J.:

FACTS:

2 separate complaints for sexual harassment and various malfeasances were filed
against Dr. Belagan, the Superintendent of DECS

1st (MAGDALENA’s): She was applying for a permit to operate a pre


-school and during the inspection of the pre-school, Belagan placed his arms around her
shoulders and kissed her cheeks. When she followed up her
application, Belagan replied, “Magdate muna tayo.”

2nd (LIGAYA ANNAWI): She alleged in her complaint that on four separate occasions,
respondent touched her breasts, kissed her cheek, touched her groins, embraced her
from behind and pulled her close to him, his organ pressing the lower part of her back.

DECS joint investigation: Belagan denied sexual harassment accusations. Presented


evidence against admin acts.

DECS Sec: GUILTY of 4 counts of sexual “indignities or harassments” committed


against Ligaya; and two (2)counts of “sexual advances or indignities” against
Magdalena; DISMISSED from service. Absolved of admin malfeasance and dereliction
of duty.

CSC: affirm DECS Sec but dismissed complaint of Ligaya. Transgression against
Magdalena constitutes grave misconduct.
CA: dismissed Magdalena’s complaint, reversed CSC Resolutions. Magdalena is an
unreliable witness, her character being questionable. Magdalena was previously
charged with 22 offenses before MTC Baguio and 23 complaints before brgy captains of
Brgy Silang and Hillside in Baguio. “Given her aggressiveness and propensity for
trouble, “she is not one whom any male would attempt to steal a kiss.”

ISSUE1.
WON Magdalena is a credible witness
WON Belegan is guilty of grave misconduct.

HELD1.

YES. Rules on character evidence provision pertain only to criminal cases, not to
administrative offenses. Even if it is applicable to admin cases, only character evidence
that would establish the probability or improbability of the offense charged may be
proved. Character evidence must be limited to the traits and characteristics involved in
the type of offense charged. In this case, no evidence bearing on Magdalena’s chastity.
What were presented were charges for grave oral defamation, grave threats, unjust
vexation, physical injuries, malicious mischief, etc. filed against her.
Regarding Magdalena’s credibility as a witness, the charges and complaints against her
happened way back in the70s and 80s while the act complained of happened in 1994,
thus, the said charges are no longer reliable proofs of Magdalena’s character or
reputation. Evidence of one’s character or reputation must be confined to a time not too
remote from the time in question. In other words, what is to be determined is the
character or reputation of the person at the time of the trial and prior thereto, but not at
a period remote from the commencement of the suit.
“It is unfair to presume that a person who has wandered from the path of moral
righteousness can never retrace his steps again. Certainly, every person is capable to
change or reform.”
The general rule prevailing in a great majority of jurisdictions is that it is not
permissible to show that a witness has been arrested or that he has been charged with
or prosecuted for a criminal offense, or confined in jail for the purpose of impairing his
credibility. This view has usually been based upon one or more of the following
grounds or theories: (a) that a mere unproven charge against the witness does not
logically tend to affect his credibility, (b)that innocent persons are often arrested or
accused of a crime, (c) that one accused of a crime is presumed to be innocent until his
guilt is legally established, and (d) that a witness may not be impeached or discredited
by evidence of particular acts of misconduct.
G.R. No. 165565 July 14, 2008

SCHOOL OF THE HOLY SPIRIT OF QUEZON CITY and/or SR. CRISPINA A.


TOLENTINO, S.Sp.S., Petitioners,
vs.
CORAZON P. TAGUIAM, Respondent.

QUISUMBING, J.:

Facts:

Corazon P. Taguiam was the Class Adviser of Grade 5-Esmeralda of the petitioner,
School of the Holy Spirit of Quezon City. On March 10, 2000, the class president, wrote
a letter to the grade school principal requesting permission to hold a year-end
celebration at the school grounds. The principal authorized the activity and allowed the
pupils to use the swimming pool. In this connection, respondent distributed the
parent's/guardian's permit forms to the pupils. Corazon P. Taguiam admitted that
Chiara Mae Federico's permit form was unsigned.

Nevertheless, she concluded that Chiara Mae was allowed by her mother to join the
activity since her mother personally brought her to the school with her packed lunch
and swimsuit. Before the activity started, she warned the pupils who did not know how
to swim to avoid the deeper area. However, while the pupils were swimming, two of
them sneaked out. Respondent went after them to verify where they were going.
Unfortunately, while respondent was away, Chiara Mae drowned. When she returned,
the maintenance man was already administering cardiopulmonary resuscitation on
Chiara Mae. The child was still alive when respondent rushed her to the General
Malvar Hospital where she was pronounced dead on arrival. Corazon P. Taguiam was
dismissed for gross negligence resulting to loss of confidence.

Issue:

Was the dismissal based on the ground as stated valid?


Ruling:

Yes the dismissal was valid. Under Article 282 of the Labor Code, gross and habitual
neglect of duties is a valid ground for an employer to terminate an employee. Gross
negligence implies a want or absence of or a failure to exercise slight care or diligence,
or the entire absence of care. It evinces a thoughtless disregard of consequences without
exerting any effort to avoid them. Habitual neglect implies repeated failure to perform
one's duties for a period of time, depending upon the circumstances. Respondent had
been grossly negligent. First , it is undisputed that Chiara Mae's permit form was
unsigned. Yet, respondent allowed her to join the activity because she assumed that
Chiara Mae's mother has allowed her to join it by personally bringing her to the school
with her packed lunch and swimsuit. The purpose of a permit form is precisely to
ensure that the parents have allowed their child to join the school activity involved.
Respondent cannot simply ignore this by resorting to assumptions. Respondent
admitted that she was around when Chiara Mae and her mother arrived. She could
have requested the mother to sign the permit form before she left the school or at least
called her up to obtain her conformity. Second, it was respondent's responsibility as
Class Adviser to supervise her class in all activities sanctioned by the school. Thus, she
should have coordinated with the school to ensure that proper safeguards, such as
adequate first aid and sufficient adult personnel, were present during their activity. She
should have been mindful of the fact that with the number of pupils involved, it would
be impossible for her by herself alone to keep an eye on each one of them. Notably,
respondent's negligence, although gross, was not habitual. In view of the considerable
resultant damage, however, the cause is sufficient to dismiss respondent.

This is not the first time that the SC have departed from the requirements laid down by
the law that neglect of duties must be both gross and habitual. In Philippine Airlines,
Inc. v. NLRC, we ruled that Philippine Airlines (PAL) cannot be legally compelled to
continue with the employment of a person admittedly guilty of gross negligence in the
performance of his duties although it was his first offense. In that case, we noted that a
mere delay on PAL's flight schedule due to aircraft damage entails problems like hotel
accommodations for its passengers, re-booking, the possibility of law suits, and
payment of special landing fees not to mention the soaring costs of replacing aircraft
parts. In another case, Fuentes v. National Labor Relations Commission, we held that it
would be unfair to compel Philippine Banking Corporation to continue employing its
bank teller. In that case, we observed that although the teller's infraction was not
habitual, a substantial amount of money was lost. The deposit slip had already been
validated prior to its loss and the amount reflected thereon is already considered as
current liabilities in the bank's balance sheet. Indeed, the sufficiency of the evidence as
well as the resultant damage to the employer should be considered in the dismissal of
the employee. In this case, the damage went as far as claiming the life of a child. As a
result of gross negligence in the present case, petitioners lost its trust and confidence in
respondent. Loss of trust and confidence to be a valid ground for dismissal must be
based on a willful breach of trust and founded on clearly established facts. A breach is
willful if it is done intentionally, knowingly and purposely, without justifiable excuse,
as distinguished from an act done carelessly, thoughtlessly, heedlessly or inadvertently.
Otherwise stated, it must rest on substantial grounds and not on the employer's
arbitrariness, whims, caprices or suspicion; otherwise, the employee would eternally
remain at the mercy of the employer. It should be genuine and not simulated; nor
should it appear as a mere afterthought to justify earlier action taken in bad faith or a
subterfuge for causes which are improper, illegal or unjustified. It has never been
intended to afford an occasion for abuse because of its subjective nature.
There must, therefore, be an actual breach of duty committed by the employee which
must be established by substantial evidence. All told, there being a clear showing that
respondent was culpable for gross negligence resulting to loss of trust and confidence,
her dismissal was valid and legal.
G.R. No. 82471 February 18, 1991

PHILIPPINE AIRLINES, INC., petitioner,


vs.
NATIONAL LABOR RELATIONS COMMISSION, NATHANIEL PINUELA and
PHILIPPINE AIRLINES EMPLOYEES ASSOCIATION, respondents.

FERNAN, C.J.:p

FACTS:

Private respondent Nathaniel Pinuela was ground equipment and tug operator of
petitioner Philippine Air Lines (PAL), responsible for towing aircraft from the hangar to
the ramp area and vice-versa. In line with his function, he held a professional driver's
license and an operator's permit issued by PAL after he had passed corresponding
theoretical and practical tests to operate PAL vehicles used for towing aircraft. He was
already five (5) years in this particular line of work when the incident in question arose.

On May 31, 1985, Pinuela was assigned as a member of the group tasked to park and
position the Boeing 747 aircraft registered as N-745 at the Manila International Airport
(now Ninoy Aquino International Airport, [NAIA]) for a flight to the United States. The
group members and their respective assignments were: Pinuela –– tow tug operator,
Rolando Manalaysay –– leadman and wingtip guide; Rodrigo Camina-headsetman;
Arturo Balagat –– breakman, and a certain Tañada –– guide. The head of the group in
charge of the parking/positioning of said aircraft was Nolie Domingo.

On or about 12:55 in the afternoon, the aircraft was towed from the PAL technical center
to Bay 16 area at the NAIA. While the Boeing 747 was being towed, the airplane
collided with the bridge at Bay 16 causing damage to the plane's left landing light and
the left wing flop and scratching its No. 2 engine. Consequently, on June 1, 1985,
Pinuela was placed under preventive suspension and was charged administratively.
After investigation by the PAL Administrative Board, he was dismissed from the
service effective July 1, 1985.

Thereafter, Pinuela filed a complaint for illegal dismissal and unfair labor practice
against PAL. On July 21, 1987, the Labor Arbiter ruled in favor of Pinuela and ordered
PAL to reinstate him to his former position without loss of seniority rights with full
backwages until actual reinstatement. 1
On appeal, the National Labor Relations Commission (NLRC) on January 29, 1988
affirmed the decision of the Labor Arbiter but limited the award of backwages to two
(2) years only. 2 The Labor Tribunal opined that "Pinuela could not be blamed for the
accident as he relied on the signal of the headsetman (Camina) who still signaled to him
despite the fact that the nose of the aircraft being towed was about to overshoot the
yellow line and the aircraft wing was about to hit the airbridge." 3

Hence, this recourse, the issue being whether or not the NLRC gravely abused its
discretion in appreciating the facts of the case. Petitioner PAL contends that the records
do not reveal that a tow operator can rely only on a headsetman. According to the
Engineering Manual of petitioner, a tug operator must tipguide positive visual contact
with the wing tipguide when towing aircraft in congested areas. Thus, PAL avers that
Pinuela should have relied on the signal of Manalaysay, then wing tipguide, and not on
Camina, the headsetman.

ISSUE:

Whether or not the acts of Nathaniel Pinuela constitute gross and habitual neglect of
duties

RULING:

One must admit that towing an aircraft is a group activity necessitating group
coordination. This is explicit in petitioner's Engineering and Maintenance Manual
which states, "that the tug operator must undertake and/or continue on
towing/pushing procedure only when positive visual contact with all guidemen is
possible." The use of, "all necessary guidemen" indicates plurality or group
coordination. Thus, instead of relying solely on the signals of Camina, Pinuela should
have also checked with the other ground crew personnel.

Particularly, Pinuela should have relied on Manalaysay, Exhibit 2 distinctly shows the
relative position of the plane, ground crew personnel and the airport's aerobridge when
the incident happened. Manalaysay, who was near the marked line and the nearest
obstruction which was the aerobridge and the parked service vehicle, was strategically
located. For Pinuela to claim that he relied on Camina for signals is not credible, for he
demonstrated before the Labor Arbiter that he had to turn 180 degrees to see Camina
who was directly at his back. 9
In contradiction to the Labor Arbiter's opinion that petitioner's stance is weak because
PAL failed to tipguide normal towing speed, we say that there is no necessity of
determining the exact numerical towing speed. From daily operations, we can safely
assume that the ground crew knows what "creeping" speed means. Through their
working experience, they must certainly have gained a sufficient degree of competence
to determine whether a Boeing 747, weighing tons, was being towed beyond normal
speed or not. Indeed, the Labor Arbiter acted with grave abuse of discretion in ignoring
this vital piece of evidence for the petitioner.

Pinuela's act of towing beyond normal speed, his failure to observe proper parking
procedure as provided in the Engineering and Maintenance Manual, and the
unanimous statement of the members of the towing crew that he completely
disregarded their warning shouts indicate that Pinuela is grossly negligent of his
responsibilities as a tug operator. Pinuela's dismissal must therefore follow for a
company has the right to dismiss its erring employees if only as a measure of self-
protection against acts inimical to its interest. 10 Philippine Airlines, as employer,
cannot be legally compelled to continue with the employment of a person admittedly
guilty of gross negligence in the performance of his duties. 11

Lastly, Pinuela should not compare the penalty of dismissal imposed on him in relation
to lesser sanctions previously meted by PAL on its other employees. We are solely
concerned here with the sufficiency of the evidence surrounding Pinuela's dismissal.
Besides, Pinuela's examples do not involve a plane with a scheduled flight. A mere
delay on petitioner's flight schedule due to aircraft damage entails problems like hotel
accommodations for its passengers, re-booking, the possibility of law suits, and
payment of special landing fees not to mention the soaring costs of replacing aircraft.
parts. All told, Pinuela's gross negligence which called for dismissal is evident and it
was grave abuse of discretion on the part of the labor tribunal to have ruled otherwise.

G.R. No. 75955 October 28, 1988

MARIA LINDA FUENTES, petitioner,


vs.
NATIONAL LABOR RELATIONS COMMISSION (NLRC), PHILIPPINE BANKING
CORPORATION and JOSE LAUREL IV, as its President, respondents.
FERNAN, C.J.:

FACTS:

Petitioner was employed as a teller at the Philbanking's office at Ayala Avenue, Makati,
Metro Manila. On May 28, 1982, at about 10:30 a.m., petitioner, who was acting as an
overnight teller, received a cash deposit of P200,000.00. She counted the money with the
assistance of a co-teller, finishing the task at 10:40 a.m. or ten (10) minutes after her
closing time. Before she could start balancing her transactions, the Chief Teller handed
her several payroll checks for validation. Finding the checks to be incomplete, petitioner
left her cage to get other checks, without, however, bothering to put the P200,000.00
cash on her counter inside her drawer. When she returned to her cubicle after three (3)
to five (5) minutes, she found that the checks for validation were still lacking, so she
went out of her cubicle again to get the rest of the checks. On her way to a co-teller's
cubicle, she noticed that the P200,000.00 pile on her counter had been re-arranged. She
thus returned to her cage, counted the money and discovered that one (1) big bundle
worth P50,000.00 was missing therefrom. She immediately asked her co-teller about it
and getting a negative reply, she reported the matter to the Chief Teller. A search for the
P50,000.00 having proved unavailing, petitioner was asked to explain why she should
not be held liable for the loss. She submitted her explanation on June 24, 1982.

Subsequently, on June 3, 1983, petitioner was dismissed for gross negligence. On June
21, 1983, she filed a complaint for illegal dismissal with reinstatement and backwages.

Private respondent bank seasonably filed an answer with counterclaim that petitioner
be ordered to restitute the amount of P50,000.

ISSUE:

Whether or not the dismissal of Fuentes on the ground of gross and habitual neglect of
duties valid?

RULING:

In the case at bar, the bank's inaction merely created a condition under which the loss
was sustained. Regardless of whether there was a failure to investigate, the fact is that
the money was lost in the first place due to petitioner's gross negligence. Such gross
negligence was the immediate and determining factor in the loss.

Besides, the petitioner's position is anathema to banking operations. By conducting an


instant search on its depositors for every loss that occurs, management holds suspect
each depositor within its premises. Considering that currency in the form of money bills
bears no distinct earmarks which would distinguish it from other similar bills of similar
denominations except as to its serial numbers, any innocent depositor with P50,000 in
his possession would be a likely suspect. Such act would do violence to the fiduciary
relationship between a bank and its depositors. Ultimately it will result in the loss of
valued depositors.

Petitioner argues further that the NLRC failed to consider that petitioner left her cage at
the instance of the Chief Teller. Again we are not persuaded. The findings of the NLRC
are clear. Petitioner left at her own volition to approach her Chief Teller to ask for the
remaining checks to ascertain their authenticity and completeness. Besides, irrespective
of who summoned her, her responsibility over the cash entrusted to her remained.

Although petitioner's infraction was not habitual, we took into account the substantial
amount lost. Since the deposit slip for P200,000.00 had already been validated prior to
the loss, the act of depositing had already been complete and from thereon, the bank
had already assumed the deposit as a liability to its depositors. Cash deposits are not
assets to banks but are recognized as current liabilities in its balance sheet.

G.R. No. 139847 March 5, 2004

PROCTER AND GAMBLE PHILIPPINES, petitioner,


vs.
EDGARDO BONDESTO, respondent.
TINGA, J.:
FACTS:

On July 18, 1975, respondent Edgardo Bondesto started work in the employ of
petitioner Procter and Gamble Philippines, Inc. Nineteen (19) years later, the events
which preceded the respondent’s dismissal from work unfolded. At that time, he was
working as production technician at the company’s Tondo Plant in Tondo, Manila.

On June 13, 1994, the respondent received a letter2 dated June 3, 1994, asking him to
explain why his absences consisting of 35 days3 should not be classified as
"unauthorized absence." Unauthorized absence, as a company policy, is a ground for
termination of employment.4

The respondent presented his explanation in two (2) separate letters5, both dated June
16, 1994. However, on June 22, 1994, he received another letter, this time informing him
that his employment in the company was to be terminated effective June 23, 1994 on the
ground of "unauthorized absences."

Claiming that his dismissal was without just cause, the respondent, represented by the
United Employees Union of Procter and Gamble Phils., Inc., filed a complaint for illegal
dismissal before the National Labor Relations Commission (NLRC). The respondent
contended that his absences were justified.

Sometime in November 1993, the respondent alleged, the petitioner directed him to go
to Mindanao for field assignment. Except for the plane fare which the petitioner paid
prior to his departure, the respondent advanced all the other work-related expenses
incurred during the assignment. One of the petitioner’s Staff District Managers issued a
check in the amount of Ten Thousand Pesos (P10,000.00) supposedly to cover
respondent’s traveling expenses, but it bounced after he presented it to the bank.

On January 31, 1994, the respondent was re-assigned in Manila. He immediately


worked on the reimbursement of his advances. But as the reimbursements were not
immediately released, he was constrained to go to the petitioner’s General Office
located in Makati to follow-up the reimbursement.

Meanwhile, the children of the respondent became sick. He spent time attending to
them. And as he needed money, he also went to the company’s Makati office to follow-
up the reimbursement process. The delay in the release of his reimbursement even
forced him to apply for wage advances under the collective bargaining agreement
between the company and the union.

On April 6, 1994, or after more than two months, the petitioner finally released the
respondent’s reimbursements.

One week later, or on April 13, 1994, the respondent received a letter asking him to
explain his "excessive absences"7, which according to him, included the days he worked
on his reimbursements. The respondent demurred. He claimed that the seventeen (17)
days should be considered as compensable working time since he was then at the
Makati office working on the reimbursement of his money.

ISSUE:

Whether or not habitual absences constitute gross and habitual neglect of duties

RULING

The Court agrees, however, with the petitioner that the respondent failed to justify his
prolonged absences during the months of May to June. While his intention to go back to
work was manifest, he regrettably failed to show that he exerted any effort to locate his
physician. Nevertheless, the failure to locate the physician cannot amount to "serious
misconduct or willful disobedience," as the petitioner would like this Court to believe.
"Misconduct" has been defined as "the transgression of some established and definite
rule of action, a forbidden act, a dereliction of duty, willful in character, and implies
wrongful intent and not mere error in judgment."22 On the other hand, "willful
disobedience" envisages the concurrence of at least two (2) requisites: the employee’s
assailed conduct has been willful or intentional, the willfulness being characterized by a
"wrongful and perverse attitude;" and the order violated must have been reasonable,
lawful, made known to the employee and must pertain to the duties which he had been
engaged to discharge.

Even assuming that the respondent’s absenteeism constitutes willful disobedience, such
offense does not warrant the respondent’s dismissal. Not every case of insubordination
or willful disobedience by an employee reasonably deserves the penalty of dismissal.
There must be a reasonable proportionality between the offense and the penalty.

At the time of the filing of the complaint, the respondent had worked with the
petitioner for nineteen (19) years. It has not been shown that the respondent committed
any infraction of company rules during his two (2) – decade stint in the company.
Undoubtedly, dismissal is too harsh a sanction. Dismissal has always been regarded as
the ultimate penalty.
While the Court recognizes the rights of an employer to terminate the services of an
employee for a just or authorized cause, the dismissal of an employee must be made
within the parameters of law and pursuant to the tenets of equity and fair play. Truly,
the employer’s power to discipline its workers may not be exercised in such an arbitrary
manner as to erode the constitutional guarantee of security of tenure.The Constitution
mandates the protection of labor. This command the Court has to heed and cannot
disregard.

In sum, the Court is convinced that the respondent has been illegally terminated from
employment. The normal consequences of illegal dismissal are reinstatement without
loss of seniority rights and the payment of back wages computed from the time the
employee’s compensation was withheld from him. However, in view of the Court’s
finding that some of the respondent’s absences were not wholly justified, the Court
agrees with the NLRC and the Court of Appeals that backwages should be limited to
one (1) year.

The petitioner claims that the existence of strained relationship between the parties
militates against the reinstatement of the respondent. While the Court agrees that
human nature engenders, in the normal course of things, a certain degree of hostility as
a result of litigation, the strained relations are not necessarily sufficient to rule out
reinstatement. As aptly put by the Court of Appeals, "if petitioner’s contention should
be sustained, reinstatement would thus become the exception rather than the rule in
cases of illegal dismissal."

[G.R. No. 153477 : July 04, 2007]

DEL MONTE PHILIPPINES, INC. V. LOLITA VELASCO

Facts:

Respondent was a regular employee of petitioner, Del Monte Philippines. On 17


September 1994, a notice of hearing was sent to respondent notifying her of the charges
filed against her by the company, more specifically, for violating the Absence Without
Official Leave Rule. She was subsequently dismissed. The respondent filed a case for
illegal dismissal, asserting that her dismissal was illegal because she was suffering from
a urinary tract infection related to her pregnancy and that her doctor had advised her to
stay home for a few days. She declared that she did not file the adequate leave of
absence because a medical certificate was already sufficient per company policy. The
Labor Arbiter dismissed the complaint. On appeal, both the NLRC and the CA declared
the dismissal of complainant as illegal for being contrary to Art. 137 (2) of the Labor
Code, prohibiting the discharge of a woman on account of her pregnancy.

ISSUE:

Whether the absences of the respondent constitute gross and habitual neglect of duties

Held:

The Court affirmed the ruling of the Court of Appeals. Respondent’s rule penalizing
with discharge any employee incurring six (6) or more absences without permission or
subsequent justification is admittedly within the purview of the labor standard.
However, while it is not disputed that complainant incurred absences exceeding 6 days,
respondent was able to subsequently justify her absences in accordance with company
rules and policy: that she was pregnant at the time she incurred the absences; that the
fact of pregnancy and its related illnesses had been duly proven through substantial
evidence; that the respondent attempted to file leaves of absence but the petitioner’s
supervisor refused to received them; that she could not have filed prior leaves due to
her continuing condition, and that petitioner dismissed the respondent on account of
her pregnancy, a prohibited act. Art. 137 of the Labor Code provides: “It shall be
unlawful for any employer: x x x (2.) To discharge such woman on account of her
pregnancy, while on leave or in confinement due to her pregnancy; x x x.”
WPP Marketing Communications Inc., et al., vs. Galera,
G.R. No. 169207, March 25, 2010

Facts:

Petitioner is Jocelyn Galera, an American citizen who was recruited from the US by
private respondent John Steedman, Chairman-WPP Worldwide and Chief Executive
Officer of Mindshare, Co., a corporation based in Hong Kong, China, to work in the
Philippines for private respondent WPP Marketing Communications, Inc. (WPP). On
December 14, 2000, GALERA alleged she was verbally notified by private
STEEDMAN that her services had been terminated from private respondent WPP.
Atermination letter followed the next day. On 3 January 2001, Galera filed a complaint
for illegal dismissal, holiday pay, service incentive leave pay, 13th month pay, incentive
plan, actual and moral damages, and attorney's fees against WPP and/or John
Steedman (Steedman), Mark Webster (Webster) and Nominada Lansang (Lansang). The
Labor Arbiter's Ruling for illegal dismissal and damages in favor of GALERA. The First
Division of the NLRC reversed the ruling of Arbiter Madriaga. Yet it was reversed
again by CA.

Issue:

Whether Galera is an Employee or a Corporate Officer.


Whether WPP illegally dismissed Galera.

Ruling:

Employee. Galera, on the belief that she is an employee, filed her complaint before the
Labor Arbiter. On the other hand, WPP, Steedman, Webster and Lansang contend that
Galera is a corporate officer; hence, any controversy regarding her dismissal is under
the jurisdiction of the Regional Trial Court. We agree with Galera. Corporate officers are
given such character either by the Corporation Code or by the corporation's by-laws.
Galera's appointment as a corporate officer (Vice-President with the operational title of
Managing Director of Mindshare) during a special meeting of WPP's Board of Directors
is an appointment to a non-existent corporate office. At the time of Galera's
appointment, WPP already had one Vice-President in the person of Webster and all five
directorship positions provided in the by-laws are already occupied. Another
indicator that she was a regular employee and not a corporate officer is Section 14 of the
contract, which clearly states that she is a permanent employee — not a Vice-President
or a member of the Board of Directors. Another convincing indication that she was only
a regular employee and not a corporate officer is the disciplinary procedure, which
states that her right of redress is through Mindshare's Chief Executive Officer for the
Asia-Pacific. This implies that she was not under the disciplinary control of private
respondent WPP's Board of Directors (BOD), which should have been the case if in fact
she was a corporate officer because only the Board of Directors could appoint and
terminate such a corporate officer.

WPP's dismissal of Galera lacked both substantive and procedural due process. Apart
from Steedman's letter dated 15 December 2000 to Galera, WPP failed to prove any just
or authorized cause for Galera's dismissal. Steedman's letter to Galera reads: The
operations are currently in a shamble. There is lack of leadership and confidence in
your abilities from within, our agency partners and some clients. Most of the staff I
spoke with felt they got more guidance and direction from Minda than yourself. In your
role as Managing Director, that is just not acceptable. I believe your priorities are
mismanaged. The recent situation where you felt an internal strategy meeting was more
important than a new business pitch is a good example. You failed to lead and advise
on the two new business pitches. In both cases, those involved sort (sic) Minda's input.
As I discussed with you back in July, my directive was for you to lead and review all
business pitches. It is obvious [that] confusion existed internally right up until the day
of the pitch. The quality output is still not to an acceptable standard, which was also
part of my directive that you needed to focus on back in July. I do not believe you
understand the basic skills and industry knowledge required to run a media special
operation.

WPP, Steedman, Webster, and Lansang, however, failed to substantiate the allegations
in Steedman's letter. Galera, on the other hand, presented documentary evidence 22 in
the form of congratulatory letters, including one from Steedman, which contents are
diametrically opposed to the 15 December 2000 letter. The law further requires that the
employer must furnish the worker sought to be dismissed with two written notices
before termination of employment can be legally effected: (1) notice which apprises the
employee of the particular acts or omissions for which his dismissal is sought; and (2)
the subsequent notice which informs the employee of the employer's decision to dismiss
him. Failure to comply with the requirements taints the dismissal with illegality. 23
WPP's acts clearly show that Galera's dismissal did not comply with the two-notice rule.

ARSENIO S. QUIAMBAO, vs.MANILA ELECTRIC COMPANY


G.R. No. 171023 December 18, 2009

FACTS:
Petitioner was employed as branch teller by respondent Manila Electric Company-
Mandaluyong Office for the handling and processing of payments.

Petitioner has repeatedly violated the Company Code of Employee Discipline


specifically excessive unauthorized absences. Through a Notice of Dismissal,
petitioner’s employment was terminated effective March 29, 2000.
On July 3, 2001, petitioner filed a complaint before the Arbitration Branch of the NLRC
against respondent assailing the legality of his dismissal. He also claimed that he was
denied due process. The Labor Arbiter dismissed the complaint for a lack of merit.

Petitioner appealed to the NLRC which affirmed the legality of his dismissal due to
habitual absenteeism.

On appeal to the Court of Appeals, the CA nullified the NLRC’s Decision and reinstated
the Labor Arbiter’s Decision dismissing the complaint. It ruled that the award of
separation pay is neither justified nor warranted under the circumstances

The Motion for Reconsideration was denied, hence this petition for review on certiorari.

ISSUE:
Whether or not a validly dismissed employee may be entitled to separation pay.

RULING:
The Supreme Court DENIED petition for lack of merit.

The Labor Arbiter, the NLRC and the Court of Appeals found that petitioner’s
unauthorized absences and repeated infractions of company rules on employee
discipline manifest gross and habitual neglect of duty that merited the imposition of the
supreme penalty of dismissal from work. Serious misconduct is said to be a
transgression of some established and definite rule of action, a forbidden act, a
dereliction of duty, willful in character, and indicative of wrongful intent and not mere
error of judgment. Oddly, petitioner never advanced any valid reason to justify his
absences.

Following jurisprudence, it is held that a series of irregularities when put together may
constitute serious misconduct. Hence, the petitioner is not entitled to separation pay.
The liberality of the law can never be extended to the unworthy and undeserving.
G.R. No. 112206 December 11, 1995

GONZALO D. LABUDAHON, KAPATIRANG ANAK-PAWIS SA PIONEER


TEXTURIZING, petitioners,

vs.

NATIONAL LABOR RELATIONS COMMISSION, PIONEER TEXTURIZING


CORPORATION, and MR. JULIANO LIM, respondents.

PADILLA, J.:

FACTS:
On 27 April 1990, petitioner applied for a 13-day paternity leave, but respondent
company allowed him only five (5) days of leave effective on 30 April 1990 and until 5
May 1990 because of lack of manpower at the Texturizing Preparatory Section. From 7
May 1990 to 12 May 1990, petitioner absented himself from work without respondent
company's approval.

On 11 May 1990, Venus Dy, the personnel manager of respondent company, wrote
petitioner a letter directing him to report to work lest his absences be considered as
abandonment of duty.

On 12 May 1990, petitioner asked for two (2) weeks of vacation leave from 7 May to 20
May 1990, but the same was disapproved except for two (2) days of leave on 14 and 15
of May 1990 to enable petitioner to attend to family problems. Petitioner did not report
for work from 16 to 19 of May 1990. Upon orders of respondent company, petitioner
submitted a written explanation citing his wife's childbirth and family problems as
reasons for his absences. Petitioner was meted five (5) days suspension for unexcused
absences and for insubordination.

In spite of his previous absences, petitioner, as union officer, asked for fifty-four (54)
days of leave from 9 July 1990 to 31 August 1990 to prepare for CBA negotiations and
union activities. The request was denied and instead the management advised
petitioner to file his leave on a weekly basis, as approval thereof was contingent on the
necessity of his presence in the operations of the Texturizing Department of respondent
company. Petitioner completely ignored this directive and absented himself from work
starting 21 July 1990 until 16 August 1990. In a memorandum of the personnel
department dated 3 August 1990, petitioner was asked to submit a written explanation
for his absences. Respondent company never received any letter of explanation. In a
memorandum dated 28 August 1990, the company, through its personnel manager,
terminated petitioner's services to take effect on 29 August 1990 "for excessive absences,
insubordination, and violation of existing company rules and regulations".

ISSUE:

Whether or not the excessive absences of the petitioner a valid ground for his
termination.

RULING:
The NLRC found that petitioner had no regard for his work. His applications for a
series of leaves of absence attest to his unconcern for his duties in respondent company.
On the other hand, respondent company has to protect its interests in order to have an
efficient and productive enterprise. It is in this light that the law recognizes what are
clearly "management prerogatives", or the right of the employer to hire, fire, transfer,
demote or promote employees. Doubtless, what respondent did in this case was a
management prerogative. The need for petitioner's presence in the company's
Texturizing Department cannot be denied. Therefore, the continuous and unauthorized
absences of petitioner adversely affected the operations of respondent company. The
petitioner left the company with no other choice but to terminate his employment.

The NLRC decision to indemnify petitioner is also affirmed, as there is no evidence in


the records to show that respondent company observed the two-notice requirement and
hearing before dismissing petitioner. Applying the Omnibus Rules Implementing the
Labor Code 4 on the requirements of notice and hearing, this Court in the case of Tiu vs.
NLRC 5 ruled:

It is evident from the said provisions that the employer is required to furnish an
employee who is to be dismissed two (2) written notices before such termination. The
first is the notice to apprise the employee of the particular acts or omissions for which
his dismissal is sought. This may be loosely considered as the proper charge. The
second is the notice informing the employee of the employer's decision to dismiss him.
This decision, however, must come only after the employee is given a reasonable period
from receipt of the first notice within which to answer the charge, and ample
opportunity to be heard and defend himself with the assistance of his representative, if
he so desires.

In the case at bar, petitioner was given only a letter of dismissal without earlier
informing him of the charges against him and without giving him the opportunity to
defend himself. Non-compliance by private respondent with these requirements is a
violation of the petitioner's right to due process.
BREW MASTER INTERNATIONAL INC. V NATIONAL FEDERATION OF LABOR
UNIONS (NAFLU)
DAVIDE, JR; April 17, 1997

FACTS
- Private respondent NAFLU, a co-complainant in the labor case, is a labor union of
which complainant is amember.- Complainant was first employed by Brew Master on
16 September 1991 as route helper with the latest daily wageof P119.00.- From 19 April
1993 up to 19 May 1993, for a period of 1 month, complainant went on absent without
permission(AWOP).- On 20 May 1993, Brew master sent him a Memo: “Please explain
in writing within 24 hours of your receipt of thismemo why no disciplinary action
should be taken against you for the following offense: You were absent sinceApril 19,
1993 up to May 19, 1993.”- In answer to the aforesaid memo, complainant explained:“Sa
dahilan po na ako ay hindi nakapagpaalam sainyo dahil inuwi ko ang mga anak ko sa
Samar dahil ang asawa koay lumayas at walang mag-aalaga sa mga anak ko. Kaya
naman hindi ako naka long distance or telegrama dahilwala akong pera at ibinili ko ng
gamot ay puro utang pa.”- Finding said explanation unsatisfactory, the company issued
a Notice of Termination: “...we regret to inform youthat we do not consider it valid. You
are aware of the company Rules and Regulations that absence without permission for 6
consecutive working days is considered abandonment of work...”- Complainants
contend that individual complainant’s dismissal was done without just cause; that it
was notsufficiently established that individual complainant’s absence from April 19,
1993 to June 16, 1993 are unjustified;that the penalty of dismissal for such violation is
too severe; that in imposing such penalty, respondent should havetaken into
consideration complainant’s length of service and as a first offender, a penalty less
punitive will sufficesuch as suspension for a definite period.- Upon the other hand,
respondent contends that individual complainant was dismissed for cause allowed by
thecompany Rules and Regulations and the Labor Code; that the act of complainant in
absenting from work for 1month without official leave is deleterious to the business of
respondent; that it will result to stoppage of productionwhich will not only destructive
to respondent’s interests but also to the interest of its employees in general; that
thedismissal of complainant from the service is legal.- The Labor Arbiter dismissed the
complaint for lack of merit, citing the principle of managerial control, whichrecognizes
the employer’s prerogative to prescribe reasonable rules and regulations to govern the
conduct of hisemployees. He relied on
Shoemart, Inc. vs. NLRC
: “...that individual complainant has indeed abandoned his work...therefore, under the
law and jurisprudence which upholds the right of an employer to discharge an
employee whoincurs frequent, prolonged and unexplained absences as being grossly
remiss in his duties to the employer and istherefore, dismissed for cause. An employee
is deemed to have abandoned his position or to have resigned from thesame, whenever
he has been absent therefrom without previous permission of the employer for three
consecutivedays or more. “- the NLRC modified the Labor Arbiter's decision and held
that complainant’s dismissal was invalid for thefollowing reasons:Complainant-
appellant’s prolonged absences, although unauthorized, may not amount to gross
neglect or abandonment of work to warrant outright termination of employment.
Dismissal is too severe a penalty...Relianceon the ruling enunciated in the cited case of
Shoemart is quite misplaced because of the obvious dissimilarities--complainant in the
Shoemart Case was “an inveterate absentee who does not deserve reinstatement”
compared toherein complainant-appellant who is a first offender

ISSUE

WON the NLRC committed grave abuse of discretion in modifying the decision of the
Labor Arbiter
HELD

NO. Petitioner’s finding that complainant was guilty of abandonment is misplaced.


Abandonment as a just andvalid ground for dismissal requires the deliberate,
unjustified refusal of the employee to resume his employment.Two elements must then
be satisfied: (1) the failure to report for work or absence without valid or justifiable
reason;and (2) a clear intention to sever the employer-employee relationship. b) Verily,
relations between capital and labor are not merely contractual. They are impressed with
public interestand labor contracts must, perforce, yield to the common good.While the
employer is not precluded from prescribing rules and regulations to govern the conduct
of his employees,these rules and their implementation must be fair, just and reasonable.
Reasoning
- complainant’s absence was precipitated by a grave family problem as his wife
unexpectedly deserted him andabandoned the family. Considering that he had a full-
time job, there was no one to whom he could entrust thechildren and he was thus
compelled to bring them to the province. He was then under emotional,
psychological,spiritual and physical stress and strain. The reason for his absence is,
under these circumstances, justified. Whilehis failure to inform and seek petitioner's
approval was an omission which must be corrected and chastised, he didnot merit the
severest penalty of dismissal from the service.- the elements of abandonment are not
present here. First, as held above, complainant's absence was justified under the
circumstances. As to the second requisite, complainant immediately complied with the
memo requiring him toexplain his absence, and upon knowledge of his termination,
immediately sued for illegal dismissal. These plainlyrefuted any claim that he was no
longer interested in returning to work.- our Constitution looks with compassion on the
workingman and protects his rights not only under a generalstatement of a state policy,
but under the Article on Social Justice and Human Rights, thus placing labor contracts
ona higher plane and with greater safeguards.- While we do not decide here the validity
of petitioner's Rules and Regulations on continuous, unauthorizedabsences, what is
plain is that it was wielded with undue haste resulting in a deprivation of due process,
thus notallowing for a determination of just cause or abandonment. In this light,
petitioner's dismissal was illegal. This isnot to say that his absence should go
unpunished, as impliedly noted by the NLRC in declining to award back wages.
G.R. No. 168931 September 12, 2006
PAULINO ALITEN,
Petitioner,- versus -
U-NEED LUMBER & HARDWARE, and COURT OF APPEALS, Respondents.

FACTS:

Petitioner, a native of Calanasan, Kalinga-Apayao, was employed on January 3, 1988[2]


as helper in U-Need, private respondent’s business in Baguio City. On October 22, 1990,
petitioner was promoted as regular driver. Petitioner was registered as a member of the
Social Security System (SSS).

On April 30, 1992, petitioner asked permission from U-Need Manager Virginia Tan for
a 15-day leave of absence. He planned to go to his hometown to visit his parents and
vote for his uncle Elias Balot who was running for congressman in the May 11, 1992
national and local elections. He needed a 15-day leave because his hometown was a
remote area; and considering the mountains and rivers he had to cross, travel could be
very difficult and could take roughly 12 days.[3] Petitioner signed a typewritten
application for a one-week vacation leave starting May 4, 1992,[4] a Monday, wherein
he declared that should he fail to report back for work at the end of his vacation, it is
understood that he would be automatically terminated by his employer. He signed the
typewritten application without reading its contents because he was being scolded by
Virginia Tan, who likewise forced him to sign.
Thereafter, he took some of his belongings from the lumber’s bunkhouse and left with
his brother using a government vehicle.[5] When he left U-Need on May 4, 1992, he
had an outstanding account. However, it was not in the sum of P1,750.00 as alleged by
private respondent because as far as he knew, amounts had been automatically
deducted from his wages.[6]

It turned out that, as gleaned from the records of the Election Registrar, Baguio City, he
was a registered voter of Middle Rock Quarry, Baguio City, not Calanasan.[7] He
claimed that he only registered therein so that he could cast his vote in Baguio should
he fail to return to his domicile.[8] When he reported for work on May 14, 1992, he was
told that he had already been dismissed from employment.

[G.R. No. 149180. February 14, 2005]

HODIENG CONCRETE PRODUCTS and/or HENRY GO and ERIC B. GO,


petitioners, vs. DANTE EMILIA, respondent.
SANDOVAL-GUTIERREZ, J.:

FACTS:

Respondent, in his complaint, alleged that sometime in January 1985, he was employed
as a truck helper by petitioner with a daily salary of P40.00. Eventually, he became a
regular truck driver with a salary of P76.00 per trip. Feeling that he was underpaid, he
asked petitioners’ secretary if he is entitled to 13th month pay and other benefits.
Upon receiving a negative response, he reported the matter to the Department of Labor
and Employment. This prompted petitioners to terminate his services effective
January 2, 1997.

Petitioners denied respondent’s allegations in his complaint. They claimed that


sometime in 1987, respondent was employed as a truck helper with a salary of P40.00
per trip. Later, in 1991, he recklessly drove and damaged petitioner’s truck. As a
consequence, he failed to report for work. But, in 1995, he was re-hired as a truck driver
with a salary of P76.00 per trip. In 1997, he abandoned his work. Petitioners then came
to know that he was employed by Vortex, another company.

On March 30, 1998, the Labor Arbiter rendered a Decision holding that respondent was
illegally dismissed from employment and ordering petitioners, jointly and severally, to
pay his backwages and other benefits, separation pay, and attorney’s fee equivalent to
10% of the monetary awards,

ISSUE:

Whether or not the dismissal was valid?

RULING:

“On the alleged abandonment of employment by private respondent, the contention is


not meritorious. x x x. The burden of proof is on the employer to show unequivocal
intent on the part of the employee to discontinue employment. Other than the self-
serving declarations in the affidavits of their two witnesses (pp. 59-61, ibid), petitioners
failed to adduce other evidence on any overt act of private respondent showing an
actual intent to abandon his employment. Moreover, private respondent filed an illegal
dismissal case against petitioners, an act which negates any intention on the part of the
employee to forsake his work. x x x.

Considering our finding that private respondent is a regular employee of petitioners,


and considering that there was no legal cause for private respondent’s termination from
employment, We so hold that petitioners violated the constitutional right of private
respondent to security of tenure and due process. As a consequence of the illegal
dismissal of the private respondent, he is entitled under Articles 279 of the Labor Code,
to his full backwages, without deduction of earnings derived elsewhere from the time
his compensation was withheld from him up to the time of his actual reinstatement. By
virtue however of private respondent’s refusal to be reinstated, the harmonious
relationship between him and petitioners had been severed such that reinstatement
would no longer be beneficial to either party in which case, a separation pay equivalent
to one (1) month pay for every year of service be paid. For failure of petitioners to
submit the corresponding employment records such as the payroll to controvert the
private respondent’s claim for service incentive leave pay and holiday pay, We find the
award for the said claim proper.
[G.R. No. 133259. February 10, 2000]

WENIFREDO FARROL, petitioner, vs. The HONORABLE COURT OF APPEALS and


RADIO COMUNICATIONS of the PHILIPPINES INC. (RCPI), respondents.
YNARES_SANTIAGO, J.:

FACTS:

Petitioner Wenifredo Farrol was employed as station cashier at respondent RCPI’s


Cotabato City station. On June 18, 1993, respondent RCPI’s district manager in Cotabato
City informed their main office that "Peragram funds"[1] from said branch were used
for the payment of retirement benefits of five employees. On October 1, 1993, petitioner
verified as correct RCPI’s Field Auditor’s report that there was a shortage of P50,985.37
in their branch’s Peragram, Petty and General Cash Funds. Consequently, petitioner
was required by the Field Auditor to explain the cash shortage within 24 hours from
notice.[2] The next day, petitioner paid to RCPI P25,000.00 of the cash shortage.

On October 16, 1993, RCPI required petitioner to explain why he should not be
dismissed from employment.[3] Two days thereafter, petitioner wrote a letter to the
Field Auditor stating that the missing funds were used for the payment of the
retirement benefits earlier referred to by the branch manager and that he had already
paid P25,000.00 to RCPI. After making two more payments of the cash shortage to
RCPI, petitioner was informed by the district manager that he is being placed under
preventive suspension.[4] Thereafter, he again paid two more sums on different dates to
RCPI leaving a balance of P6,995.37 of the shortage.
Respondent RCPI claims that it sent a letter to petitioner on November 22, 1993
informing him of the termination of his services as of November 20, 1993 due to the
following reasons:

"a) Your allegation that part of your cash shortages was used for payment of
salaries/wages and retirement benefits is not true because these have been accounted
previously per auditor’s report;

"b) As Station Cashier you must be aware of our company Circular No. 63 which strictly
requires the daily and up-to-date preparation of Statistical Report and depositing of
cash collections twice a day. But these procedures - more particularly on depositing of
cash collections twice a day - was completely disregarded by you;

"c) Deliberate withholding of collections to hide shortages/malversation or


misappropriation in any form, as emphasized under Section No. 20 of our Rules and
Regulations, is penalized by immediate dismissal;

"d) The position of Station Cashier is one which requires utmost trust and confidence.[5]

Unaware of the termination letter, petitioner requested that he be reinstated considering


that the period of his preventive suspension had expired.

Sometime in September 1995, petitioner manifested to RCPI his willingness to settle his
case provided he is given his retirement benefits. However, RCPI informed petitioner
that his employment had already been terminated earlier as contained in the letter
dated November 22, 1993. The conflict was submitted to the grievance committee.
Despite the lapse of more than two years, the case remained unresolved before the
grievance committee, hence, it was submitted for voluntary arbitration.

ISSUE:

Whether or not the acts committed by Farrol constituted fraud or willful breach of trust

RULING:

A perusal of RCPI’s dismissal notice reveals that it merely stated a conclusion to the
effect that the withholding was deliberately done to hide alleged malversation or
misappropriation without, however, stating the facts and circumstances in support
thereof. It further mentioned that the position of cashier requires utmost trust and
confidence but failed to allege the breach of trust on the part of petitioner and how the
alleged breach was committed. On the assumption that there was indeed a breach, there
is no evidence that petitioner was a managerial employee of respondent RCPI. It should
be noted that the term "trust and confidence" is restricted to managerial employees.[15]
It may not even be presumed that when there is a shortage, there is also a
corresponding breach of trust. Cash shortages in a cashier’s work may happen, and
when there is no proof that the same was deliberately done for a fraudulent or wrongful
purpose, it cannot constitute breach of trust so as to render the dismissal from work
invalid.

Assuming further that there was breach of trust and confidence, it appears that this is
the first infraction committed by petitioner. Although the employer has the prerogative
to discipline or dismiss its employee, such prerogative cannot be exercised wantonly,
but must be controlled by substantive due process and tempered by the fundamental
policy of protection to labor enshrined in the Constitution.[16] Infractions committed by
an employee should merit only the corresponding sanction demanded by the
circumstances. The penalty must be commensurate with the act, conduct or omission
imputed to the employee[17] and imposed in connection with the employer’s
disciplinary authority.

RCPI alleged that under its rules, petitioner’s infraction is punishable by dismissal.
However, employer’s rules cannot preclude the State from inquiring whether the strict
and rigid application or interpretation thereof would be harsh to the employee.
Petitioner has no previous record in his twenty-four long years of service - this would
have been his first offense. The Court thus holds that the dismissal imposed on
petitioner is unduly harsh and grossly disproportionate to the infraction which led to
the termination of his services. A lighter penalty would have been more just, if not
humane. In any case, petitioner paid back the cash shortage in his accounts.
Considering, however, that the latter is about to retire or may have retired from work, it
would no longer be practical to order his reinstatement.
[G.R. No. 158758. April 29, 2005]

P.J. LHUILLIER INC. and PHILIPPE J. LHUILLIER, petitioners, vs. NATIONAL


LABOR RELATIONS COMMISSION and HERMINIA MONTENEGRO,
respondents.
CHICO-NAZARIO, J.:

FACTS:

Vincent “Vicente” Montenegro was the appraiser/manager of the petitioner


corporation’s Bauan Branch in Batangas. Lailanie Palma, a trainee of the company,
charged him with sexual harassment. A committee was formed to investigate him for
the alleged sexual harassment. On the basis of the formal investigation conducted, the
Chairman of the Investigation Committee issued a Notice of Disciplinary Action dated
06 September 1997 wherein said employee was meted a ten (10)-day suspension and
transfer of assignment to the CLH-Zobel Branch, Makati City, effective the next
working day from receipt thereof for violation of Section 9 of the Handbook on
Company Policies and Guidelines and Employee’s Code of Conduct, with a warning
that a repetition of said violation will be penalized with the supreme sanction of
dismissal. Vincent Montenegro claims that for the sexual harassment case, he was
meted 35 days suspension which he contends is a violation of the 30-day suspension.
Thereafter, he was transferred to Makati.

Herminia Montenegro was charged with dishonest acts committed by causing the
redemption of two (2) pieces of jewelry specifically described in pawn tickets 008664
and 008665, allegedly, through the use of falsified affidavit of loss. A formal
administrative investigation was conducted on 15 October 1997. Findings of said
investigation showed that respondent Herminia Montenegro committed dishonesty and
misconduct violative of Rule 22, Section 2 of the Handbook on Company Policies,
hence, she was dismissed from employment. Herminia Montenegro averred, however,
that her only participation was the approval of the redemption of the pawned items by
a certain Agnes Moradas who submitted an affidavit of loss of pawnshop tickets.

Carlos Pedro Sara was charged with incompetence and dishonesty. During the
administrative investigation conducted on 05 December 1997, the investigating
committee reported that Sara admitted having intentionally overweighed an item in
favor of a customer but the report about which he refused to sign. It was also
discovered that Sara was directly responsible for the loss of certain jewelry as disclosed
in an audit report.

Marites Noble was charged with having involved in the over-appraisal of an item and
having accepted a gold plated item. She claims that she had to accept the over-
appraised item to attract customers as the branch has just opened. As for the fake item
she accepted, Noble avers that the item is so thickly plated that it could not be detected
by merely applying the usual procedure. During the formal investigation conducted on
05 December 1997, it was discovered and admitted by Noble that she intentionally over-
appraised the subject pawned fake item by increasing their true weights. Later, it
turned out that the fake items belong to Noble herself.

ISSUE:

Whether or not the termination of the respondents were with just cause?

RULING:

At the onset, it is pertinent to note that the second issue raised in the instant petition
inquires into the factual findings of the court a quo. The petitioners are fundamentally
raising a question of fact regarding the appellate court’s finding that the charge of
falsification was not substantially proved. The petitioner would have us sift through
the evidence on record and pass upon whether the signatures found on the Affidavit of
Loss vis-à-vis the pawn tickets are similar or not. This clearly involves a factual inquiry,
the determination of which is the statutory function of the NLRC.[20]

Elementary is the principle that this court is not a trier of facts. Judicial review of labor
cases does not go beyond the evaluation of the sufficiency of the evidence upon which
its labor officials’ findings rest.[21] As such, the findings of facts and conclusion of the
NLRC are generally accorded not only great weight and respect but even clothed with
finality and deemed binding on this Court as long as they are supported by substantial
evidence.[22] We find no basis for deviating from the aforestated doctrine without any
clear showing that the findings of the labor arbiter, as affirmed by the NLRC and the
Court of Appeals, are bereft of sufficient substantiation. “Well-settled is the rule that
the jurisdiction of this Court in a petition for review on certiorari under Rule 45 of the
Revised Rules of Court is limited to reviewing only errors of law, not of fact, unless the
factual findings complained of are completely devoid of support from the evidence on
record or the assailed judgment is based on a gross misapprehension of facts.”[23] What
is more, factual findings of quasi-judicial agencies like the NLRC, when affirmed by the
Court of Appeals, are conclusive upon the parties and binding on this Court.[24]

In the case at bar, the issue of the veracity of the signature appearing on the questioned
Affidavit of Loss has been undoubtedly passed upon by, not one, not two, but three
tribunals all having the same findings that there is no evident showing that the said
document is indeed falsified.

Be that as it may, we believe it proper to address and clarify petitioners’ postulation that
the Court of Appeals adopted the view that the degree of proof required in Labor cases
is proof beyond reasonable doubt instead of merely substantial evidence.

Petitioners allege that they have lost trust and confidence in the respondent due to the
latter’s actions. The Labor Arbiter, however, found it hard to see the basis of the loss of
trust and confidence in the light of the insufficiency of evidence presented by the
petitioners, succinctly put thus:

. . . There is no showing that the affidavit of loss was a falsified one. Neither is there any
competent evidence submitted by the respondents to prove that Mrs. Montenegro was
the one who caused the execution thereof, granting that the same is a falsified one.
Henceforth, her dismissal from employment based on the charge against her is illegal.
[25]

The NLRC, affirming the aforequoted pronouncement of the Labor Arbiter, added that:

The Affidavit of Loss not having been repudiated by the one who executed the same,
said affidavit stands and cannot be said to have been forged or a fake one. Hence, we
sustain the findings and ruling of the Labor Arbiter relative to complainant Ms.
Montenegro.
PHILIPPINE PIZZA, INC., formerly G.R. No. 154315
PHILIPPINE FRANCHISE
PROGRESSIVE DEVELOPMENT Present:
CORP., PIZZA HUT DIVISION,
and JANET RUTH M. SOLSOLOY, Davide, Jr., C.J.,
Petitioners, (Chairman),
Quisumbing,
Ynares-Santiago,
- versus - Carpio, and
Azcuna, JJ.

KIM M. BUNGABONG, Promulgated:


Respondent.
May 9, 2005
x - - - - - - - - - - - - - - - - - - - - - - - - - - - - - - - - - - - - - - - - - - - - - - - - - - -x

DECISION
QUISUMBING, J.:

FACTS:

Respondent Bungabong had been working for five years as a food attendant in peti-
tioners’ Ermita outlet. On December 6, 1997 at around 1:30 a.m., the Duty Manager
Alvin Biscocho, allegedly caught one Felix Sabado, another employee, consuming some
beer from the establishment’s beer dispenser. While the duty manager did not actually
see respondent, he concluded that respondent was involved too, because earlier that
night, a driver, Jonathan Andra, reported that he saw respondent with Sabado drinking
beer from the dispenser. The next day, the duty manager called respondent, inquiring
about the latter’s involvement, and showed him a letter of Sabado admitting to the
offense of drinking beer, and then told him to file an incident report. Thereafter,
Criselda Cusi, the outlet’s unit manager, issued an offense notice. Respondent denied
any involvement in the theft of beer, asserting that only Sabado was involved and was
caught. Cusi reported the incident to the head office of Pizza Hut.
On December 15, respondent was informed of his preventive suspension. He was told
to report to the Human Resources Department (HRD) of the company for investigation.
During the investigation, respondent stated, driver Andra was with the Vice-President
for HRD, co-petitioner Janet Ruth M. Solsoloy. Andra then pointed to respondent, and
stated that respondent was with Sabado in drinking the company’s beer on December 5,
1997, at around 11:30 to 12:00 p.m. A guard on duty, Rossman Manaloto, also stated
that on the evening of said date, he confronted respondent and asked why respondent
smelled of beer, but respondent ignored the inquiry and hurriedly left. A crew member
of the outlet, Daniel Gatdula, also reported on how the respondent bragged how much
beer he could drink on his way passing out of the beer dispenser area.
After the investigation, a certain Ms. Ellen of the HRD explained to the respondent the
penalty for his alleged offense. She told respondent he should no longer report for
work. Respondent was advised to go home. He then refused to receive his letter of
termination, which followed after the investigation.

ISSUE:

Whether or not respondent held a position entrusted with trust and confidence?

RULING:

Contrary to the ruling of the Labor Arbiter and the NLRC, which even tually the
Court of Appeals affirmed, we find that petitioner Philippine Pizza, Inc. established the
existence of just cause to terminate respondent on the ground of loss of trust and
confidence.
Where the employee has access to the employer’s property in the form of
merchandise and articles for sale, the relationship of the employer and the employee
necessarily involves trust and confidence.[24] Hence, when respondent drank stolen
beer from the dispenser of Pizza Hut-Ermita on December 6, 1997, he gave cause for his
termination and his termination was within the ambit of Article 282 of the Labor Code.
Now, however, as regards violations of the procedural requirement for valid
dismissal, the petitioners could be justly faulted. Book V, Rule XIV of the Omnibus
Rules Implementing Batas Pambansa Blg. 130 in effect at the time respondent was
terminated, outlines the procedure for termination of employment.[25] It provides as
follows:
Sec. 1. Security of tenure and due process. – No worker shall be dis missed except
for a just or authorized cause provided by law and after due process.
Sec. 2. Notice of Dismissal. – Any employer who seeks to dis miss a worker shall
furnish him a written notice stating the particular acts or omissions constituting the
grounds for his dismissal. In cases of abandonment of work, the notice shall be served
at the worker’s last known address.
. . .
Sec. 5. Answer and hearing. – The worker may answer the allegations stated against
him in the notice of dismissal within a reasonable period from receipt of such notice.
The employer shall afford the worker ample opportunity to be heard and to defend
himself with the assistance of his representatives, if he so desires.
Sec. 6. Decision to dismiss. – The employer shall immediately notify a worker in
writing of a decision to dismiss him stating clearly the reasons therefor.

In this case, we find that petitioners violated respondent’s right to due process,
particularly the requirement of first notice. The offense notice[26] petitioners gave to
respondent is insufficient first notice because it did not comply with the requirement of
the law that the first written notice must apprise the employee that his termination is
being considered due to the acts stated in the notice.[27] The first notice issued in this
case merely stated that respondent is being charged of dispensing and drinking beer on
December 5, 1997, around 11:30 to 11:45 p.m.,[28] and nothing more.
HERMINIGILDO INGUILLO AND ZENAIDA BERGANTE,
Petitioners
- versus -
FIRST PHILIPPINE SCALES, INC. and/or AMPARO POLICARPIO, MANAGER,
Respondents.
G.R. No. 165407

FACTS:

First Philippine Scales, Inc. (FPSI), a domestic corporation engaged in the


manufacturing of weighing scales, employed Bergante and Inguillo as assemblers on
August 15, 1977 and September 10, 1986, respectively.

In 1991, FPSI and First Philippine Scales Industries Labor Union (FPSILU)[3]
entered into a Collective Bargaining Agreement (CBA),[4] the duration of which was for
a period of five (5) years starting on September 12, 1991 until September 12, 1996. On
September 19, 1991, the members of FPSILU ratified the CBA in a document entitled
RATIPIKASYON NG KASUNDUAN.[5] Bergante and Inguillo, who were members of
FPSILU, signed the said document.[6]

During the lifetime of the CBA, Bergante, Inguillo and several FPSI employees
joined another union, the Nagkakaisang Lakas ng Manggagawa (NLM), which was
affiliated with a federation called KATIPUNAN (NLM-KATIPUNAN, for brevity).
Subsequently, NLM-KATIPUNAN filed with the Department of Labor and
Employment (DOLE) an intra-union dispute[7] against FPSILU and FPSI. In said case,
the Med-Arbiter decided[8] in favor of FPSILU. It also ordered the officers and
members of NLM-KATIPUNAN to return to FPSILU the amount of P90,000.00
pertaining to the union dues erroneously collected from the employees. Upon finality
of the Med-Arbiter's Decision, a Writ of Execution[9] was issued to collect the adjudged
amount from NLM-KATIPUNAN. However, as no amount was recovered, notices of
garnishment were issued to United Coconut Planters Bank (Kalookan City Branch)[10]
and to FPSI[11] for the latter to hold for FPSILU the earnings of Domingo Grutas, Jr.
(Grutas) and Inguillo, formerly FPSILU's President and Secretary for Finance,
respectively, to the extent of P13,032.18. Resultantly, the amount of P5,140.55 was
collected,[12] P1,695.72 of which came from the salary of Grutas, while the P3,444.83
came from that of Inguillo.
Meanwhile, on March 29, 1996, the executive board and members of the FPSILU
addressed a document dated March 18, 1996 denominated as “Petisyon”[13] to FPSI's
general manager, Amparo Policarpio (Policarpio), seeking the termination of the
services of the following employees, namely: Grutas, Yolanda Tapang, Shirley Tapang,
Gerry Trinidad, Gilbert Lucero, Inguillo, Bergante, and Vicente Go, on the following
grounds:[14] (1) disloyalty to the Union by separating from it and affiliating with a rival
Union, the NLM-KATIPUNAN; (2) dereliction of duty by failing to call periodic
membership meetings and to give financial reports; (3) depositing Union funds in the
names of Grutas and former Vice-President Yolanda Tapang, instead of in the name of
FPSILU, care of the President; (4) causing damage to FPSI by deliberately slowing down
production, preventing the Union to even attempt to ask for an increase in benefits from
the former; and (5) poisoning the minds of the rest of the members of the Union so that
they would be enticed to join the rival union.

ISSUE:

Whether or not the security union clause was valid

RULING:

In terminating the employment of an employee by enforcing the Union Security


Clause, the employer needs only to determine and prove that: (1) the union security
clause is applicable; (2) the union is requesting for the enforcement of the union security
provision in the CBA; and (3) there is sufficient evidence to support the union's decision
to expel the employee from the union or company.[43]

We hold that all the requisites have been sufficiently met and FPSI was justified in
enforcing the Union Security Clause, for the following reasons:

First. FPSI was justified in applying the Union Security Clause, as it was a valid
provision in the CBA, the existence and validity of which was not questioned by either
party. Moreover, petitioners were among the 93 employees who affixed their
signatures to the document that ratified the CBA. They cannot now turn their back and
deny knowledge of such provision.

Second. FPSILU acted on its prerogative to recommend to FPSI the dismissal of


the members who failed to maintain their membership with the Union. Aside from
joining another rival union, FPSILU cited other grounds committed by petitioners and
the other employees which tend to prejudice FPSI’s interests, i.e., dereliction of duty -
by failing to call periodic membership meetings and to give financial reports; depositing
union funds in the names of Grutas and former Vice-President Yolanda Tapang,
instead of in the name of FPSILU care of the President; causing damage to FPSI by
deliberately slowing down production, preventing the Union from even attempting to
ask for an increase in benefits from the former; and poisoning the minds of the rest of
the members of the Union so that they would be enticed to join the rival union.

Third. FPSILU's decision to ask for the termination of the employees in the
“Petisyon” was justified and supported by the evidence on record. Bergante and
Inguillo were undisputably former members of FPSILU. In fact, Inguillo was the
Secretary of Finance, the underlying reason why his salary was garnished to satisfy the
judgment of the Med-Arbiter who ordered NLM-KATIPUNAN to return the Union
dues it erroneously collected from the employees. Their then affiliation with FPSILU
was also clearly shown by their signatures in the document which ratified the CBA.
Without a doubt, they committed acts of disloyalty to the Union when they failed not
only to maintain their membership but also disaffiliated from it. They abandoned
FPSILU and even joined another union which works against the former's interests.
This is evident from the intra-union dispute filed by NLM-KATIPUNAN against
FPSILU. Once affiliated with NLM-KATIPUNAN, Bergante and Inguillo proceeded to
recruit other employees to disaffiliate from FPSILU and even collected Union dues from
them.

G.R. No. 75037 April 30, 1987


TANDUAY DISTILLERY LABOR UNION VS NLRC

FACTS:
Private respondents were all employees of Tanduay Distillery, Inc., (TDI) and members
of the Tanduay Distillery Labor Union (TDLU), a duly organized and registered labor
organization and the exclusive bargaining agent of the rank and file employees of the
petitioner company.

A Collective Bargaining Agreement (CBA), was executed between TDI and TDLU. The
CBA was duly ratified by a majority of the workers in TDI including herein private
respondents and contained a union security clause which provides that “all workers
who are or may during the effectivity of the CBA, become members of the Union in
accordance with its Constitution and By-Laws shall, as a condition of their continued
employment, maintain membership in good standing in the Union for the duration of
the agreement.”

While the CBA was in effect and within the contract bar period the private respondents
joined another union, the Kaisahan Ng Manggagawang Pilipino (KAMPIL) and
organized its local chapter in TDI. KAMPIL filed a petition for certification election to
determine union representation in TDI, which development compelled TDI to file a
grievance with TDLU.
TDLU created a committee to investigate its erring members in accordance with its by-
laws which are not disputed by the private respondents. Thereafter, TDLU, through the
Investigating Committee and approved by TDLU's Board of Directors, expelled the
private respondents from TDLU for disloyalty to the Union. By letter, TDLU notified
TDI that private respondents had been expelled from TDLU and demanded that TDI
terminate the employment of private, respondents because they had lost their
membership with TDLU.
The private respondents were later on terminated. In their petition, private respondents
contend that their act of organizing a local chapter of KAMPIL and eventual filing of a
petition for certification election was pursuant to their constitutional right to self-
organization.

ISSUES:
a) whether or not TDI was justified in terminating private respondents' employment in
the company on the basis of TDLU's demand for the enforcement of the Union Security
Clause of the CBA between TDI and TDLU; and
b) whether or not TDI is guilty of unfair labor practice in complying with TDLU's
demand for the dismissal of private respondents.

HELD:
The dismissal of an employee pursuant to a demand of the majority union in accordance
with a union security agreement following the loss of seniority rights is valid and
privileged and does not constitute an unfair labor practice.
Article 249 (e) of the Labor Code as amended specifically recognizes the closed shop
arrangement as a form of union security. The closed shop, the union shop, the
maintenance of membership shop, the preferential shop, the maintenance of treasury
shop, and check-off provisions are valid forms of union security and strength. They do
not constitute unfair labor practice nor are they violations of the freedom of association
clause of the Constitution. There is no showing in these petitions of any arbitrariness or
a violation of the safeguards enunciated in the decisions of this Court interpreting
union security arrangements brought to us for review.

[G.R. No. 146621. July 30, 2004]

RENE P. VALIAO, petitioner, vs. HON. COURT OF APPEALS, NATIONAL LABOR


RELATIONS COMMISSION-FOURTH DIVISION (Cebu City), WEST NEGROS
COLLEGE, respondents.

FACTS:

On February 5, 1990, petitioner Rene Valiao was appointed by private respondent West
Negros College (WNC) as Student Affairs Office (SAO) Director, with a starting salary
of P2,800 per month. On May 14, 1990, he was assigned as Acting Director, Alumni
Affairs Office.
On July 29, 1990, petitioner was transferred to a staff position and designated as Records
Chief at the Registrar’s Office but was again re-assigned as a typist on June 24, 1991.

The latest re-assignment was due to his tardiness and absences, as reflected in the
summary of tardiness and absences report, which showed him to have been absent or
late for work from a minimum of seven (7) to a maximum of seventy-five (75) minutes
for the period March to October 31, 1991, and to have reported late almost every day for
the period November to December 1991.

Copies of his tardiness/absences reports were furnished petitioner, along with


memoranda requiring him to explain but his explanations were either unacceptable or
unsatisfactory. Subsequent reports also showed that he did not change his habits
resulting in tardiness and absences. He was even caught one time manipulating the
bundy clock, thus necessitating another memorandum to him asking him to explain his
dishonest actuations in accomplishing the daily attendance logbook and in using the
bundy clock.

On December 10, 1991, petitioner received a suspension order without pay for fifteen
(15) days effective January 1, 1992, because of dishonesty in reporting his actual
attendance. After serving the suspension, the petitioner reported back to office on
January 16, 1992.

On June 15, 1992, another adverse report on tardiness and absences from the Registrar
was made against the petitioner prompting WNC to send him another memorandum
with an attached tardiness and absences report, calling his attention on his tardiness
and absences for the period February to April 1992.

On June 20, 1992, petitioner sent a letter of appeal and explained his side to the new
college president, Suzette Arbolario-Agustin, who gave petitioner another chance. The
petitioner was then appointed as Information Assistant effective immediately.
However, the petitioner did not immediately assume the post of Information Assistant
prompting the President of private respondent WNC to call his attention. When the
petitioner finally assumed his post, he was allowed a part-time teaching job in the same
school to augment his income.

Sometime in December 1992, WNC won a case against the officials of the union before
the NLRC. Petitioner was ordered to prepare a media blitz of this victory but the
petitioner did not comply with the order on the ground that such a press release would
only worsen the already aggravated situation and strained relations between WNC
management and the union officials.

When petitioner reported for work on the first day of January 1993, he was relieved
from his post and transferred to the College of Liberal Arts as Records Evaluator. Not
for long, the Dean of the Liberal Arts sent a letter to the Human Resources Manager
complaining about the petitioner’s poor performance and habitual absenteeism, as
shown in the daily absence reports.

ISSUE:

Whether or not the acts of the petitioner constitute gross and habitual neglect of duties

RULING

Considering the submissions of the parties as well as the records before us, we find the
petition without merit. Petitioner’s dismissal from employment is valid and justified.

For an employee’s dismissal to be valid, (a) the dismissal must be for a valid cause and
(b) the employee must be afforded due process.[7]

Serious misconduct and habitual neglect of duties are among the just causes for
terminating an employee under the Labor Code of the Philippines. Gross negligence
connotes want of care in the performance of one’s duties. Habitual neglect implies
repeated failure to perform one’s duties for a period of time, depending upon the
circumstances.[8] The Labor Arbiter’s findings that petitioner’s habitual absenteeism
and tardiness constitute gross and habitual neglect of duties that justified his
termination of employment are sufficiently supported by evidence on record.
Petitioner’s repeated acts of absences without leave and his frequent tardiness reflect his
indifferent attitude to and lack of motivation in his work. More importantly, his
repeated and habitual infractions, committed despite several warnings, constitute gross
misconduct unexpected from an employee of petitioner’s stature. This Court has held
that habitual absenteeism without leave constitute gross negligence and is sufficient to
justify termination of an employee.[9]

However, petitioner claims that he was dismissed not for his tardiness or absences but
for his arrest as a suspected drug user. His claim, however, is merely speculative. We
find such contention devoid of basis. First, the decisions of the Labor Arbiter, the
NLRC, and the Court of Appeals are indubitable. They show that indeed petitioner had
incurred numerous and repeated absences without any leave. Moreover, he was not
punctual in reporting for work. These unexplained absences and tardiness were
reflected on the summary reports submitted by WNC before the labor arbiter, but
petitioner failed to controvert said reports. Second, contrary to petitioner’s assertion,
the NLRC did not base its conclusions on the fact of the arrest of petitioner for violation
of Rep. Act No. 6425 but on the totality of the number of infractions incurred by the
petitioner during the period of his employment in different positions he occupied at
WNC. Thus:

In the case of petitioner Valiao, his services were terminated by private respondent after
having been found guilty of serious misconduct and gross habitual neglect of duty
which was aggravated by the January 28, 1993 incident. In exercising such
management prerogative, due process was properly observed. Private respondent
presented sufficient evidence to support its act in terminating the services of petitioner.
Private respondent took into consideration the totality of the infractions or the number
of violations committed by petitioner during the period of employment. Furthermore,
it hardly needs reminding that, in view of petitioner’s position and responsibilities, he
must demonstrate a scrupulous regard for rules and policies befitting those who would
be role models for their young charges.[10] (Emphasis and italics supplied)

Indeed, even without the arrest incident, WNC had more than enough basis for
terminating petitioner from employment. It bears stressing that petitioner’s absences
and tardiness were not isolated incidents but manifested a pattern of habituality. In one
case, we held that where the records clearly show that the employee has not only been
charged with the offense of highgrading but also has been warned 21 times for absences
without official leave, these repeated acts of misconduct and willful breach of trust by
an employee justify his dismissal and forfeiture of his right to security of tenure. The
totality of infractions or the number of violations committed during the period of
employment shall be considered in determining the penalty to be imposed upon an
erring employee. The offenses committed by him should not be taken singly and
separately but in their totality. Fitness for continued employment cannot be
compartmentalized into tight little cubicles of aspects of character, conduct, and ability
separate and independent of each other.

Needless to say, so irresponsible an employee like petitioner does not deserve a place in
the workplace, and it is within the management’s prerogative of WNC to terminate his
employment. Even as the law is solicitous of the welfare of employees, it must also
protect the rights of an employer to exercise what are clearly management prerogatives.
As long as the company’s exercise of those rights and prerogative is in good faith to
advance its interest and not for the purpose of defeating or circumventing the rights of
employees under the laws or valid agreements, such exercise will be upheld.

Alvarez vs. Golden Tri Bloc Inc., G.R. No. 202158, September 25, 2013

FACTS:

Sometime in November 1996, respondent GTBI hired the petitioner as a Service Crew in
one of its Dunkin Donuts franchise store in Antipolo City, Rizal. Six (6) months later, he
attained the status of a regular employee. He was thereafter promoted as Shift Leader
and served as such for four (4) years. Sometime in 2001, he was again promoted as
Outlet Supervisor and was assigned to three (3) Dunkin Donuts outlets located at San
Roque, Cogeo and Super 8, Masinag, all in Antipolo City. He received a monthly salary
of P10,000.00.

On May 27, 2009, the petitioner reported for duty at around 12:30 in the afternoon at
Dunkin Donuts, Super 8, Masinag branch. Since his time card was at the San Roque
branch, he telephoned Chastine3 Kaye Sambo (Sambo), shift leader, and requested her
to “punch-in” his time card to reflect that he is already on duty. She obliged. Roland
Salindog (Salindog), the petitioner’s senior officer called the Super 8, Masinag branch
and verified that he has indeed reported for work.

The following day, however, the petitioner was informed by Sambo that both of them
are suspended and that he had to prepare an incident report regarding his time card.
In his incident report4 dated May 29, 2009, the petitioner admitted instructing Sambo to
punch-in his timecard. He explained that he went straight to and arrived at the Super 8,
Masinag branch at around 12:35 p.m. He inspected the stocks in the branch and taught a
certain ‘Ritz’ on how to prepare stocks acquisition report for June 2009. He owned up to
his fault and stated that he should have instead recorded the time of his arrival by
writing on the time card and that he should have brought it with him. He apologized
and promised that a similar incident will not happen again.

On June 5, 2009, GTBI sent him a letter directing him to report to the main office for a
dialogue on June 9, 2009 failing which would amount to the waiver of his right to be
heard and the management may make a decision based only on his written
explanation.5 The dialogue pushed through. After which the petitioner was placed on
preventive suspension for 30 days without pay.

On June 23, 2009, GTBI notified the petitioner of its decision to terminate his
employment effective that day on the ground of loss of trust.

ISSUE:

Whether or not the termination on the ground of loss was valid?

RULING:

"In Merin v. NLRC, the Court ruled that in determining the sanction imposable to an
employee, the employee may consider and weigh his other past infractions, thus:
"'The totality of infractions or the number of violations committed during the period of
employment shall be considered in determining the penalty to be imposed upon an
erring employee. The offenses committed by petitioner should not be taken singly and
separately. Fitness for continued employment cannot be compartmentalized into tight
little cubicles of aspects of character, conduct and ability separate and independent of
each other. While it may be true that petitioner was penalized for his previous
infractions, this does not and should not mean that his employment record would be
wiped clean of his infractions. After all, the record of an employee is a relevant
consideration in determining the penalty that should be meted out since an employee's
past misconduct and present behavior must be taken together in determining the
proper imposable penalty. Despite the sanctions imposed upon petitioner, he continued
to commit misconduct and exhibit undesirable behavior on board. Indeed, the employer
cannot be compelled to retain a misbehaving employee, or one who is guilty of acts
inimical to its interests. It has the right to dismiss such an employee if only as a measure
of self-protection. (Citations omitted)'"
The NLRC and the CA were thus correct in applying the totality of infractions rule and
in adjudging that the petitioner's dismissal was grounded on a just and valid cause."

Yrasuegui vs. Phil Airlines, G.R. No. 168081, Oct. 17, 2008

Facts:

Armando G. Yrasuegui was a former international flight steward of Philippine Airlines,


Inc. (PAL). He stands five feet and eight inches (5‘8‖) with a large body frame. The
proper weight for a man of his height and body structure is from 147 to 166 pounds, the
ideal weight being 166 pounds, as mandated by the Cabin and Crew Administration
Manual of PAL. His weight problem dates back to 1984 when PAL advised him to go on
an extended vacation leave from December 29, 1984 to March 4, 1985 to address his
weight concerns. For failure to meet the weight standards another leave without pay
from March 5, 1985 to November 1985 was imposed. He met the required weight and
was allowed to work but his weight problem recurred, thus another leave without pay
from October 17, 1988 to February 1989. From 1989 to 1992 his weight fluctuated from
209lb, 215lb, 217lb, 212lb, and 205. During that period he was requested to lose weight
and to report for weight checks which he constantly failed to do. In the meantime his
status was ―off-duty.‖ Finally in 1993, petitioner was formally informed by PAL that
due to his inability to attain his ideal weight, ―and considering the utmost leniency‖
extended to him ―which spanned a period covering a total of almost five (5) years,‖ his
services were considered terminated ―effective immediately.‖ He then filed a complaint
for illegal dismissal against PAL. The Labor Arbiter ruled that he was illegally
dismissed and entitles to reinstatement, backwages and attorney‘s fees. The NLRC
affirmed the LA. The CA reversed the NLRC.

Issue:
Whether or not petitioner was illegally dismissed.

Ruling:

The obesity of petitioner is a ground for dismissal under Article 282(e) of the Labor
Code. The weight standards of PAL constitute a continuing qualification of an
employee in order to keep the job. Tersely put, an employee may be dismissed the
moment he is unable to comply with his ideal weight as prescribed by the weight
standards. The dismissal would fall under Article 282(e) of the Labor Code. As
explained by the CA:

x x x [T]he standards violated in this case were not mere ―orders‖ of the employer; they
were the ―prescribed weights‖ that a cabin crew must maintain in order to qualify for
and keep his or her position in the company. In other words, they were standards that
establish continuing qualifications for an employee‘s position. … The failure to meet the
employer‘s qualifying standards is in fact a ground that does not squarely fall under
grounds (a) to (d) and is therefore one that falls under Article 282(e) – the ―other causes
analogous to the foregoing.‖

By its nature, these ―qualifying standards‖ are norms that apply prior to and after an
employee is hired. x x x

We hold that the obesity of petitioner, when placed in the context of his work as flight
attendant, becomes an analogous cause under Article 282(e) of the Labor Code that
justifies his dismissal from the service. His obesity may not be unintended, but is
nonetheless voluntary.
John Hancock Life Insurance Corp. vs. Davis, G.R. No. 169549, Sept. 3, 2008

Facts:

Joanna Cantre Davis was agency administration officer of John Hancock Life Insurance
Corporation. On October 18, 2000, Patricia Yuseco, JHLIC‘s corporate affairs manager,
discovered that her wallet was missing. She immediately reported the loss of her credit
cards to AIG and BPI Express. To her surprise, she was informed that "Patricia Yuseco"
had just made substantial purchases using her credit cards in various stores in the City
of Manila. She was also told that a proposed transaction in Abenson's-Robinsons Place
was disapproved because "she" gave the wrong information upon verification. Because
loss of personal property among its employees had become rampant in its office,
petitioner sought the assistance of NBI. The NBI, obtained a security video from
Abenson's showing the person who used Yuseco's credit cards. Yuseco and other
witnesses positively identified the person in the video as Davis NBI and Yuseco filed a
complaint for qualified theft against Davis but because the affidavits presented by the
NBI (identifying respondent as the culprit) were not properly verified, the city
prosecutor dismissed the complaint due to insufficiency of evidence. Meanwhile,
petitioner placed Davis under preventive suspension and instructed her to cooperate
with its ongoing investigation. Davis filed a complaint for illegal dismissal alleging that
petitioner terminated her employment without cause. The labor arbiter, in
May 21, 2002, found that Davis committed serious misconduct (she was the principal
suspect for qualified theft committed inside petitioner's office during work hours).
There was a valid cause for her dismissal. Thus, the complaint was dismissed for lack of
merit. Upon appeal, NLRC affirmed the labor arbiter in July 31, 2003 and denied her
motion for reconsideration in October 30, 2003. Upon petition for certiorari filed with
the CA, CA on July 4, 2005 granted the petition holding that the labor arbiter and NLRC
merely adopted the findings of the NBI regarding respondent's culpability. Because the
affidavits of the witnesses were not verified, they did not constitute substantial
evidence. The labor arbiter and NLRC should have assessed evidence independently as
"unsubstantiated suspicions, accusations and conclusions of employers (did) not
provide legal justification for dismissing an employee". Petitioner moved for
reconsideration but it was denied. Hence, this petition where petitioner argues that the
ground for an employee's dismissal need only be proven by substantial evidence. Thus,
the dropping of charges against an employee (especially on a technicality such as lack of
proper verification) or his subsequent acquittal does not preclude an employer from
dismissing him due to serious misconduct.

Issue:

Whether or not petitioner substantially proved the presence of valid cause for
respondent's termination.

Ruling:

Supreme Court granted the petition and ruled that petitioner validly dismissed Davis
for cause analogous to serious misconduct. Article 282 of the Labor Code provides:
Termination by Employer. — An employer may terminate an employment for any of
the following causes:
(a) Serious misconduct or willful disobedience by the employee of the lawful orders of
his employer or his representatives in connection with his work;
(e) Other causes analogous to the foregoing.

Misconduct involves "the transgression of some established and definite rule of action,
forbidden act, a dereliction of duty, willful in character, and implies wrongful intent
and not mere error in judgment". For misconduct to be serious and therefore a valid
ground for dismissal, it must be: of grave and aggravated character and not merely
trivial or unimportant and connected with the work of the employee.

In this case, petitioner dismissed Davis based on the NBI's finding that the latter stole
and used Yuseco's credit cards. But since the theft was not committed against petitioner
itself but against one of its employees, respondent's misconduct was not work-related
and therefore, she could not be dismissed for serious misconduct. Nonetheless, Article
282 (e) of the Labor Code talks of other analogous causes or those which are susceptible
of comparison to another in general or in specific detail. For an employee to be validly
dismissed for a cause analogous to those enumerated in Article 282, the cause must
involve a voluntary and/or willful act or omission of the employee. A cause analogous
to serious misconduct is a voluntary and/or willful act or omission attesting to an
employee's moral depravity. Theft committed by an employee against a person other
than his employer, if proven by substantial evidence, is a cause analogous to serious
misconduct. Did petitioner substantially prove the existence of valid cause for
respondent's separation? Yes. The labor arbiter and the NLRC relied not only on the
affidavits of the NBI's witnesses but also on that of respondent. They likewise
considered petitioner's own investigative findings. Clearly, they did not merely adopt
the findings of the NBI but independently assessed evidence presented by the parties.
Their conclusion (that there was valid cause for respondent's separation from
employment) was therefore supported by substantial evidence.
Magnolia Dairy Products Corporation v. NLRC
G.R. NO. 114952, January 29, 1996

Facts: Petitioner, a division of San Miguel Corporation (SMC), entered into a contract of
service with Skillpower, Inc., a duly organized corporation engaged in the business of
offering and providing manpower services to the public. On June 11, 1983, Skillpower,
Inc., assigned private respondent Jenny A. Calibo to petitioner’s Tetra Paster Division
with these functions: “(i)to remove “bulgings” (damaged goods) from dilapidated
cartons; (ii)to replace damaged goods and re-paste the carton thereof; (iii)to dispose the
damaged goods or returned goods from Magnolia’s warehouse to avoid bad odors; and
(iv)to clean leftovers of leaking tetra pack by mopping or washing the contaminated
premises.” In September 1986, Skillpower, Inc., pulled-out private respondent from
petitioner’s Tetra Paster Division, but assigned her back on May 2, 1987 with the same
functions. When petitioner’s contract with Skillpower, Inc., expired, private respondent
applied with Lippercon Services, Inc., also a corporation engaged in providing
manpower services. In July 1987, Lippercon Services, Inc., assigned her to petitioner’s
Tetra Paster Division as a cleaning aide. In December 1987, she was terminated from
service due to petitioner’s installation of automated machines. On July 11, 1989, private
respondent instituted a complaint for illegal dismissal against petitioner. In answer
thereto, petitioner averred that it has no employer-employee relationship with private
respondent and that the dismissal was prompted by the installation of labor saving
devices - an authorized cause for dismissal under the Labor Code, as amended. The
Labor Arbiter ruled that petitioner is the private respondent’s employer because
Skilipower, Inc., and Lippercon Services, Inc., were mere “labor-only” contractors
falling under Section 9, Rule VIII, Book III of the Omnibus Rules Implementing the
Labor Code. The installation of labor saving devices was also ruled a valid ground for
the termination of private respondent’s employment, but the Labor Arbiter emphasized
that this did not exculpate petitioner from the charge of illegal dismissal for its failure to
observe the due process of law in terminating from service its employee.

Issue: Was there an authorized cause for the termination?

Ruling: The law authorizes an employer, like the herein petitioner, to terminate the
employment of any employee due to the installation of labor saving devices. The
installation of these devices is a management prerogative, and the courts will not
interfere with its exercise in the absence of abuse of discretion, arbitrariness, or
maliciousness on the part of management, as in this case. Nonetheless, this did not
excuse petitioner from complying with the required written notice to the employee and
to the Department of Labor and Employment (DOLE) at least one month before the
intended date of termination. This procedure enables an employee to contest the reality
or good faith character of the asserted ground for the termination of his services before
the DOLE.

Redundancy

Asian Alcohol Corporation v. NLRC


G.R. No. 131108, March 25, 1999

Facts: In September, 1991, the Parsons family, who originally owned the controlling
stocks in Asian Alcohol, were driven by mounting business losses to sell their majority
rights to prior Holdings, Inc. (hereinafter referred to as Prior Holdings). The next
month, Prior Holdings took over its management and operation. To thwart further
losses, Prior Holdings implemented a reorganizational plan and other cost-saving
measures. Some one hundred seventeen (117) employees out of a total workforce of
three hundred sixty (360) were separated. Seventy two (72) of them occupied redundant
positions that were abolished. Of these positions, twenty one (21) were held by union
members and fifty one (51) by non-union members. The six (6) private respondents are
among those union members whose positions were abolished due to redundancy.
Private respondents Carias, Martinez, and Sendon were water pump tenders; Amacio
was a machine shop mechanic; Verayo was a briquetting plant operator while Tormo
was a plant helper under him. They were all assigned at the Repair and Maintenance
Section of the Pulupandan plant. In October, 1992, they received individual notices of
termination effective November 30, 1992.

Issue: Was the termination valid due to redundancy?

Ruling: In the case at bar, Prior Holdings took over the operations of Asian Alcohol in
October 1991. Plain to see, the last quarter losses in 1991 were already incurred under
the new management. There were no signs that these losses would abate. Irrefutable
was the fact that losses have bled Asian Alcohol incessantly over a span of several
years. They were incurred under the management of the Parsons family and continued
to be suffered under the new management of Prior Holdings. Ultimately, it is Prior
Holding that will absorb all the losses, including those incurred under the former
owners of the company. The law gives the new management every right to undertake
measures to save the company from bankruptcy. We find that the reorganizational plan
and comprehensive cost-saving program to turn the business around were nor designed
to bust the union of the private respondent. Retrenched were one hundred seventeen
(117) employees. Seventy two (72) of them including private respondent were separated
because their positions had become redundant. In this context, what may technically be
considered as redundancy may verily be considered as retrenchment measures. Their
positions had to be declared redundant to cut losses. Redundancy exists when the
service capability of the work is in excess of what is reasonably needed to meet the
demands on the enterprise. A redundant position is one rendered superfluous by any
number of factors, such as overhiring of workers, decreased volume of business,
dropping of a particular product line previously manufactured by the company or
phasing out of a service activity priorly undertaken by the business. Under these
conditions, the employer has no legal obligation to keep in its payroll more employees
than are necessary for the operation of its business. For the implementation of a
redundancy program to be valid, the employer must comply with the following
requisites: (1) written notice served on both the employees and the Department of Labor
and Employment at least one moth prior to the intended date of retrenchment; (2)
payment of separation pay equivalent to at least one month pay or at least one month
pay for every year of service whichever is higher; (3) good faith in abolishing the
redundant positions; and (4) fair and reasonable criteria in ascertaining what positions
are to be declared redundant and accordingly abolished.

Wiltshire File Co., Inc., v. NLRC


G.R. No. 82249 February 7, 1991

Facts: Private respondent Vicente T. Ong was the Sales Manager of petitioner Wiltshire
File Co., Inc. ("Wiltshire") from 16 March 1981 up to 18 June 1985. As such, he received a
monthly salary of P14,375.00 excluding commissions from sales which averaged
P5,000.00 a month. He also enjoyed vacation leave with pay equivalent to P7,187,50 per
year, as well as hospitalization privileges to the extent of P10,000.00 per year. On 13
June 1985, upon private respondent's return from a business and pleasure trip abroad,
he was informed by the President of petitioner Wiltshire that his services were being
terminated. Private respondent maintains that he tried to get an explanation from
management of his dismissal but to no avail. On 18 June 1985, when private respondent
again tried to speak with the President of Wiltshire, the company's security guard
handed him a letter which formally informed him that his services were being
terminated upon the ground of redundancy. Private respondent filed, on 21 October
1985, a complaint before the Labor Arbiter for illegal dismissal alleging that his position
could not possibly be redundant because nobody (save himself) in the company was
then performing the same duties. Private respondent further contended that retrenching
him could not prevent further losses because it was in fact through his remarkable
performance as Sales Manager that the Company had an unprecedented increase in
domestic market share the preceding year. For that accomplishment, he continued, he
was promoted to Marketing Manager and was authorized by the President to hire four
(4) Sales Executives five (5) months prior to his termination.

Issue: Is the termination valid due to redundancy?

Ruling: Turning to the legality of the termination of private respondent's employment,


The Court finds merit in petitioner's basic argument. The Court was unable to sustain
public respondent NLRC's holding that private respondent's dismissal was not justified
by redundancy and hence illegal. In the first place, we note that while the letter
informing private respondent of the termination of his services used the word
"redundant", that letter also referred to the company having "incur[red] financial losses
which [in] fact has compelled [it] to resort to retrenchment to prevent further losses". 3
Thus, what the letter was in effect saying was that because of financial losses,
retrenchment was necessary, which retrenchment in turn resulted in the redundancy of
private respondent's position.

Smart Communications, Inc., v. Astorga


G.R. No. January 28, 2008

Facts: Regina M. Astorga (Astorga) was employed by respondent Smart


Communications, Incorporated (SMART) on May 8, 1997 as District Sales Manager of
the Corporate Sales Marketing Group/ Fixed Services Division (CSMG/FSD). She was
receiving a monthly salary of P33,650.00. As District Sales Manager, Astorga enjoyed
additional benefits, namely, annual performance incentive equivalent to 30% of her
annual gross salary, a group life and hospitalization insurance coverage, and a car plan
in the amount of P455,000.00. In February 1998, SMART launched an organizational
realignment to achieve more efficient operations. This was made known to the
employees on February 27, 1998.6 Part of the reorganization was the outsourcing of the
marketing and sales force. Thus, SMART entered into a joint venture agreement with
NTT of Japan, and formed SMART-NTT Multimedia, Incorporated (SNMI). Since SNMI
was formed to do the sales and marketing work, SMART abolished the CSMG/FSD,
Astorga’s division. To soften the blow of the realignment, SNMI agreed to absorb the
CSMG personnel who would be recommended by SMART. SMART then conducted a
performance evaluation of CSMG personnel and those who garnered the highest ratings
were favorably recommended to SNMI. Astorga landed last in the performance
evaluation, thus, she was not recommended by SMART. SMART, nonetheless, offered
her a supervisory position in the Customer Care Department, but she refused the offer
because the position carried lower salary rank and rate. Despite the abolition of the
CSMG/FSD, Astorga continued reporting for work. But on March 3, 1998, SMART
issued a memorandum advising Astorga of the termination of her employment on
ground of redundancy, effective April 3, 1998. Astorga received it on March 16, 1998.

Issue: Was there redundancy to justify the termination of Astorga?

Ruling: The Court agrees with the CA that the organizational realignment introduced by
SMART, which culminated in the abolition of CSMG/FSD and termination of Astorga’s
employment was an honest effort to make SMART’s sales and marketing departments
more efficient and competitive. Indeed, out of our concern for those lesser
circumstanced in life, this Court has inclined towards the worker and upheld his cause
in most of his conflicts with his employer. This favored treatment is consonant with the
social justice policy of the Constitution. But while tilting the scales of justice in favor of
workers, the fundamental law also guarantees the right of the employer to reasonable
returns for his investment.38 In this light, we must acknowledge the prerogative of the
employer to adopt such measures as will promote greater efficiency, reduce overhead
costs and enhance prospects of economic gains, albeit always within the framework of
existing laws. Accordingly, we sustain the reorganization and redundancy program
undertaken by SMART. However, as aptly found by the CA, SMART failed to comply
with the mandated one (1) month notice prior to termination. The record is clear that
Astorga received the notice of termination only on March 16, 199839 or less than a
month prior to its effectivity on April 3, 1998. Likewise, the Department of Labor and
Employment was notified of the redundancy program only on March 6, 1998.

Retrenchment

Flight Attendants and Stewards Association of the Philippines (FASAP) v. PAL G.R.
No. 178083, July 22, 2008

Facts: Petitioner FASAP is the duly certified collective bargaining representative of PAL
flight attendants and stewards, or collectively known as PAL cabin crew personnel.
Respondent PAL is a domestic corporation organized and existing under the laws of the
Republic of the Philippines, operating as a common carrier transporting passengers and
cargo through aircraft. On June 15, 1998, PAL retrenched 5,000 of its employees,
including more than 1,400 of its cabin crew personnel, to take effect on July 15, 1998.
PAL adopted the retrenchment scheme allegedly to cut costs and mitigate huge
financial losses as a result of a downturn in the airline industry brought about by the
Asian financial crisis. During said period, PAL claims to have incurred P90 billion in
liabilities, while its assets stood at P85 billion. In implementing the retrenchment
scheme, PAL adopted its so-called “Plan 14” whereby PAL’s fleet of aircraft would be
reduced from 54 to 14, thus requiring the services of only 654 cabin crew personnel.
PAL admits that the retrenchment is wholly premised upon such reduction in fleet, and
to “the strike staged by PAL pilots since this action also translated into a reduction of
flights.” PAL claims that the scheme resulted in “savings x x x amounting to
approximately P24 million per month – savings that would greatly alleviate PAL’s
financial crisis.” Prior to the full implementation of the assailed retrenchment program,
FASAP and PAL conducted a series of consultations and meetings and explored all
possibilities of cushioning the impact of the impending reduction in cabin crew
personnel. However, the parties failed to agree on how the scheme would be
implemented. Thus PAL unilaterally resolved to utilize the criteria set forth in Section
112 of the PAL-FASAP Collective Bargaining Agreement (CBA) in retrenching cabin
crew personnel: that is, that retrenchment shall be based on the individual employee’s
efficiency rating and seniority. While consultations between FASAP and PAL were
ongoing, the latter began implementing its retrenchment program by initially
terminating the services of 140 probationary cabin attendants only to rehire them in
April 1998. Moreover, their employment was made permanent and regular.

Issue: Was the termination valid due to retrenchment?

Ruling: The burden clearly falls upon the employer to prove economic or business losses
with sufficient supporting evidence. Its failure to prove these reverses or losses
necessarily means that the employee’s dismissal was not justified. Any claim of actual
or potential business losses must satisfy certain established standards, all of which must
concur, before any reduction of personnel becomes legal. These are: (1) That
retrenchment is reasonably necessary and likely to prevent business losses which, if
already incurred, are not merely de minimis, but substantial, serious, actual and real, or
if only expected, are reasonably imminent as perceived objectively and in good faith by
the employer; (2) That the employer served written notice both to the employees and to
the Department of Labor and Employment at least one month prior to the intended date
of retrenchment; (3) That the employer pays the retrenched employees separation pay
equivalent to one (1) month pay or at least one-half (½) month pay for every year of
service, whichever is higher; (4) That the employer exercises its prerogative to retrench
employees in good faith for the advancement of its interest and not to defeat or
circumvent the employees’ right to security of tenure; and, (5) That the employer used
fair and reasonable criteria in ascertaining who would be dismissed and who would be
retained among the employees, such as status, efficiency, seniority, physical fitness, age,
and financial hardship for certain workers. In view of the facts and the issues raised, the
resolution of the instant petition hinges on a determination of the existence of the first,
fourth and the fifth elements set forth above, as well as compliance therewith by PAL,
taking to mind that the burden of proof in retrenchment cases lies with the employer in
showing valid cause for dismissal; that legitimate business reasons exist to justify
retrenchment.

Meaning of “retrenchment to prevent losses”

Revidad v. NLRC G.R. No. 111105, June 27, 1995

Facts: It appears that sometime in March, 1988, private respondent Atlantic, Gulf and
Pacific Company of Manila, Inc. (hereafter, AG & P ) terminated the services of 178
employees, including herein petitioners, under a redundancy program. As a
consequence, a complaint for illegal dismissal with prayer for reinstatement was filed
by herein petitioners (except Jose Español) with public respondent. These cases were
subsequently decided in favor of petitioners, as a result of which they were reinstated
on July 8, 1991 and assigned to the Batangas plant of private respondent. The records
show, however, that pursuant to Presidential Directive No. 0191 2 issued on July 25,
1991 by the company's president and containing management's decision to lay off 40%
of the employees due to financial losses incurred from 1989-1990, AG & P implemented
and effected, starting August 3, 1991, the temporary lay-off of some 705 employees. By
reason thereof, the AG & P United Rank and File Association (URFA, for facility), which
was the employees' union, staged a strike. In a conciliation conference over the labor
dispute held before the National Conciliation and Mediation Board on August 13, 1991,
the parties agreed to submit the legality of the lay-offs to voluntary arbitration.

Issue: Was there a need for retrenchment to prevent losses?

Ruling: The Court is accordingly convinced, and so hold, that both the retrenchment
program of private respondent and the dismissal of petitioners were valid and legal.
First, it has been sufficiently and convincingly established by AG & P before the
voluntary arbitrator that it was suffering financial reverses. Even the rank and file
union at AG & P did not contest the fact that management had been undergoing
financial difficulties for the past several years. Hence, the voluntary arbitrator
considered this as an admission that indeed AG & P was actually experiencing adverse
business conditions which would justify the exercise of its management prerogative to
retrench in order to avoid the not so remote possibility of the closure of the entire
business which, in the opinion of the voluntary arbitrator, would in the last analysis be
adverse to both the management and the union. Second, the voluntary arbitrator's
conclusions were premised upon and substantiated by the audited financial statements
and the auditor's reports of AG & P for the years 1987 to 1991. 14 These, financial
statements audited by independent external auditors constitute the normal and reliable
method of proof of the profit and loss performance of a company. Third, contrary to
petitioners' asseverations, proof of actual financial losses incurred by the company is
not a condition sine qua non, for retrenchment. Retrenchment is one of the economic
grounds to dismiss employees, which is resorted to by an employer primarily to avoid
or minimize business losses. In its ordinary connotation, the phrase "to prevent losses"
means that retrenchment or termination of the services of some employees is authorized
to be undertaken by the employer sometime before the anticipated losses are actually
sustained or realized. It is not, in other words, the intention of the lawmaker to compel
the employer to stay his hand and keep all his employees until after losses shall have in
fact materialized. If such an intent were expressly written into the law, that law may
well be vulnerable to constitutional attack as unduly taking property from one man to
be given to another.

Best evidence of losses


Saballa v. NLRC G.R. Nos. 102472-84, August 22, 1996

Facts: On April 23, 1988, Arturo Margallo, General Manager of the Camarines Sur III
Electric Cooperative, Inc. (private respondent herein), issued Memorandum No. 24-88
providing for “austerity measures (retrenchment).” On the same date, private
respondent filed with the Department of Labor and Employment’s Regional Office No.
V in Legaspi City a Notice of Retrenchment covering some thirty (30) employees on the
basis of the guidelines and priorities specified in the abovementioned memorandum.
The Regional Director in his Resolution dated June 6, 1988 granted authority to
terminate the employment of the said 30 employees “pursuant to the categories,
priorities and effective dates under Memo No. 24-88.” On June 20, 1988, Margallo
issued Memorandum No. 60-88[8] declaring some fifty-two (52) employees, including
herein petitioners Juan Saballa, Lailani Miranda, Nelia Ibarrientos, Helen Quiambao,
Wilberto Amparado and Fidel Manaog, on “forced leave without pay for a period of
three (3) months,” effective five (5) days after receipt by the employees concerned. Such
“forced leave” was purportedly part of the cost-saving measures instituted to enable the
Coop “to meet (its) financial obligations especially with NPC and NEA.” The memo
assured the subject employees of rehiring “as soon as the Coop shall have financially
recovered/regained its financial viability expected within the above specified period.”
A copy of said memorandum was furnished the Regional Office of the DOLE in Legaspi
City on June 23, 1988.

Issue: Is the termination justified?

Ruling: Given the preceding discussion, it is indisputable that private respondent failed
to meet the abovestated requirements for a valid retrenchment due to imminent
business losses, since (1) the expected losses were not proven to be substantial; (2) the
expected losses were not shown to be imminent as private respondent was able to
afford re-hiring of some of the non-tenurial employees on “forced leave”; and (3) the
retrenchment was not shown to be reasonably necessary and likely to effectively
prevent the expected losses. And, neither the losses already realized nor the imminent
losses sought to be forestalled were proven by sufficient and convincing evidence.
Moreover, the private respondent admitted but failed to explain why it rehired
previously retrenched employees who were even non-tenurial, during the pendency of
the complaints for illegal dismissal, when there were still a number of regular
employees in the same situation. Petitioners also alleged that, immediately after their
termination, private respondent hired replacements to fill their positions. This
allegation, supported by the affidavit of petitioners’ witness Marlene Cerillo, remained
unrebutted and uncontroverted by private respondent. This militates strongly against
private respondent’s claim of good faith in implementing reductions of its work force to
reduce costs. And, although Memorandum No. 24-88 set out the priorities/categories to
be observed in implementing the personnel reduction program, the same was not
applied to the petitioners, who, being regular employees, did not fall under any of the
categories mentioned in said memorandum, and who therefore ought not have been
retrenched — at least not under said memorandum. This Court has repeatedly enjoined
employers to adopt and observe fair and reasonable standards to effect retrenchment.
The private respondent adopted in its Memo No. 24-88 a set of criteria in retrenching
employees in accordance with its cost-reduction program, but discarded these self-
imposed criteria when it came to the retrenchment of petitioners, thus rendering its
action arbitrary. Further, it is undisputed that Sec. 1, Article XI of the Collective
Bargaining Agreement of September 13, 1988 between private respondent and the
employees’ union stipulates that “seniority in service to the company shall be x x x
considered in lay-off or reduction of working force.” Thus, the subject retrenchment is
violative of this stipulation as well. The private respondent’s demonstrated arbitrariness
in the selection of which of its employees to retrench is further proof of the illegality of
the subject retrenchment, not to mention private respondent’s bad faith. And lastly, we
note that the termination was made effective five (5) days after receipt of notice. The
lack of the thirty (30) days notice prior to retrenchment as required under Article 283 of
the Labor Code further bolster the conclusion that the subject retrenchment was illegal.
Thus, it is ineludible that we should agree with petitioners’ contention that, contrary to
the public respondent’s finding, the retrenchment of petitioners by private respondent
constituted illegal dismissal.

Redundancy vs. Retrenchment

Andrada v. NLRC G.R. No. 173231, December 28, 2007

Facts: Petitioners Ruben Andrada, Jovencio Poblete, Filamer Alfonso, Harvey Cayetano,
Vicente Mantala, Jr., Bernaldo delos Santos, and Joven Pabustan were hired on various
dates from 1995 up to 1997 and worked as architects, draftsmen, operators, engineers,
and surveyors in the Subic Legend Resorts and Casino, Inc. (Legend) Project
Development Division on various projects. Hwa Puay, Flordeliza Maria Reyes Rayel,
and other corporate officers are impleaded in this case in their official capacities as
officers of Legend. On January 6, 1998, Legend sent notice to the Department of Labor
and Employment of its intention to retrench and terminate the employment of thirty-
four (34) of its employees, which include petitioners, in the Project Development
Division. Legend explained that it would be retrenching its employees on a last-in-first-
out basis on the strength of the updated status report of its Project Development
Division, as follows: (1) shelving of the condotel project until economic conditions in the
Philippines improve; (2) completion of the temporary casino in Cubi by mid-February
1998; (3) subcontracting the super structure work of Grand Legend to a third party; (4)
completion of most of the rectification work at the Legenda Hotel; (5) completion of the
temporary casino in Cubi; and (6) abolition of the Personnel and Administrative
Department of the Project Development Division and transfer of its function back to
Legend’s Human Resources Department. The following day, on January 7, 1998,
Legend sent the 34 employees their respective notices of retrenchment, stating the same
reasons for their retrenchment.

Issue: Whether or not redundancy is established in this case at bar?


Ruling: The Court ruled that Legend failed to establish redundancy. Retrenchment and
redundancy are two different concepts; they are not synonymous and therefore should
not be used interchangeably. Thus, simply put, redundancy exists when the number of
employees is in excess of what is reasonably necessary to operate the business. The
declaration of redundant positions is a management prerogative. The determination
that the employee’s services are no longer necessary or sustainable and therefore
properly terminable is an exercise of business judgment by the employer. The wisdom
or soundness of this judgment is not subject to the discretionary review of the Labor
Arbiter and NLRC. It is however not enough for a company to merely declare that
positions have become redundant. It must produce adequate proof of such redundancy
to justify the dismissal of the affected employees. In Panlilio v. NLRC, we said that the
following evidence may be proffered to substantiate redundancy: “the new staffing
pattern, feasibility studies/proposal, on the viability of the newly created positions, job
description and the approval by the management of the restructuring.” In another case,
it was held that the company sufficiently established the fact of redundancy through
“affidavits executed by the officers of the respondent PLDT, explaining the reasons and
necessities for the implementation of the redundancy program.” According to the CA,
Legend proved the existence of redundancy when it submitted a status review of its
project division where it reported that the 78-man personnel exceeded the needs of the
company. The report further stated that there was duplication of functions and
positions, or an over supply of employees, especially among architects, engineers,
draftsmen, and interior designers. Thus, in the same way, we held that the basis for
retrenchment was not established by substantial evidence, we also rule that Legend
failed to establish by the same quantum of proof the fact of redundancy; hence,
petitioners’ termination from employment was illegal.

Closure or Cessation of Business Operations

Eastridge Golf Club, Inc., v. Eastridge Golf Club, Inc., Labor Union-Super G.R. No.
166760, August 22, 2008

Facts: Petitioner employed respondents as kitchen staff in its Food and Beverage (F&B)
Department. Effective October 1, 1999, petitioner terminated the employment of
respondents on the ground that the operations of the F&B Department had been turned
over to concessionaire Mother's Choice Meat Shop & Food Services. Petitioner filed with
the Department of Labor and Employment (DOLE) an Establishment Termination
Report, stating that it laid off the respondents due to company
reorganization/downsizing and transfer of operations to a concessionaire. Respondents
filed with the National Labor Relations Commission (NLRC), Regional Arbitration
Branch, a complaint for illegal dismissal, unfair labor practice and payment of 13th
month pay. They claimed that their dismissal was not based on any of the causes
allowed by law, and that it was effected without due process. Petitioner denied
respondents' claims, pointing out that several months before their dismissal, it issued
various office memoranda informing respondents that, to minimize company losses, the
management decided to bid out a part of its operations, specifically the F&B
Department, to a concessionaire. The partial cessation of operations was bonafide.

Issue: Was there a valid cessation to justify the dismissal of n employee?

Ruling: Closure or cessation of business is the complete or partial cessation of the


operations and/or shut-down of the establishment of the employer. It is carried out to
either stave off the financial ruin or promote the business interest of the employer.
Unlike retrenchment, closure or cessation of business, as an authorized cause of
termination of employment, need not depend for validity on evidence of actual or
imminent reversal of the employer's fortune. Article 283 authorizes termination of
employment due to business closure, regardless of the underlying reasons and
motivations therefor, be it financial losses or not. In the case under review, the cause
invoked by petitioner in terminating the employment of respondents is not
retrenchment but cessation of a single aspect of its business undertaking, i.e., the F&B
Department. This is evident in the notices of termination it sent to respondents where
petitioner indicated that it had withdrawn from the direct operation of the F&B
Department and had transferred the management thereof to the concessionaire.[53]
Also, in the various office memoranda it posted, petitioner explained that the
underlying reason for the cessation of its F&B undertaking was that the economic
depression had affected its sales and operations and resulted in increased overhead
expenses and decreased incomes. Cessation of its F&B operations being the cause
invoked by petitioner to terminate the employment of respondents, it need not present
evidence of financial losses to justify such business decision. Thus, the Court agrees
with petitioner that the CA erred when it declared that, for lack of evidence of financial
losses, petitioner's cessation of its F&B operations was not a valid cause to terminate the
employment of respondents. The decision to close business is a management
prerogative exclusive to the employer, the exercise of which no court or tribunal can
meddle with, except only when the employer fails to prove compliance with the
requirements of Art. 283, to wit: a) that the closure/cessation of business is bona fide,
i.e., its purpose is to advance the interest of the employer and not to defeat or
circumvent the rights of employees under the law or a valid agreement; b) that written
notice was served on the employees and the DOLE at least one month before the
intended date of closure or cessation of business; and c) in case of closure/cessation of
business not due to financial losses, that the employees affected have been given
separation pay equivalent to ½ month pay for every year of service or one month pay,
whichever is higher. The decision to close business is a management prerogative
exclusive to the employer, the exercise of which no court or tribunal can meddle with,
except only when the employer fails to prove compliance with the requirements of Art.
283, to wit: a) that the closure/cessation of business is bona fide, i.e., its purpose is to
advance the interest of the employer and not to defeat or circumvent the rights of
employees under the law or a valid agreement; b) that written notice was served on the
employees and the DOLE at least one month before the intended date of closure or
cessation of business; and c) in case of closure/cessation of business not due to financial
losses, that the employees affected have been given separation pay equivalent to ½
month pay for every year of service or one month pay, whichever is higher.

Industrial Timber Corporation v. NLRC G.R. No. 107302 and 107306, June 10, 1997

Facts: Industrial Timber Corporation (ITC) is a corporation registered under Philippine


laws and is engaged in the business of manufacturing and processing veneer and
plywood products. It used to operate a veneer processing plant known as the Butuan
Logs Plant and a veneer and plywood processing plant known as the Stanply Plant.
Both plants occupied a single compound with a common point for ingress and egress
and were both leased from Industrial Plywood Group Corporation. Both plants had
also two (2) distinct bargaining units represented by separate labor unions and had
separate collective bargaining agreements with their respective principals. ITC Butuan
Logs Workers Union-WATU (Union) represented the rank and file employees of the
Butuan Logs Plant. Sometime in 1989, ITC decided to permanently stop and close its
veneer production at its Butuan Logs Plant “due to impending heavy financial losses
resulting from high production costs, erratic supply of raw materials and depressed
prices and market conditions for its wood products.” Accordingly, on November 9,
1989, ITC served a written notice to all its employees in the said plant and to the Butuan
District Office of the Department of Labor and Employment (DOLE) stating that
effective December 10, 1989 or thirty (30) days thereafter, it would cease operations at
said plant. After receiving the notice, the employees therein, through their union
representative, filed a formal objection to the intended shutdown. Consequently,
conciliation proceedings were conducted at the DOLE District Office pursuant to the
provisions of the Collective Bargaining Agreement (CBA) on grievances. The parties,
however, failed to settle their differences.
Issue: Was there an established valid cessation?

Ruling: Article 283 of the Labor Code clearly provides inter alia that the employer may
terminate the employment of his employees to prevent losses. Closure or cessation of
operations for economic reasons is, therefore, recognized as a valid exercise of
management prerogative. The determination to cease operations is a prerogative of
management which the State does not usually interfere with, as no business or
undertaking must be required to continue operating at a loss simply because it has to
maintain its workers in employment. Such an act would be tantamount to a taking of
property without due process of law. However, the burden of proving that such closure
is bona fide falls upon the employer. In this case, petitioner corporation presented the
analysis of an independent certified public accountant, showing in detail the imminent
losses it would suffer should it continue its operations. It is understandable that no
audited financial statements or other similar documents were presented as the company
is claiming impending future losses, not past or actual ones. Moreover, the fact that
petitioner company has ceased operations and has not resumed to do so only reinforces
its claim to a valid closure, not to mention the other established fact that its Stanply
Plant has also the capacity and capability to produce veneer, the product it solely
manufactured in its now closed plant. The foregoing notwithstanding, petitioner
corporation complied with the requirements mandated by law to effectuate valid
termination of employment on account of closure. Under the law, for an employer to
validly terminate the service of his employees under the aforesaid ground, he has to
comply with two (2) requirements, namely: (a) serving a written notice on the workers
and the DOLE at least one (1) month before the effective date of the closure and (b)
payment of separation pay equivalent to one (1) month pay or at least one-half (1/2)
month pay for every year of service, whichever is higher, with a fraction of at least six
(6) months to be considered one (1) whole year. The records bear out that petitioner had
sufficiently complied with the aforecited requirements. It informed its employees and
the DOLE District Office at Butuan of the termination of service of the employees
effective December 10, 1989 in a Letter dated November 9, 1989. The employees were,
likewise, informed of the availability for release of the funds for their separation pay
and other CBA benefits. Unfortunately, only 63 employees availed of the benefits. The
rest chose to file the instant action.

Disease or illness

Crayons Processing, Inc., v. Pula G.R. No. 167727, July 30, 2007
Facts: Petitioner Crayons Processing, Inc. (Crayons) employed respondent Felipe Pula
(Pula) as a Preparation Machine Operator beginning June 1993. On 27 November 1999,
Pula, then aged 34, suffered a heart attack and was rushed to the hospital, where he was
confined for around a week. Pula’s wife duly notified Crayons of her husband’s medical
condition. Upon his discharge from the hospital, Pula was advised by his attending
physician to take a leave of absence from work and rest for three (3) months.
Subsequently, on 25 February 2000, Pula underwent an Angiogram Test at the
Philippine Heart Center under the supervision of a Dr. Recto, who advised him to take
a two-week leave from work. Following the angiogram procedure, respondent was
certified as “fit to work” by Dr. Recto. On 11 April 2000, Pula returned to work, but 13
days later, he was taken to the company clinic after complaining of dizziness.
Diagnosed as having suffered a relapse, he was advised by his physician to take a leave
of absence from work for one (1) month. Pula reported back for work on 13 June 2000,
armed with a certification from his physician that he was “fit to work.” However, Pula
claimed that he was not given any post or assignment, but instead, on 20 June 2000, he
was asked to resign with an offer from Crayons of P12,000 as financial assistance. Pula
refused the offer and instead filed a complaint for illegal dismissal with prayer for
damages and the payment of holiday premium, 5 days service incentive leave pay, and
13th month pay for 1999. The complaint was filed against Crayons, Clothman Knitting
Corp., Nixon Lee, Paul Lee, Peter Su, and Ellen Caluag.

Issue: Was the dismissal valid on account of illness?

Ruling: For a dismissal on the ground of disease to be considered valid, two requisites
must concur: (a) the employee must be suffering from a disease which cannot be cured
within six months and his continued employment is prohibited by law or prejudicial to
his health or to the health of his co-employees; and (b) a certification to that effect must
be issued by a competent public health authority. The burden falls upon the employer
to establish these requisites and in the absence of such certification, the dismissal must
necessarily be declared illegal. As succinctly stressed in Tan v. NLRC, “it is only where
there is a prior certification from a competent public authority that the disease afflicting
the employee sought to be dismissed is of such nature or at such stage that it cannot be
cured within six (6) months even with proper medical treatment that the latter could be
validly terminated from his job.” Without the required certification, the characterization
or even diagnosis of the disease would primarily be shaped according to the interests of
the parties rather than the studied analysis of the appropriate medical professionals.
The requirement of a medical certificate under Article 284 cannot be dispensed with;
otherwise, it would sanction the unilateral and arbitrary determination by the employer
of the gravity or extent of the employee's illness and thus defeat the public policy in the
protection of labor.

Competent Public Health Authority

Cebu Royal Plant v. Minister of Labor G.R. No. L-58639, August 12, 1987

Facts: Ramon Pilones, private respondent, was employed on February 16, 1978 on a
probationary period of employment for six (6) months with petitioner CRP. After said
period, he underwent medical examination for qualification as regular employee but
the results showed that he is suffering from PTB minimal. Consequently, he was
informed of the termination of his employment by respondent since his illness was not
curable within 6 months. Pilones complained against his termination before the
Ministry of Labor which dismissed the same. The dismissal was reversed by the public
respondent who ordered the reinstatement and payment of back wages. Granting
reinstatement, the public respondent argues that Pilones was already a permanent
employee at the time of his dismissal and so was entitled to security of tenure. The
alleged ground for his removal, to wit, “pulmonary tuberculosis minimal,” was not
certified as incurable within six months as to justify his separation and that the
petitioner should have first obtained a clearance, as required by the regulations then in
force, for the termination of his employment. CRP claims that the private respondent
was still on probation at the time of his dismissal and so had no security of tenure. The
dismissal was necessary for the protection of the public health, as he was handling
ingredients in the processing of soft drinks which were being sold to the public.

Issue: Was the dismissal proper?

Ruling: No. The dismissal was not proper. Under Article 282 of the Labor Code, “an
employee who is allowed to work after a probationary period shall be considered a
regular employee.” Pilones was already on permanent status when he was dismissed
on August 21, 1978, or four days after he ceased to be a probationer. As such, he could
validly claim the security of tenure guaranteed to him by the Constitution and the
Labor Code. The petitioner claims it could not have dismissed the private respondent
earlier because the x-ray examination was made only on August 17, 1978, and the
results were not immediately available. That excuse is untenable. We note that when the
petitioner had all of six months during which to conduct such examination, it chose to
wait until exactly the last day of the probation period. The applicable rule on the
ground for dismissal invoked against him is Section 8, Rule I, Book VI, of the Rules and
Regulations Implementing the Labor Code which states that “the employer shall not
terminate his employment unless there is a certification by a competent public health
authority that the disease is of such nature or at such a stage that it cannot be cured
within a period of six (6) months even with proper medical treatment.” The record does
not contain the certification required by the above rule. Hence, dismissal was illegal. It
is also worth noting that the petitioner’s application for clearance to terminate the
employment of the private respondent was filed with the Ministry of Labor only on
August 28, 1978, or seven days after his dismissal. As the NLRC has repeatedly and
correctly said, the prior clearance rule (which was in force at that time) was not a
“trivial technicality.” It required “not just the mere filing of a petition or the mere
attempt to procure a clearance” but that “the said clearance be obtained prior to the
operative act of termination. Although we must rule in favor of his reinstatement, this
must be conditioned on his fitness to resume his work, as certified by competent
authority.

Procedural due process


1) Procedure to be observed in just causes termination
Twin-Notice Rule

Ruffy v. NLRC G.R. No. 84193, February 15, 1990

Facts: The complainant was employed in December 1977 by the respondent with the
salary of P37.31 a day. He was assigned in the Materials and Supply Section, Supply
and Warehousing Department of the respondent. His duties, among others, were to
verify and check incoming materials and supplies and issuing requisitioned materials
and supplies to authorized personnel of the various departments. On November 3,
1984, complainant issued twenty-five (25) sets of roller bearings valued at P15,650.00
covered by Material Issue Slip (M.I.S.) No. 121676 to a person who signed his name as
Role. These bearings were never received by the requisitioning section concerned. In the
investigation that ensued, it was gathered by the respondent that the bearings were sold
to factoria de Nasugbu for P8,250 by Anastacio Maulleon, Jr., an employee of the
respondent whose employment was terminated in connection with this case. In the
process, the complainant was asked whether Alfredo Role, also an employee of
respondent, was the same person who received said bearings. In reply, complainant
answered that he could not remember. Role on his part denied having received said
bearings. Consequently, on December 31, 1984, the complainant was dismissed from the
service for breach of trust, gross negligence and flagrant inefficiency with forfeiture of
all rights and privileges.

Issue: Was the procedure for dismissal proper?


Ruling: The law lays down the procedure prior to the dismissal of an employee. It need
not be observed to the letter, but at least, it must be done in the natural sequence of
notice, hearing and judgment. In the case at bar, there is no doubt that at the very
outset, that is, prior to investigation, the petitioner was informed that his services had
been terminated. He was made to air his side subsequently, it is true, yet the stubborn
fact remains that notwithstanding such an opportunity, if an opportunity it was, he had
been dismissed from the firm. The Court has held that the procedure under Batas Blg.
130 and the rules implementing it are conditions sine qua non, before dismissal may be
validly effected.

King of Kings Transport, Inc., v. Mamac G.R. No. 166208, June 29, 2007

Facts: Petitioner KKTI is a corporation engaged in public transportation and managed


by Claire Dela Fuente and Melissa Lim. Respondent Mamac was hired as bus conductor
of Don Mariano Transit Corporation (DMTC) on April 29, 1999. The KKTI employees
later organized the Kaisahan ng mga Kawani sa King of Kings (KKKK) which was
registered with DOLE. Respondent was elected KKKK president. Respondent was
required to accomplish a “Conductor’s Trip Report” and submit it to the company after
each trip. As a background, this report indicates the ticket opening and closing for the
particular day of duty. After submission, the company audits the reports. Once an
irregularity is discovered, the company issues an “Irregularity Report” against the
employee, indicating the nature and details of the irregularity. Thereafter, the
concerned employee is asked to explain the incident by making a written statement or
counter-affidavit at the back of the same Irregularity Report. After considering the
explanation of the employee, the company then makes a determination of whether to
accept the explanation or impose upon the employee a penalty for committing an
infraction. That decision shall be stated on said Irregularity Report and will be
furnished to the employee. Upon audit of the October 28, 2001 Conductor’s Report of
respondent, KKTI noted an irregularity. It discovered that respondent declared several
sold tickets as returned tickets causing KKTI to lose an income of eight hundred and
ninety pesos. While no irregularity report was prepared on the October 28, 2001
incident, KKTI nevertheless asked respondent to explain the discrepancy. In his letter,
[3] respondent said that the erroneous declaration in his October 28, 2001 Trip Report
was unintentional. He explained that during that day’s trip, the windshield of the bus
assigned to them was smashed; and they had to cut short the trip in order to
immediately report the matter to the police. As a result of the incident, he got confused
in making the trip report. On November 26, 2001, respondent received a letter
terminating his employment effective November 29, 2001. The dismissal letter alleged
that the October 28, 2001 irregularity was an act of fraud against the company.

Issue: Was there compliance with the procedural due process for dismissal?

Ruling: None. First, respondent was not issued a written notice charging him of
committing an infraction. The law is clear on the matter. A verbal appraisal of the
charges against an employee does not comply with the first notice requirement. In Pepsi
Cola Bottling Co. v. NLRC, the Court held that consultations or conferences are not a
substitute for the actual observance of notice and hearing. Also, in Loadstar Shipping
Co., Inc. v. Mesano, the Court, sanctioning the employer for disregarding the due
process requirements, held that the employee’s written explanation did not excuse the
fact that there was a complete absence of the first notice. Second, even assuming that
petitioner KKTI was able to furnish respondent an Irregularity Report notifying him of
his offense, such would not comply with the requirements of the law. We observe from
the irregularity reports against respondent for his other offenses that such contained
merely a general description of the charges against him. The reports did not even state a
company rule or policy that the employee had allegedly violated. Likewise, there is no
mention of any of the grounds for termination of employment under Art. 282 of the
Labor Code. Thus, KKTI’s “standard” charge sheet is not sufficient notice to the
employee. Third, no hearing was conducted. Regardless of respondent’s written
explanation, a hearing was still necessary in order for him to clarify and present
evidence in support of his defense. Moreover, respondent made the letter merely to
explain the circumstances relating to the irregularity in his October 28, 2001
Conductor’s Trip Report. He was unaware that a dismissal proceeding was already
being effected. Thus, he was surprised to receive the November 26, 2001 termination
letter indicating as grounds, not only his October 28, 2001 infraction, but also his
previous infractions.

IBM Philippines v. NLRC G.R. No. 117221, April 13, 1999

Facts: Private Respondent filed a complaint before the Arbitration Branch of the
Department of Labor and Employment (DOLE) for an illegal dismissal by herein
petitioner, private respondent claimed that he was not given the opportunity to be
heard and was summarily dismissed. Petitioner contend that he was given a chance or
warning to improve his attitude toward attendance but never did, and was duly
informed thru emails, it also pointed out that as an employee of IBM they are assigned
ID’s and passwords, employees may also respond/reply thru email by encoding his
message-response and admits also that the system automatically records the time and
date of each message was sent or received including the identification of the sender and
the receiver thereof. Petitioner attached to its position papers copies of print-outs which
allegedly contains computer message/entries sent by petitioner to private respondent
thru IBM’s internal computer system. Through this computer print-outs petitioner
sought to prove that private respondent was sufficiently notified of the charges and was
guilty thereof for failure to deny the same. Prior to the release of the labor arbiter’s
decision private respondent filed a “Motion to admit attached new evidence for the
complainant”. The Labor arbiter’s decision upheld the print-out attached by petitioner
as evidence and promulgated a resolution ordering petitioner to pay private respondent
salary from June 1 to August 31, 1990 excluding all benefits. Aggrieved with the
decision private respondent appealed to the NLRC which ordered reinstatement to
complainant to its former position with his seniority rights, backwages from August 31,
1990 in the amount of P40, 516, 65 a month including all its benefits and bonuses.
Hence, this petition.

Issue: Did NLRC commit grave abuse of discretion in holding that no just cause or due
process was observed in dismissing private respondent because computer print-outs
are inadmissible in evidence?

Ruling: Petitioner contend that in administrative /labor cases the technical rules on
evidence are not binding hence, the computer print-outs need not be identified nor
authenticated, same reason why private respondent was allowed to submit additional
evidences even after the case was deemed submitted for resolution. However, the
liberality of procedure in administrative actions is subject to limitations imposed by
basic requirements of due process; this procedural rule should not be construed as a
license to disregard certain fundamental evidenciary rules. The evidence presented
before us must be at least have a modicum of admissibility for it to be given some
probative value. The computer print-outs, which constitute only evidence of petitioners,
afford no assurance of their authenticity since they are unsigned. The liberal view in the
conduct of proceedings before administrative agencies, have nonetheless consistently
required some PROOF OF AUTHENTICITY OR RELIABILITY as condition for the
admission of documents. The procedural technicality and concerns are more paramount
principles and requirements of due process, which may not be sacrificed to speed or
expediency, Article 22 of the Labor Code which states that “…DUE PROCESS MUST
NEVER BE SUBORDINATED TO EXPEDIENCY OR DISPATCH”

Administrative Hearing

Asia Terminals, Inc., v. Marbella G.R. No. 149074, August 10, 2006
Facts: Asian Terminals, Inc. (Asian Terminals) petitioner, is a domestic corporation and
the exclusive provider of arrastre and stevedoring services at the Manila South Harbor.
Rodolfo G. Corvite, Jr., also a petitioner, is its President. Respondents are employees as
stevedores of Asian Terminals. It is not disputed that early in the evening of April 30,
1994, respondents and other stevedores, who formed one group, were assigned to
unload the cargo of the M/V Huang Jin Shua. The work of the group could not be
completed if one stevedore was absent. The parties, however, have opposing versions of
what transpired next. Officer-in-charge R.F. Salazar of Asian Terminals Legal
Department conducted an investigation of the incident. In his report and investigation
dated May 15, 1994, he found respondents liable for refusal to work penalized by
dismissal from the service considering that they committed the same offense for the
second time. On September 23, 1994, respondents were dismissed from the service. On
December 14, 1994, respondents filed with the Arbitration Branch of the National Labor
Relations Commission (NLRC) a complaint for illegal dismissal. Impleaded as
respondents were herein petitioners. On appeal, the NLRC, in its Decision dated
November 26, 1997, reversed the Labor Arbiter’s judgment. The NLRC found that
respondents were denied due process and that they did not refuse to work. It was an
officer of Asian Terminals who ordered respondents to stop working when no
personnel could replace the absent employee. Asian Terminals filed a motion for
reconsideration, but it was denied by the NLRC. Asian Terminals then seasonably filed
a petition for certiorari with this Court.

Issue: Was the dismissal proper?

Ruling: Here, Asian Terminals cited as a just cause respondents’ willful disobedience or
refusal to work. The offense of willful disobedience requires the concurrence of two (2)
requisites: (1) the employee’s assailed conduct must have been willful, that is
characterized by a wrongful and perverse attitude; and (2) the order violated must have
been reasonable, lawful, made known to the employee and must pertain to the duties
which he had been engaged to discharge. Settled is the rule that in an illegal dismissal
case, the onus probandi is on the employer to prove that the dismissal of an employee is
for a valid cause. In this case, we agree with the Court of Appeals that Asian Terminals
failed to prove willful disobedience on the part of respondents. Its own Position Paper
states that they did not refuse to work.

Shoppers Gain Supermart v. NLRC G.R. No. 110731, July 26, 1996
Facts: From 1982 to 1990, private respondents had worked in the Shoppers Gain
Supermarket in various capacities as "merchandiser, cashier, bagger, check-out
personnel, sales lady, printer/film and warehouseman" for at least one year each.
Private respondents were part of a pool of workers supplied by three (3) manpower
service companies under "labor-only" contracts. In December of 1990, due to an
unavoidable circumstance, petitioner constrained to stop its business and consequently
terminate its contract with the three (3) manpower service companies. Petitioner was
able to pay separation pays for its regular employees but not for private respondents. A
complaint for illegal dismissal was filed for which the Labor Arbiter rendered a
decision finding Shoppers Gain Supermarket guilty of labor-only contracting and
ordered it to pay separation pay and backwages to respondents. On appeal, the
National Labor Relations Commission affirmed said decision. Elevating the case to the
Supreme Court, petitioners raised the following grounds inter alia: (a) That for
employer-employee to exist, the following requirements must be satisfied, namely: (1)
selection and engagement of the employees; (2) the payment of wages; (3) the power of
dismissal; and (4) the power to control employees' conduct; and (b) Since the manpower
agencies themselves admitted per their respective position papers that they selected,
hired, paid, disciplined, dismissed and controlled the private respondents, it followed
that the latter are not the employees of the petitioner corporation but of the agencies
only.

Issue: Was the dismissal illegal?

Ruling: The Supreme Court held that what was controlling in the issue is the provisions
of Artcile 106 of the Labor Code and not that of Article 208. The former clearly defines
what constitute labor-only contractor as differentiated from a direct contractor,
including the legal effects of each, while the latter is merely for the purpose of
determining whether or not an employee is considered regular. Based on the provision
of Article 106, the Supreme Court ruled that the petitioner was indeed the direct
employer of private respondents and was therefore 0bliged to pay them separation pay.
The Supreme Court reasoned that since it is undeniable that the private respondents'
work as merchandisers, cashiers, baggers, check-out personnel, sales ladies,
warehousemen and so forth were directly related, necessary and vital to the day-to-day
operations of the supermarket and that their jobs involved normal and regular
functions in the ordinary business of the petitioner corporation, the provision of Article
106 clearly applied thus making the manpower agencies merely agents of petitioner
corporation. Consequently, private respondents are considered employees of petitioner
Shoppers Gain Supermart.
Caingat v. NLRC G.R. No. 154308, March 10, 2005

Facts: Petitioner Benardino A. Caingat was hired by respondent Sta. Lucia Realty and
Development, Inc. (SLRDI) as the General Manager of SLRDI’s sister companies, R.S.
Night Hawk Security and Investigation Agency, Inc., and R.S. Maintenance and
Services Inc. both organized to service the malls and subdivisions owned by SLRDI. In
connection with this, he was allowed to use 10% of the total payroll of respondent R.S.
Maintenance to defray operating expenses. Later, the Finance Manager discovered that
petitioner deposited company funds in the latter’s personal account and used the funds
to pay his credit card purchases, utility bills, trips abroad and acquisition of a lot in
Laguna. Thus, complainant received a memorandum stating that upon verification of
financial records, it was found that the latter have misappropriated company funds in
the sum of about P5, 000,000.00 and is hereby suspended from his duties as Manager of
the stated companies. Without conducting any investigation, respondent R.S.
Maintenance filed a complaint for sum of money and damages with prayer for writ of
preliminary attachment. Petitioner in turn filed a complaint for illegal dismissal against
the respondents.

Issue: Did respondents illegally dismiss petitioner?

Ruling: As firmly entrenched in our jurisprudence, loss of trust and confidence as a just
cause for termination of employment is premised on the fact that an employee
concerned holds a position where greater trust is placed by management and from
whom greater fidelity to duty is correspondingly expected. This includes managerial
personnel entrusted with confidence on delicate matters, such as the custody, handling,
or care and protection of the employer’s property. The betrayal of this trust is the
essence of the offense for which an employee is penalized. Management’s loss of trust
and confidence on petitioner was well justified. Private respondents had every right to
dismiss petitioner. Petitioner’s long period of disappearance from the scene and
departure for abroad before making a claim of illegal dismissal does not contribute to
its credibility. Nonetheless, while dismissal may truly be justified by loss of confidence,
the management failed to observe fully the procedural requirement of due process for
the termination of petitioner’s employment. Two notices should be sent to the
employee. The respondents only sent the first notice, gleaned from the memorandum.
There was no second notice.

Wenphil Corporation v. NLRC G.R. No. 80587, February 8, 1989


Facts: Private respondent was hired by petitioner on January 18, 1984 as a crew member
at its Cubao Branch. He thereafter became the assistant head of the Backroom
department of the same branch. At about 2:30 P.M. on May 20, 1985 private respondent
had an altercation with a co-employee, Job Barrameda, as a result of which he and
Barrameda were suspended on the following morning and in the afternoon of the same
day a memorandum was issued by the Operations Manager advising private
respondent of his dismissal from the service in accordance with their Personnel Manual.
The notice of dismissal was served on private respondent on May 25, 1985. Thus private
respondent filed a complaint against petitioner for unfair labor practice, illegal
suspension and illegal dismissal. After submitting their respective position papers to
the Labor Arbiter and as the hearing could not be conducted due to repeated absence of
counsel for respondent, the case was submitted for resolution. Thereafter a decision was
rendered by the Labor Arbiter on December 3, 1986 dismissing the complaint for lack of
merit.

Issue: Was there compliance with the procedural due process for dismissal?

Ruling: The failure of petitioner to give private respondent the benefit of a hearing
before he was dismissed constitutes an infringement of his constitutional right to due
process of law and equal protection of the laws. The standards of due process in judicial
as well as administrative proceedings have long been established. In its bare minimum
due process of law simply means giving notice and opportunity to be heard before
judgment is rendered.

Maneja v. NLRC G.R. No. 124013, June 5, 1998

Facts: Petitioner Rosario Maneja worked with private respondent Manila Midtown
Hotel beginning January, 1985 as a telephone operator. She was a member of the
National Union of Workers in Hotels, Restaurants and Allied Industries (NUWHRAIN)
with an existing Collective Bargaining Agreement (CBA) with private respondent. In
the afternoon of February 13, 1990, a fellow telephone operator, Rowena Loleng
received a Request for Long Distance Call (RLDC) form and a deposit of P500.00 from a
page boy of the hotel for a call by a Japanese guest named Hirota Ieda. The call was
unanswered. The P500.00 deposit was forwarded to the cashier. In the evening, Ieda
again made an RLDC and the page boy collected another P500.00 which was also given
to the operator Loleng. The second call was also unanswered. Loleng passed on the
RLDC to petitioner for follow-up. Petitioner monitored the call. On February 15, 1990, a
hotel cashier inquired about the P1,000.00 deposit made by Ieda. After a search, Loleng
found the first deposit of P500.00 inserted in the guest folio while the second deposit
was eventually discovered inside the folder for cancelled calls with deposit and official
receipts. When petitioner saw that the second RLDC form was not time-stamped, she
immediately placed it inside the machine which stamped the date “February 15, 1990.”
Realizing that the RLDC was filed 2 days earlier, she wrote and changed the date to
February 13, 1990. Loleng then delivered the RLDC and the money to the cashier. The
second deposit of P500.00 by Ieda was later returned to him. Petitioner and Loleng
thereafter submitted their written explanation. On March 20, 1990, a written report was
submitted by the chief telephone operator, with the recommendation that the offenses
committed by the operators concerned covered violations of the Offenses Subject to
Disciplinary Actions (OSDA): (1) OSDA 2.01: forging, falsifying official document(s),
and (2) OSDA 1.11: culpable carelessness - negligence or failure to follow specific
instruction(s) or established procedure(s). On March 23, 1990, petitioner was served a
notice of dismissal effective April 1, 1990. Petitioner refused to sign the notice and wrote
therein "under protest."

Issue: Whether or not the Labor Arbiter has jurisdiction over the illegal dismissal case.

Ruling: The procedure introduced in RA 6715 of referring certain grievances originally


and exclusively to the grievance machinery, and when not settled at this level, to a
panel of voluntary arbitrators outlined in CBAs does not only include grievances
arising from the interpretation or implementation of the CBA but applies as well to
those arising from the implementation of company personnel policies. No other body
shall take cognizance of these cases. x x x.’ (Sanyo v. Cañizares, 211 SCRA 361, 372)”
THE Court finds that the respondent Commission has erroneously interpreted the
aforequoted portion of our ruling in the case of Sanyo, as divesting the Labor Arbiter of
jurisdiction in a termination dispute. Moreover, the dismissal of petitioner does not fall
within the phrase “grievances arising from the interpretation or implementation of
collective bargaining agreement and those arising from the interpretation or
enforcement of company personnel policies,” the jurisdiction of which pertains to the
grievance machinery or thereafter, to a voluntary arbitrator or panel of voluntary
arbitrators. It is to be stressed that under Article 260 of the Labor Code, which explains
the function of the grievance machinery and voluntary arbitrator, “(T)he parties to a
Collective Bargaining Agreement shall include therein provisions that will ensure the
mutual observance of its terms and conditions. They shall establish a machinery for the
adjustment and resolution of grievances arising from the interpretation or
implementation of their Collective Bargaining Agreement and those arising from the
interpretation or enforcement of company personnel policies.” Article 260 further
provides that the parties to a CBA shall name or designate their respective
representative to the grievance machinery and if the grievance is unsettled in that level,
it shall automatically be referred to the voluntary arbitrators designated in advance by
the parties to a CBA of the union and the company. It can thus be deduced that only
disputes involving the union and the company shall be referred to the grievance
machinery or voluntary arbitrators.

Eastern Overseas Employment Center, Inc. v. NagkakaisangxEmpleyadongWellcome-


DFA G.R. No. 149349, March 11, 2005

Facts: On July 20, 1990, [respondent] union NAGKAKAISANG EMPLEYADO NG


WELLCOME-DFA (NEW-DFA) filed a Petition for Certification Election with the
DOLE-NCR seeking to represent the bargaining unit comprised of all the regular rank-
and-file employees of [petitioner] company GLAXO-WELLCOME. Acting upon such
petition, the Med-Arbiter ordered that a Certification Election be conducted on
September 10, 1990. The election, however, resulted in a stand-off or a tie between ‘NO
UNION’ and ‘NEW-DFA’. As a consequence thereof, NEW-DFA filed an election
protest with the Med-arbitration Branch of the Department of Labor and Employment.
[Respondent] union claimed that its failure to obtain the required majority vote can be
ascribed to several acts of manipulation, interference and intimidation committed by
[petitioner] company GLAXO-WELLCOME prior to and during the conduct of the
certification elections. Perceiving the enumerated events to be unduly oppressive to
labor, [respondent] union NEW-DFA, Jossie De Guzman and Norman Cerezo lodged a
complaint before the Labor Arbiter against [petitioner] company, GLAXO-
WELLCOME, for unfair labor practice, illegal dismissal and illegal suspension.
According to the [respondent] union, the ‘massive electioneering and manipulative acts’
of GLAXO-WELLCOME prior to and during the certification election unduly interfered
with the workers’ right to self-organization and are constitutive of unfair labor practice.
NEW-DFA likewise averred that the new Car Allocation Policy adopted by the
company was intended to harass, retaliate and discriminate against union officers and
members. [Respondent] union also challenged the legality of the suspension and
dismissal of two of its officers, namely: Norman Cerezo and Jossie Roda de Guzman. It
argued that the suspension and dismissal were effected without any prior hearing.

Issue: Did petitioner observe due process in dismissing the employees?

Ruling: The CA affirmed the findings of the labor arbiter and the NLRC that the
termination of the employment of De Guzman and the suspension of Cerezo were
based on a “just cause.” These findings are not at issue here. The only question to be
determined is whether the notice and hearing requirements were complied with. To
stress, if the dismissal is based on a just cause under Article 282 of the Labor Code, the
employer must give the employee (1) two written notices and (2) a hearing (or at least,
an opportunity to be heard). The first notice is intended to inform the employee of the
employer’s intent to dismiss and the particular acts or omissions for which the dismissal
is sought. The second notice is intended to inform the employee of the employer’s
decision to dismiss. This decision, however, must come only after the employee has
been given a reasonable period, from receipt of the first notice, within which to answer
the charge; and ample opportunity to be heard with the assistance of counsel, if the
employee so desires. The twin requirements of (a) two notices and (b) hearing are
necessary to protect the employee’s security of tenure, which is enshrined in the
Constitution, the Labor Code and related laws. The notices to be given and the hearing
to be conducted generally constitute the two-part due process requirement of law that
the employer must accord the employee. In Kingsize Manufacturing Corporation v.
NLRC, the Court held that this requirement was “not a mere technicality but a
requirement of due process to which every employee is entitled to insure that the
employer’s prerogative to dismiss or lay-off is not abused or exercised in an arbitrary
manner.”

Development of Doctrines

Wenphil Corporation v. NLRC G.R. No. 80587 February 8, 1989, En Banc

Facts: Private respondent was hired by petitioner on January 18, 1984 as a crew member
at its Cubao Branch. He thereafter became the assistant head of the Backroom
department of the same branch. At about 2:30 P.M. on May 20, 1985 private respondent
had an altercation with a co-employee, Job Barrameda, as a result of which he and
Barrameda were suspended on the following morning and in the afternoon of the same
day a memorandum was issued by the Operations Manager advising private
respondent of his dismissal from the service in accordance with their Personnel Manual.
The notice of dismissal was served on private respondent on May 25, 1985. Thus private
respondent filed a complaint against petitioner for unfair labor practice, illegal
suspension and illegal dismissal. After submitting their respective position papers to
the Labor Arbiter and as the hearing could not be conducted due to repeated absence of
counsel for respondent, the case was submitted for resolution. Thereafter a decision was
rendered by the Labor Arbiter on December 3, 1986 dismissing the complaint for lack of
merit.

Issue: Was the dismissal proper without compliance with the requirements of due
process?
Ruling: The failure of petitioner to give private respondent the benefit of a hearing
before he was dismissed constitutes an infringement of his constitutional right to due
process of law and equal protection of the laws. 2 The standards of due process in
judicial as well as administrative proceedings have long been established. In its bare
minimum due process of law simply means giving notice and opportunity to be heard
before judgment is rendered. 3 The claim of petitioner that a formal investigation was
not necessary because the incident which gave rise to the termination of private
respondent was witnessed by his co- employees and supervisors is without merit. The
basic requirement of due process is that which hears before it condemns, which
proceeds upon inquiry and renders judgment only after trial. However, it is a matter of
fact that when the private respondent filed a complaint against petitioner he was
afforded the right to an investigation by the labor arbiter. He presented his position
paper as did the petitioner. If no hearing was had, it was the fault of private respondent
as his counsel failed to appear at the scheduled hearings. The labor arbiter concluded
that the dismissal of private respondent was for just cause. He was found guilty of
grave misconduct and insubordination. This is borne by the sworn statements of
witnesses. The Court is bound by this finding of the labor arbiter.

Serrano v. NLRC G.R. No. 117040, May 4, 2000, En Banc

Facts: Respondent Isetann Department Store moves for reconsideration of the decision
in this case insofar as it is ordered to pay petitioner full backwages from the time the
latter's employment was terminated on October 11, 1991 up to the time it is determined
that the termination of employment is for an authorized cause. The motion is opposed
by petitioner. The decision is based on private respondent's failure to give petitioner a
written notice of termination at least thirty (30) days before the termination of his
employment as required by Art. 283 the Labor Code. Kycalrâ In support of its motion,
private respondent puts forth three principal arguments, to wit: (1) that its failure to
give a written notice to petitioner at least thirty (30) days in advance in accordance with
Art. 283 of the Labor Code is not in issue in this case because, as a matter of fact, it gave
its employees in the affected security section thirty (30) days pay which effectively gave
them thirty (30) days notice, and petitioner accepted this form of notice although he did
not receive payment; (2) that payment of thirty (30) days pay in lieu of the thirty (30)
days prior formal notice is more advantageous to an employee because instead of being
required to work for thirty (30) days, the employee can look for another job while being
paid by the company; and (3) that in any event the new ruling announced in this case
should only be applied prospectively.

Issue: Was the dismissal valid?


Ruling: The decision in Columbia Pictures does not mean that if a new rule is laid down
in a case, it should not be applied in that case but that said rule should apply
prospectively to cases arising afterwards. Private respondent's view of the principle of
prospective application of new judicial doctrines would turn the judicial function into a
mere academic exercise with the result that the doctrine laid down would be no more
than a dictum and would deprive the holding in the case of any force. Indeed, when the
Court formulated the Wenphil doctrine, which we reverse in this case, the Court did not
defer application of the rule laid down imposing a fine on the employer for failure to
give notice in a case of dismissal for cause. To the contrary, the new rule was applied
right then and there. For that matter, in 20th Century Fox Film Corp. v. Court of
Appeals the Court laid down the rule that in determining the existence of probable
cause for the issuance of a search warrant in copyright infringement cases, the court
must require the production of the master tapes of copyrighted films in order to
compare them with the "pirated" copies. The new rule was applied in opinion of the
Court written by Justice Hugo E. Gutierrez, Jr. in the very same case of 20th Century
Fox in which the new requirement was laid down. Where the new rule was held to be
prospective in application was in Columbia Pictures and that was because at the time
the search warrant in that case was issued, the new standard had not yet been
announced so it would be unreasonable to expect the judge issuing the search warrant
to apply a rule that had not been announced at the time. A good illustration of the scope
of overruling decisions is People v. Mapa, where the accused was charged with illegal
possession of firearms. The accused invoked the ruling in an earlier case that
appointment as a secret agent of a provincial governor to assist in the maintenance of
peace and order sufficiently put the appointee in the category of a "peace officer" equal
to a member of the municipal police authorized under §879 of the Administrative Code
of 1917 to carry firearms. The Court rejected the accused's contention and overruled the
prior decision in People v. Macarandang on the ground that §879 of the Administrative
Code of 1917 was explicit and only those expressly mentioned therein were entitled to
possess firearms. Since secret agents were not among those mentioned, they were not
authorized to possess firearms. Although in People v. Jabinal the Court refused to give
retro active effect to its decision in Mapa, because the new doctrine "should not apply to
parties who had relied on the old doctrine and acted in good faith thereon" and, for this
reason, it acquitted the accused of illegal possession of firearms, nonetheless it applied
the new ruling (that secret agents of provincial governors were not authorized to
possess firearms) in the very case in which the new rule was announced and convicted
the accused. In the case at bar, since private respondent does not even claim that it has
relied in good faith on the former doctrine of Wenphil and its progeny Sebuguero v.
NLRC, there is no reason not to apply the new standard to this case.
Agabon v. NLRC G.R. No. 158693, November 17, 2004, En Banc

Facts: Private respondent Riviera Home Improvements, Inc. is engaged in the business
of selling and installing ornamental and construction materials. It employed petitioners
Virgilio Agabon and Jenny Agabon as gypsum board and cornice installers on January
2, 1992 until February 23, 1999 when they were dismissed for abandonment of work.
Petitioners then filed a complaint for illegal dismissal and payment of money claims
and on December 28, 1999, the Labor Arbiter rendered a decision declaring the
dismissals illegal and ordered private respondent to pay the monetary claims.

Issue: Was the termination valid?

Ruling: The rule thus evolved: where the employer had a valid reason to dismiss an
employee but did not follow the due process requirement, the dismissal may be upheld
but the employer will be penalized to pay an indemnity to the employee. This became
known as the Wenphil or Belated Due Process Rule. On January 27, 2000, in Serrano, the
rule on the extent of the sanction was changed. We held that the violation by the
employer of the notice requirement in termination for just or authorized causes was not
a denial of due process that will nullify the termination. However, the dismissal is
ineffectual and the employer must pay full backwages from the time of termination
until it is judicially declared that the dismissal was for a just or authorized cause. The
rationale for the re-examination of the Wenphil doctrine in Serrano was the significant
number of cases involving dismissals without requisite notices. We concluded that the
imposition of penalty by way of damages for violation of the notice requirement was
not serving as a deterrent. Hence, we now required payment of full backwages from the
time of dismissal until the time the Court finds the dismissal was for a just or
authorized cause. Serrano was confronting the practice of employers to “dismiss now
and pay later” by imposing full backwages. We believe, however, that the ruling in
Serrano did not consider the full meaning of Article 279 of the Labor Code which states:
ART. 279. Security of Tenure. – In cases of regular employment, the employer shall not
terminate the services of an employee except for a just cause or when authorized by this
Title. An employee who is unjustly dismissed from work shall be entitled to
reinstatement without loss of seniority rights and other privileges and to his full
backwages, inclusive of allowances, and to his other benefits or their monetary
equivalent computed from the time his compensation was withheld from him up to the
time of his actual reinstatement. This means that the termination is illegal only if it is
not for any of the justified or authorized causes provided by law. Payment of
backwages and other benefits, including reinstatement, is justified only if the employee
was unjustly dismissed.

Reinstatement to previous position or substantially equivalent position

Asian Terminals, Inc., v. Villanueva G.R. No. 143219, November 28, 2006

Facts: Respondents were employees of Marina Port Services, Inc. (MPSI) and members
of the Associated Workers Union of the Philippines (AWU). In a letter dated 9 June 1993
to MPSI, the AWU president sought the dismissal from service of respondents who
were expelled from AWU. On 11 June 1993, the MPSI issued a memorandum to
respondents terminating them effective immediately pursuant to the closed-shop
provision of the MPSI-AWU Collective Bargaining Agreement. Respondents filed a
complaint for constructive illegal dismissal and unfair labor practice with the
Arbitration Branch of the NLRC.

Issue: Whether MPSI reinstated respondents to their former or equivalent positions.

Ruling: Reinstatement means restoration to a state or condition from which one had
been removed or separated. The person reinstated assumes the position he had
occupied prior to his dismissal. Reinstatement presupposes that the previous position
from which one had been removed still exists, or that there is an unfilled position which
is substantially equivalent or of similar nature as the one previously occupied by the
employee. Reinstatement means restoration to the former position occupied prior to
dismissal or to substantially equivalent position. Reinstatement does not mean
promotion. Promotion is based primarily on an employee’s performance during a
certain period. Just because their contemporaries are already occupying higher
positions does not automatically entitle respondents to similar positions.

Separation pay as financial assistance

PLDT v. NLRC G.R. No. L-80609, August 23, 1988, En Banc


Facts: Marilyn Abucay, a traffic operator of the Philippine Long Distance Telephone
Company, was accused by two complainants of having demanded and received from
them the total amount of P3,800.00 in consideration of her promise to facilitate approval
of their applications for telephone installation. 1 Investigated and heard, she was found
guilty as charged and accordingly separated from the service. She went to the Ministry
of Labor and Employment claiming she had been illegally removed. After consideration
of the evidence and arguments of the parties, the company was sustained and the
complaint was dismissed for lack of merit.

Issue: Should she be awarded with separation pay?

Ruling: The Court holds that henceforth separation pay shall be allowed as a measure of
social justice only in those instances where the employee is validly dismissed for causes
other than serious misconduct or those reflecting on his moral character. Where the
reason for the valid dismissal is, for example, habitual intoxication or an offense
involving moral turpitude, like theft or illicit sexual relations with a fellow worker, the
employer may not be required to give the dismissed employee separation pay, or
financial assistance, or whatever other name it is called, on the ground of social justice.
A contrary rule would, as the petitioner correctly argues, have the effect, of rewarding
rather than punishing the erring employee for his offense. And we do not agree that the
punishment is his dismissal only and that the separation pay has nothing to do with the
wrong he has committed. Of course it has. Indeed, if the employee who steals from the
company is granted separation pay even as he is validly dismissed, it is not unlikely
that he will commit a similar offense in his next employment because he thinks he can
expect a like leniency if he is again found out. This kind of misplaced compassion is not
going to do labor in general any good as it will encourage the infiltration of its ranks by
those who do not deserve the protection and concern of the Constitution.

Separation pay in lieu of reinstatement – Strained relation rule

Golden Ace Builders v. Talde G.R. No. 187200, May 5, 2010

Facts: In 1990, Golden Ace Builders hired Jose A. Talde (Talde) as a carpenter. In
February 1999, the owner-manager, Arnold Azul, stopped giving Talde work
assignment due allegedly to the unavailability of construction projects. Consequently,
Talde filed a complaint for illegal dismissal. The Labor Arbiter ruled in Talde’s favor
and ordered his immediate reinstatement without loss of seniority rights, with payment
of full backwages as well as premium pay for rest days, service incentive leave pay and
13th month pay. The company brought the case to the National Labor Relations
Commission (NLRC) for review. Pending such appeal, the company advised Talde to
report for work within 10 days from notice. Talde, however, manifested to the Labor
Arbiter that due to actual animosity between him and the company and threats to his
life and his family’s safety, he opted for payment of separation pay. The company
denied there was such an animosity. The NLRC later dismissed the company’s appeal.
The company’s appeal to the Court of Appeals was likewise dismissed. The Court of
Appeals’ decision attained finality. The monetary award, as recomputed by the NLRC’s
Fiscal Examiner, was approved by the Labor Arbiter who thereupon issued the writ of
execution. The company questioned the recomputation before the NLRC, arguing that
since Talde refused to report back to work as the company advised, he should be
deemed to have abandoned the same, thus, the re-computation should not be beyond 15
May 2001, the day he manifested his refusal to be reinstated. T he NLRC vacated the re-
computation, holding that since Talde did not appeal the Labor Arbiter’s decision
granting him only reinstatement and backwages, not separation pay in lieu of
reinstatement, he may not be afforded affirmative relief, and since he refused to go back
to work, he may recover backwages only up to 20 May 2001, the day he was supposed
to return to the job site. When Talde’s motion for reconsideration was denied by the
NLRC, he filed a petition for certiorari with the Court of Appeals. The Court of Appeals
set aside the NLRC findings and held that Talde was entitled to both backwages and
separation pay, even if separation pay was not granted by the Labor Arbiter, in view of
the strained relations between the parties. Consequently, the company filed a petition
for review on certiorari before the Supreme Court.

Issue: Whether or not Talde was entitled to separation pay in lieu of actual
reinstatement on account of strained relations between him and the company.

Ruling: An illegally dismissed employee is entitled to two reliefs: backwages and


reinstatement. The two reliefs are separate and distinct. When reinstatement is no
longer feasible because of strained relations between the employee and the employer,
separation pay equivalent to one (1) month salary for every year of service should be
awarded as an alternative. The payment of separation pay is in addition to payment of
backwages. In effect, an illegally dismissed employee is entitled to either reinstatement,
if viable, or separation pay if reinstatement is nolonger viable, and backwages. (Citing
Macasero v. Southern Industrial Gases Philippines, G.R. No. 178524; 30 January 2009)
Under the doctrine of strained relations, the payment of separation pay is considered an
acceptable alternative to reinstatement when the latter option is no longer desirable or
viable. On one hand, such payment liberates the employee from what could be a highly
oppressive work environment. On the other hand, it releases the employer from the
grossly unpalatable obligation of maintaining in its employ a worker it could no longer
trust. Strained relations must be demonstrated as a fact and must be supported by
substantial evidence showing that the relationship between the employer and the
employee is indeed strained as a necessary consequence of the judicial controversy. In
this case, the Labor Arbiter found that actual animosity existed between the owner-
manager Azul and Talde as a result of the filing of the illegal dismissal case. Such
finding, especially when affirmed by the appellate court as in the case at bar, is binding
upon the Supreme Court, consistent with the prevailing rules that the Supreme Court
will not try facts anew and that findings of facts of quasi-judicial bodies are accorded
great respect, even finality. The Court of Appeals erroneously computed his separation
pay from 1990 (when he was hired to 1999 (when he was unjustly dismissed), covering
a period of 8 years. He must be considered to have been in the service of the company
not only until 1999, but until 30 June 2005, the day he is deemed to have been actually
separated (his reinstatement having been rendered impossible) from the company, or
for a total of 15 years.

Leopard Security and Investigation Agency v. Quitoy G.R. No. 186344, February 20,
2013

Facts: Alongside Numeriano Ondong, respondents Tomas Quitoy, Raul Sabang and
Diego Morales were hired as security guards by petitioner Leopard Security and
Investigation Agency (LSIA). Thru its representative, Rogelio Morales, LSIA informed
respondents on 29 April 2005 of the termination of its contract with Union Bank which
had decided to change its security provider. Upon Morales’ instruction, respondents
went to the Union Bank Cebu Business Park Branch on 30 April 2005, for the turnover
of their service firearms to Arnel Cortes, Union Bank’s Chief Security Officer. On 3 May
2005, respondents and Ondong filed a complaint for illegal dismissal, unpaid 13th
month pay and service incentive leave pay (SILP), moral and exemplary damages as
well as attorney’s fees.

Issue: Are respondents entitled to separatin pay and reinstatement?

Ruling: As a relief granted in lieu of reinstatement, however, it consequently goes


without saying that an award of separation pay is inconsistent with a finding that there
was no illegal dismissal. Standing alone, the doctrine of strained relations will not
justify an award of separation pay, a relief granted in instances where the common
denominator is the fact that the employee was dismissed by the employer. Even in cases
of illegal dismissal, the doctrine of strained relations is not applied indiscriminately as
to bar reinstatement, especially when the employee has not indicated an aversion to
returning to work or does not occupy a position of trust and confidence in or has no say
in the operation of the employer’s business. Although litigation may also engender a
certain degree of hostility, it has likewise been ruled that the understandable strain in
the parties’ relations would not necessarily rule out reinstatement which would,
otherwise, become the rule rather than the exception in illegal dismissal cases. Absent
illegal dismissal on the part of LSIA and abandonment of employment on the part of
respondents, we find that the latter’s reinstatement without backwages is, instead, in
order. In addition to respondent’s alternative prayer therefor in their position paper,
reinstatement is justified by LSIA’s directive for them to report for work at its
Mandaluyong City office as early of 10 May 2005. As for the error ascribed the CA for
failing to correct the NLRC’s disregard of the evidence showing LSIA’s payment of
respondents’ SILP, suffice it to say that the NLRC is not precluded from receiving
evidence, even for the first time on appeal, because technical rules of procedure are not
binding in labor cases. Considering that labor officials are, in fact, encouraged to use all
reasonable means to ascertain the facts speedily and objectively, with little resort to
technicalities of law or procedure, LSIA correctly faults the CA for likewise brushing
aside the evidence of SILP payments it submitted during the appeal stage before the
NLRC.

Backwages and Separation Pay, Distinguished

Golden Ace Builders v. Talde G.R. No. 187200, May 5, 2010

Facts: In 1990, Golden Ace Builders hired Jose A. Talde (Talde) as a carpenter. In
February 1999, the owner-manager, Arnold Azul, stopped giving Talde work
assignment due allegedly to the unavailability of construction projects. Consequently,
Talde filed a complaint for illegal dismissal. The Labor Arbiter ruled in Talde’s favor
and ordered his immediate reinstatement without loss of seniority rights, with payment
of full backwages as well as premium pay for rest days, service incentive leave pay and
13th month pay. The company brought the case to the National Labor Relations
Commission (NLRC) for review. Pending such appeal, the company advised Talde to
report for work within 10 days from notice. Talde, however, manifested to the Labor
Arbiter that due to actual animosity between him and the company and threats to his
life and his family’s safety, he opted for payment of separation pay. The company
denied there was such an animosity. The NLRC later dismissed the company’s appeal.
The company’s appeal to the Court of Appeals was likewise dismissed. The Court of
Appeals’ decision attained finality. The monetary award, as recomputed by the NLRC’s
Fiscal Examiner, was approved by the Labor Arbiter who thereupon issued the writ of
execution. The company questioned the recomputation before the NLRC, arguing that
since Talde refused to report back to work as the company advised, he should be
deemed to have abandoned the same, thus, the re-computation should not be beyond 15
May 2001, the day he manifested his refusal to be reinstated. The NLRC vacated the re-
computation, holding that since Talde did not appeal the Labor Arbiter’s decision
granting him only reinstatement and backwages, not separation pay in lieu of
reinstatement, he may not be afforded affirmative relief, and since he refused to go back
to work, he may recover backwages only up to 20 May 2001, the day he was supposed
to return to the job site. When Talde’s motion for reconsideration was denied by the
NLRC, he filed a petition for certiorari with the Court of Appeals. The Court of Appeals
set aside the NLRC findings and held that Talde was entitled to both backwages and
separation pay, even if separation pay was not granted by the Labor Arbiter, in view of
the strained relations between the parties. Consequently, the company filed a petition
for review on certiorari before the Supreme Court.

Issue: Is respondent entitled to backwages?

Ruling: The basis for the payment of backwages is different from that for the award of
separation pay. Separation pay is granted where reinstatement is no longer advisable
because of strained relations between the employee and the employer. Backwages
represent compensation that should have been earned but were not collected because of
the unjust dismissal. The basis for computing backwages is usually the length of the
employee’s service while that for separation pay is the actual period when the employee
was unlawfully prevented from working. As to how both awards should be computed,
Macasero v. Southern Industrial Gases Philippines instructs: [T]he award of separation
pay is inconsistent with a finding that there was no illegal dismissal, for under Article
279 of the Labor Code and as held in a catena of cases, an employee who is dismissed
without just cause and without due process is entitled to backwages and reinstatement
or payment of separation pay in lieu thereof: Thus, an illegally dismissed employee is
entitled to two reliefs: backwages and reinstatement. The two reliefs provided are
separate and distinct. In instances where reinstatement is no longer feasible because of
strained relations between the employee and the employer, separation pay is granted.
In effect, an illegally dismissed employee is entitled to either reinstatement, if viable, or
separation pay if reinstatement is no longer viable, and backwages. The normal
consequences of respondents’ illegal dismissal, then, are reinstatement without loss of
seniority rights, and payment of backwages computed from the time compensation was
withheld up to the date of actual reinstatement. Where reinstatement is no longer viable
as an option, separation pay equivalent to one (1) month salary for every year of service
should be awarded as an alternative. The payment of separation pay is in addition to
payment of backwages.

Constructive dismissal

Uniwide Sales Warehouse Club v. NLRC G.R. No. 154503, February 29, 2008

Facts: Amalia P. Kawada (private respondent) started her employment with Uniwide
sometime in 1981 as a saleslady. Over the years, private respondent worked herself
within Uniwide’s corporate ladder until she attained the rank of Full Assistant Store
Manager with a monthly compensation of P13,000.00 in 1995. As a Full Assistant Store
Manager, private respondent’s primary function was to manage and oversee the
operation of the Fashion and Personal Care, GSR Toys, and Home Furnishing
Departments of Uniwide, to ensure its continuous profitability as well as to see to it that
the established company policies and procedures were properly complied with and
implemented in her departments. Sometime in 1998, Uniwide received reports from the
other employees regarding some problems in the departments managed by the private
respondent. Thus, on March 15, 1998, Uniwide, through Store Manager Apduhan,
issued a Memorandum addressed to the private respondent summarizing the various
reported incidents signifying unsatisfactory performance on the latter’s part which
include the commingling of good and damaged items, sale of a voluminous quantity of
damaged toys and ready-to-wear items at unreasonable prices, and failure to submit
inventory reports. Uniwide asked private respondent for concrete plans on how she can
effectively perform her job. In a letterdated March 23, 1998, private respondent
answered all the allegations contained in the March 15, 1998 Memorandum. On June 30,
1998, Apduhan sent another Memorandum seeking from the private respondent an
explanation regarding the incidents reported by Uniwide employees and security
personnel for alleged irregularities committed by the private respondent such as
allowing the entry of unauthorized persons inside a restricted area during non-office
hours, falsification of or inducing another employee to falsify personnel or company
records, sleeping and allowing a non-employee to sleep inside the private office,
unauthorized search and bringing out of company records, purchase of damaged home
furnishing items without the approval from superior, taking advantage of buying
damaged items in large quantity, alteration of approval slips for the purchase of
damaged items and abandonment of work. In a letter dated July 9, 1998, private
respondent answered the allegations made against her.

Issue: Was there constructive dismissal in this case?

Ruling: The test of constructive dismissal is whether a reasonable person in the


employee’s position would have felt compelled to give up his position under the
circumstances. It is an act amounting to dismissal but made to appear as if it were not.
In fact, the employee who is constructively dismissed may be allowed to keep on
coming to work. Constructive dismissal is therefore a dismissal in disguise. The law
recognizes and resolves this situation in favor of employees in order to protect their
rights and interests from the coercive acts of the employer. In the present case, private
respondent claims that from the months of February to June 1998, she had been
subjected to constant harassment, ridicule and inhumane treatment by Apduhan, with
the hope that the latter can get the private respondent to resign. The harassment
allegedly came in the form of successive memoranda which private respondent would
receive almost every week, enumerating a litany of offenses and maligning her
reputation and spreading rumors among the employees that private respondent shall
be dismissed soon. The last straw of the imputed harassment was the July 31, 1998
incident wherein private respondent’s life was put in danger when she lost
consciousness due to hypertension as a result of Apduhan’s alleged hostility and
shouting. The Court finds that private respondent’s allegation of harassment is a
specious statement which contains nothing but empty imputation of a fact that could
hardly be given any evidentiary weight by this Court. Private respondent’s bare
allegations of constructive dismissal, when uncorroborated by the evidence on record,
cannot be given credence.
Globe Telecom v. Florendo-Flores G.R. No. 150092, September 27, 2002

Facts: Petitioner GLOBE TELECOM, INC. (GLOBE) is a corporation duly organized and
existing under the laws of the Philippines. Petitioners Delfin Lazaro Jr. was its President
and Roberto Galang its former Director-Regional Sales. Respondent Joan Florendo-
Flores was the Senior Account Manager for Northern Luzon. On 1 July 1998 Joan
Florendo-Flores filed with the Regional Arbitration Branch of the National Labor
Relations Commission (NLRC) an amended complaint for constructive dismissal
against GLOBE, Lazaro, Galang, and Cacholo M. Santos, her immediate superior, Luzon
Head-Regional Sales. In her affidavit submitted as evidence during the arbitration
proceedings, Florendo-Flores bared that Cacholo M. Santos never accomplished and
submitted her performance evaluation report thereby depriving her of salary increases,
bonuses and other incentives which other employees of the same rank had been
receiving; reduced her to a house-to-house selling agent (person-to-person sales agent
or direct sales agent) of company products ("handyphone") despite her rank as
supervisor of company dealers and agents; never supported her in the sales programs
and recommendations she presented; and, withheld all her other benefits, i.e., gasoline
allowance, per diems, representation allowance, and car maintenance, to her extreme
pain and humiliation.

Issue: Was there constructive dismissal in this case?

Ruling: Where there is conflict between the dispositive portion of the decision and the
body thereof, the dispositive portion controls irrespective of what appears in the body.
[14] While the body of the decision, order or resolution might create some ambiguity in
the manner the court's reasoning preponderates, it is the dispositive portion thereof that
finally invests rights upon the parties, sets conditions for the exercise of those rights,
and imposes the corresponding duties or obligations. Hence, for the Court of Appeals to
have affirmed the assailed judgment is to adopt and uphold the NLRC finding of
abandonment and its award of full back wages to respondent as an "act of grace" from
petitioners. However, the Court believes this is not the proper view as the records
reveal that respondent was constructively dismissed from service. Constructive
dismissal exists where there is cessation of work because "continued employment is
rendered impossible, unreasonable or unlikely, as an offer involving a demotion in rank
and a diminution in pay." All these are discernible in respondent's situation. She was
singularly edged out of employment by the unbearable or undesirable treatment she
received from her immediate superior Cacholo M. Santos who discriminated against
her without reason - not preparing and submitting her performance evaluation report
that would have been the basis for her increased salary; not forwarding her project
proposals to management that would have been the source of commendation;
diminishing her supervisor stature by assigning her to house-to-house sales or direct
sales; and withholding from her the enjoyment of bonuses, allowances and other similar
benefits that were necessary for her efficient sales performance. Although respondent
continued to have the rank of a supervisor, her functions were reduced to a mere house-
to-house sales agent or direct sales agent. This was tantamount to a demotion. She
might not have suffered any diminution in her basic salary but petitioners did not
dispute her allegation that she was deprived of all benefits due to another of her rank
and position, benefits which she apparently used to receive.

You might also like